Die 10 Punkte der Klimarealisten – Nur Fiction oder fundierte Wissenschaft

Auf FOCUS Online (9.12.09. s. link siehe unten) hatten unter der Regie der Focus Redakteuin Christina Steinlein, die medien-bekannten und forschen Klimaforscher Prof. Marotzke, Prof. Claußen, Prof. Schellnhuber und Prof. Rahmstorf aus Ihrer Sicht versucht, anhand von 10 ausgesuchten Thesen der Klimarealisten, die Argumente der von dieser Gruppe so bezeichneten “Klimaskeptiker“ zu widerlegen. Dass es zuweilen in der Wissenschaft zielführender ist, Forschheit durch Nachdenken zu ersetzen und anstatt dem Begriff “Klimaskeptiker“, “Klimarealist“ angebrachter ist, zeigen nun die Entgegenhaltungen von EIKE.

1.)  These lt. Focus der Klimarealisten

„Das Klima wandelt sich schon immer“

dazu im Text von Focus Redakteurin Christina Steinlein

Die Erde ist 4,6 Milliarden Jahre alt, Eiszeiten folgten auf Warmzeiten und umgekehrt – alles ohne menschliches Zutun. Erstens heißt das aber nicht, dass der momentane Klimawandel nicht vom Menschen verursacht wird. Zweitens, so Jochem Marotzke vom Max-Planck-Institut für Meteorologie, sollte jeder, der diese These vertritt, sich mit den Größenordnungen vertraut machen: „Der heutige Klimawandel läuft zehnmal schneller ab als jeder andere zuvor – wenn man singuläre Ereignisse wie Asteroideneinschläge oder Supervulkanausbrüche ausschließt.“ Beispielsweise habe es 5000 Jahre gedauert, aus der letzten Eiszeit herauszukommen: Pro 1000 Jahre erwärmte sich das globale Klima um etwa ein Grad.

Heute findet der gleiche Temperaturanstieg in 100 Jahren, einem Zehntel der Zeit, statt. „Diese Zahlen sind sehr grob, aber das sind die Größenordnungen, von denen wir sprechen.“ Es ist ausgeschlossen, dass natürliche Prozesse einen so schnellen globalen Wandel bewirken.

EIKE zu Punkt 1 

„Der heutige Klimawandel läuft zehnmal schneller ab als jeder andere zuvor – wenn man singuläre Ereignisse wie Asteroideneinschläge oder Supervulkanausbrüche ausschließt.“

Dies ist definitiv eine Falschaussage, wie die Untersuchungen von Rahmstorf (2003, siehe Abbildung) zeigen. In dieser Untersuchung ist zu sehen, dass es Klimawandel mit einer Temperaturänderung von mehr als 6°C innerhalb von wenigen Jahrzehnten gab. In Spektrum der Wissenschaft, 11/09, S. 70 steht zu diesen krassen Klimawandel: “Die Schwankungen vollzogen sich bisweilen so schnell, dass einzelne Individuen sogar im Lauf ihres Lebens beobachten konnten, wie Pflanzen und Tiere, mit denen sie noch aufgewachsen waren, verschwanden und durch eine ihnen unvertraute Flora und Fauna ersetzt wurden.“ Und da die damaligen Individuen ein sehr kurzes Leben hatten (mit 30 Jahren war man alt), fand dieser Klimawandel von 6°C und mehr innerhalb von 2-3 Jahrzehnten statt. Selbst wenn die Temperaturreihen, die eine Erwärmung in den letzten 100 Jahren zeigen, nicht gefälscht wären, das IPCC redet heute lediglich von einem Temperaturanstieg von 0,7°C im 20. Jahrhundert.

 Aber wir brauchen gar nicht soweit zurückzugehen, um die obigen Aussage zu falsifizieren. 

Nehmen wir die letzten 100 Jahre: Steiler Temperaturanstieg seit 1920 bis 1942 (der steile Anstieg der Temperaturen nach 1960 ist, wie seit Climategate Gewissheit, durch manipulierte Temperaturdaten des GISS und CRU zustande gekommen. Temperaturen ohne Fälschungen zeigen keine nennenswerte Erwärmung). Nehmen wir weiter die letzten 200 Jahre: Der Temperaturanstieg seit 1815 bis 1825 (Angell et al. 1985: http://ams.allenpress.com/archive/1520-0450/24/9/pdf/i1520-0450-24-9-937.pdf) ist steiler als der (gefälschte) Anstieg seit 1960 bis heute nach CRU und GISS, bzw. IPCC.

Die folgende Abbildung, aus dem IPCC Bericht und statistisch im Auftrag von Lord Monkton analysiert, zeigt, dass es bereits in den letzten 150 Jahren, zwei vergleichbare Temperaturanstiege mit gleicher Steigungsrate gab, wie der, zwischenzeitlich als Fälschung entlarvte Temperaturanstieg seit den 1980-Jahre, den Herr Marotzke als einzigartig zu erkennen glaubt.

Fazit:  Zum einen scheinen die befragten “Klimaforscher“ um Herrn Marotzke ihre eigenen Untersuchungen nicht zu kennen(!) und zum anderen fügt sich der Temperaturanstieg im 20. Jahrhundert in die Temperaturschwankungen der letzten 200 Jahre ein und liegt innerhalb der natürlichen Variabilität.

2.) These lt. Focus der Klimarealisten

Die Ozeane sind schuld

dazu im Text von Focus Redakteurin Christina Steinlein

Das Kohlendioxid in der Atmosphäre stamme hauptsächlich aus den Ozeanen, argumentieren die Leugner.

„Fakt ist: Jährlich werden etwa 100 Gigatonnen Kohlenstoff zwischen Ozeanen und Atmosphäre ausgetauscht. Das ist ein sehr dynamischer Prozess. In manchen Regionen gibt der Ozean CO2 ab, in anderen nimmt er es auf“, erklärt Jochem Marotzke, Direktor des Max-Planck-Instituts für Meteorologie. Die Menschen blasen jährlich etwa sieben Gigatonnen in die Atmosphäre – das mag im Vergleich wenig klingen, aber was zählt, ist der Netto-Effekt: „Das Netto ist beim Austausch zwischen Ozean und Atmosphäre ungefähr bei null, ein bis zwei Gigatonnen nehmen die Meere sogar auf. Im Gegensatz dazu führen die Menschen der Atmosphäre kontinuierlich Kohlendioxid zu.“ Von den sieben Gigatonnen jährlich, die die Menschheit verursacht, bleibt etwa die Hälfte in der Atmosphäre.

„Netto gibt der Ozean gar kein CO2 ab, sondern nimmt seit Jahrzehnten welches auf“, stellt Stefan Rahmstorf vom Potsdam-Institut für Klimafolgenforschung klar. „Das ist durch rund 10 000 CO2-Messungen im Ozean von Forschungsschiffen belegt.“ In der Folge steigt die CO2-Konzentration in den Meeren, was zur Versauerung des Meerwassers führt.

Ein weiteres Argument führt Rahmstorf an: „Wir wissen ja, wie viele fossile Brennstoffe gefördert und verbrannt wurden und wie viel CO2 dabei in die Atmosphäre gelangt ist.“ Drittens hat Hans Suess in den 50er-Jahren eine Untersuchung durchgeführt, die belegt, dass das CO2 in der Atmosphäre nicht aus den Ozeanen stammt, sondern aus der Verbrennung von Erdöl, Erdgas oder Kohle. Anhand der Isotope der Kohlenstoffatome konnte der Forscher nachvollziehen, ob sie aus den Ozeanen oder aus der Verbrennung fossiler Stoffe stammen.

EIKE zu Punkt 2

Die Werte, die Herr Marotzke angibt, beruhen auf spekulativen Abschätzungen der CO2-Kreisläufe (was es mit der Genauigkeit der IPCC-Werte auf sich hat, ist spätestens seit den Himalaya-Gletschern bekannt), die einer wissenschaftlichen Überprüfung nicht standhalten: Die Abschätzung, die zur Anreicherung von anthropogenem CO2 in der Atmosphäre führt, beruht auf den 13C Isotopenanalysen. Hier liegt ein einfacher Rechenfehler vor (siehe E. Schulze), wie auch die Arbeit von Knorr bestätigt. Anthropogenes CO2 ist seit 200 Jahren gleichbleibend und nicht vom Rauschen zu unterscheiden, E. Schulze in "Über den Anteil am atmosphärischen Kohlendioxyd durch Verbrennungsvorgänge", August 2009, sowie Knorr, W. “Is the airborne fraction of anthropogenic CO2 emissions increasing? Geophysical Research Letters, 2009; 36 (21). Des Weiteren ist das Statement von Herrn Marotzke nicht hilfreich, der Bedeutung der Meere für die atmosphärische CO2-Konzentration auch nur annähernd gerecht zu werden, wie die folgende Abbildung zeigt.

Die Abbildung zeigt den aus Eisbohrkernen rekonstruierten Temperaturverlauf und dazu die atmosphärische CO2-Konzentration. Zum einen ist überdeutlich zu erkennen, dass das CO2 der Temperatur folgt und nicht umgekehrt und zum anderen handelt es sich um einen linearen Zusammenhang. Beide Kurven stehen in einem linearen Verhältnis zueinander, was in der folgenden Datenreihe, die einen längeren Zeitraum abdeckt, verdeutlicht wird (http://tucsoncitizen.com/wryheat/2009/06/24/al-gores-favorite-graph/). Jeder Wissenschaftler oder wissenschaftlich Interessierte weiß, wenn die Temperatur dem CO2-Gehalt der Atmosphäre folgen würde, müsste der Zusammenhang nicht linear, sondern logarithmisch sein! Es handelt sich demnach um Ausgasungserscheinungen des Ozeans (und der Böden). Im umgekehrten Fall (wird es kälter) nimmt der Ozean das CO2 auf. In der Physik ist bekannt, dass der CO2-Gehalt des Wassers neben dem Druck, vor allem von der Temperatur abhängt. Bei zunehmenden Temperaturen gast folglich gebundenes CO2 aus, wodurch die maßgebliche Rolle der Ozeane belegt ist. Noch ein paar Zahlen unsererseits: Nach Angaben der NASA sind in den Meeren ca. 40.000 GT C gespeichert, in den Böden nach den Angaben des IPCC gar die unvorstellbare Menge von 66.000.000 – 100.000.000 GT C. Aus beiden Senken gast CO2 temperaturabhängig aus. 

Die Aussage “Netto gibt der Ozean gar kein CO2 ab, sondern nimmt seit Jahrzehnten welches auf“ ist demnach eine Falschaussage.

Fazit:   Rahmstorf und Marotzke scheinen grundlegende physikalische Zusammenhänge aus dem Blickfeld gerückt zu sein. Die atmosphärische CO2-Konzentation folgt der Temperatur. Beide stehen in einem linearen Verhältnis zu einander, was nur den Rückschluss auf Ausgasungsvorgänge (Ozean/Land) zu lässt.

3.) These lt. Focus der Klimarealisten :

Vulkane stoßen viel mehr CO2 aus als menschliche Aktivität

dazu im Text von Focus Redakteurin Christina Steinlein

Vulkane und Gestein, vor allem in Vulkangebieten, geben tatsächlich CO2 ab. Die Menge ist schwer zu messen, „aber garantiert deutlich niedriger als die vom Menschen verursachte Menge“, stellt Marotzke klar. Das Umweltbundesamt geht davon aus, dass die CO2-Emissionen durch Vulkane ungefähr zwei Prozent der vom Menschen verursachten Emissionen ausmachen würde. Eine sehr viel höhere Emissionsrate aus Vulkanen und Gesteinen gilt als unwahrscheinlich: Vor der Industrialisierung war die CO2-Konzentration in der Atmosphäre relativ konstant.

Wäre der Vulkanismus der Hauptemittent, hätte der CO2-Gehalt der Atmosphäre auch vor der Industrialisierung kontinuierlich ansteigen müssen.

EIKE zu Punkt 3

Zu dieser Behauptung verweisen die Autoren auf die Arbeiten des Instituts für Angewandte-Geologie der Uni-Erlangen: Insgesamt stammt der Hauptteil des CO2 in der Erdatmosphäre aus dem Erdinneren, kommt aber nicht nur aus den Vulkanen, sondern tritt als sog. "juveniles Kohlendioxid" aus einer Vielzahl von Stellen, die teilweise recht unspektakulär aussehen. Es ist dies eine natürliche Folge der Plattentektonik,… Eine exakte Quantifizierung der gesamten, aus dem Erdinneren ausströmenden CO2-Mengen ist heute noch nicht möglich. Dies liegt einfach darin begründet, daß es unmöglich ist, an einer genügend großen Anzahl von Stellen (so z.B. an den Ozeanböden) kontinuierlich zu messen“ (http://www.angewandte-geologie.geol.uni-erlangen.de/klima1.htm). 

Noch ein paar weitere Zahlen, die die Dominanz der durch vulkanische Aktivität austretenden sog. Treibhausgase zeigt. Das Wissenschaftsmagazin “scinexx“ schreibt: “Denn egal ob an Land oder am Grund der Ozeane: Schlammvulkane stoßen jährlich Millionen Megatonnen an Methan aus und nehmen dadurch Einfluss auf das Weltklima.“ Bisher hatten Wissenschaftler den mittleren Gesamtanteil von Methan auf der Basis von 160 Schlammvulkanen aus 15 verschiedenen Gebieten mit 30 bis 35 Megatonnen/Jahr beziffert, was anhand der jüngsten Untersuchungen des Alfred Wegener Instituts und der obige Meldung von scinexx deutlich zu niedrig ist. Prof. Dr. Achim J. Kopf  (Marine Geotechnik am DFG Forschungszentrum der Universität Bremen) gibt folgende Werte an, die aus den Schlammvulkanen entweichen: 1,97 . 1011 bis 1,23 . 1014 m³ Methan/Jahr, wovon 4,66 . 107 bis 3,28 . 1011 m³ aus Oberflächenvulkanen stammen. Dies entspricht einer Gewichtsmenge von 141 – 88.000 Tg/Jahr (1Tg = 1 Million metrische Tonnen). 

Selbst von den etwa ca. 1.400 aktiven Vulkanen (ein Vulkan gilt als aktiv, wenn er in den letzten 10.000 Jahren ausgebrochen ist) sind derzeit erst 24*) vermessen, wobei die Bodenausgasungen um die Vulkane kaum berücksichtigt sind. 

*) Die Angaben schwanken je nach Literatur um ein paar Vulkane.

Fazit:   Die befragten “Klimaforscher“ sind ganz offensichtlich in ihren Erkenntnissen nicht auf der Höhe der Zeit. Die vulkanisch bedingten Ausgasungen sind nicht einmal zu 1% vermessen und damit bekannt und der Hauptanteil des atmosphärischen CO2-Gehalts stammt aus tektonischen Prozessen und ist somit weitgehend vulkanischen Ursprungs.

  

4.) These lt. Focus der Klimarealisten

Der Wasserdampf

dazu im Text von Focus Redakteurin Christina Steinlein 

„Klar ist Wasserdampf das wichtigste Treibhausgas“, sagt Martin Claußen, Mitdirektor des Max-Planck-Instituts für Meteorologie und Vorsitzender der Deutschen Meteorologischen Gesellschaft: „Er wirkt wie ein Verstärker.“ Der Wasserdampf ist aber nicht der Grund für die Erwärmung, sondern die Folge: Die Temperatur bestimmt, wie viel Wasserdampf in der Atmosphäre ist und nicht umgekehrt.

Im Gegensatz zum Kohlendioxid bleibt Wasserdampf meist nur wenige Tage in der Atmosphäre – und kehrt dann als Regen auf die Erdoberfläche zurück. Je wärmer allerdings die Atmosphäre ist, umso mehr Wasserdampf kann sie aufnehmen. Damit spielt der Wasserdampf tatsächlich eine sehr große Rolle beim natürlichen Treibhauseffekt. Die Wasserdampf-Moleküle in der Erdatmosphäre können wiederum Wärmestrahlung absorbieren und die Temperatur so weiter erhöhen.

EIKE zu Punkt 4

„Leider“ gibt es kein Wasserdampf-Feedback, auch wenn dies noch so sehr beschwört wird, wie die folgende Abbildung zeigt.

Die Abbildung zeigt die Änderung der globalen relativen Luftfeuchte für unterschiedliche Höhen anhand von NASA-Satellitendaten. Entgegen der Aussage von Herrn Claußen, ist keine Zunahme des Wasserdampfgehalts, sondern eine globale Abnahme zu verzeichnen. Die Studie kommt zu dem Schluss, dass es in den letzten 60 Jahren keinen Anhaltspunkt für eine treibhausgasbasierte Erwärmung gibt.

700 mbar  entspricht ca.   3.000 m-Höhe

600 mbar  entspricht ca.   4.000 m-Höhe

500 mbar  entspricht ca.   5.500 m-Höhe

400 mbar  entspricht ca.   7.000 m-Höhe

300 mbar  entspricht ca.   9.000 m-Höhe

Auch stellte die NASA bereits 2004 fest, dass die Klimamodelle den Einfluss von Wasserdampf überbewerten: “A NASA-funded study found some climate models might be overestimating the amount of water vapor entering the atmosphere as the Earth warms. They found the increases in water vapor were not as high as many climate-forecasting computer models have assumed.” (http://www.nasa.gov/centers/goddard/news/topstory/2004/0315humidity.html).

Zur Wirkung des sog. Wasserdampf-Feedback hat EIKE in ihrem Artikel “Das wissenschaftliche Ende des Treibhauseffekts“ (http://www.eike-klima-energie.eu/lesezeichen-anzeige/das-wissenschaftliche-ende-der-klimakatastrophe/) eingehend Stellung genommen. Wir zitieren: “Die unbefriedigende Situation änderte sich nun schlagartig mit zwei neuen Arbeiten, von denen die eine im Feb. 2009 bei Theoretical and Applied Climatology [2], die andere im Sept. 2009 bei Geophysical Research Letters erschien [1].

 G. Paltridge, A. Arking und M. Pook zeigten in der Zeitschrift Theoretical and Applied Climatology, dass die spezifische und relative Feuchte in der mittleren und oberen Troposphäre (oberhalb 850 hPa) im Gegensatz zu den Voraussagen der Klimamodelle des IPCC in den Jahren 1973 bis 2007 mit den steigenden Temperaturen in dieser Zeit abnimmt, was einer negativen Rückkopplung entspricht.” 

Und genau diese neg. Rückkopplung, die das IPCC geflissentlich unterlassen hat zu erwähnen, hat der Atmosphärenforscher am MIT, Prof. Dr. Lindzen, beschrieben. Eine deutsche Zusammenfassung findet sich unter (http://wetterjournal.wordpress.com/2009/05/21/die-argumente-der-klimaskeptiker-i-treibhauseffekt-und-wolken/)

Weitere Fachliteratur:

Paltridge, G., Arking, A. and Pook, M. 2009. Trends in middle- and upper-level tropospheric humidity from NCEP reanalysis data. Theoretical and Applied Climatology: 10.1007/s00704-009-0117-x. A comparison of tropical temperature trends with model predictions”, by Douglass, D.H., J.R. Christy, B.D. Pearson, and S.F. Singer, 2007 – International Journal of Climatology

Fazit:   In den Aussagen der befragten “Klimaforscher“ kommt zum Vorschein, dass Vermutungen mit der Realität, die längst das Postulat der Wasserdampfverstärkung widerlegte und einen Schritt weiter ist, verwechselt werden.

5.) These lt. Focus der Klimarealisten

Die Sonne beeinflusst das Klima

dazu im Text von Focus Redakteurin Christina Steinlein

Die Sonne ist der Energielieferant der Erde. Ihre Aktivität schwankt in einem ungefähr elfjährigen Zyklus. Es ist unbestritten, dass diese Schwankungen des Sonnenzyklus in der Vergangenheit das Klima verändert haben. Sie können aber aus zwei Gründen die Erwärmung im 20. Jahrhundert nicht erklären. Erstens: Seit Ende der 70er-Jahre wird die Sonnenstrahlung direkt von Satelliten aus gemessen. Diese Beobachtungen zeigten, dass der Unterschied zwischen Maximum und Minimum der Sonnenstrahlung während eines Zyklus etwa 0,1 Prozent der Strahlungsintensität ausmachen. „Die Stärke der Schwankungen ist zu gering. Die Strahlungswirkung der vom Menschen verursachten Treibhausgase ist inzwischen um ein Mehrfaches stärker“, Rahmstorf.

Und zweitens: Hätte der Zyklus der Sonne tatsächlich einen signifikanten Einfluss auf das Klima, dann ist es nicht zu erklären, dass sich die Erde in den letzten dreißig Jahren erwärmt und nicht im Takt mit der Sonne wieder abgekühlt hat. Eine aktuelle Studie, die im Magazin „Nature“ veröffentlicht wurde, kam gar zu dem Ergebnis, dass die Sonnenaktivität seit über 50 Jahren nicht zugenommen, in den letzten 20 sogar abgenommen hat.

EIKE zu Punkt 5

“Erstens… Die Stärke der Schwankungen ist zu gering. Die Strahlungswirkung der vom Menschen verursachten Treibhausgase ist inzwischen um ein Mehrfaches stärker“, Rahmstorf.

Dies ist eine Falschaussage, bezogen auf die solaren Aktivitätsschwankungen, wie die folgende Abbildung belegt.

Die Abbildung, Quelle: GISS, zeigt die sich ändernde Strahlungsabgabe (Variabilität, TOP = Top Of Atmosphere) der Erde im Zeitraum 1984 – 2005. Anmerkung: Dem GISS ist bei der Beschriftung der Zeitachse ein Fehler unterlaufen, es muss natürlich heißen, Year plus 1900. Der Hub zwischen Min. – Max. beträgt 7 W/m2.

Dazu noch die solare Aktivitätskurve, aus der ersichtlich wird, wie stark die Strahlungsabgabe der Erde mit der Sonnenaktivität korreliert – die Kurven sind nahezu in ihrer Form deckungsgleich.

 

“Und zweitens: Hätte der Zyklus der Sonne tatsächlich einen signifikanten Einfluss auf das Klima,… dass die Sonnenaktivität seit über 50 Jahren nicht zugenommen, in den letzten 20 sogar abgenommen hat.“ Rahmstorf

Auch dies ist leider eine Falschaussage. Der Hauptsonnenzyklus, der im Mittel 208-jährige de Vries/Suess-Zyklus hatte um 2003 sein langjähriges Maximum, als in Deutschland die Presse vom “Hitzesommer“ redete. In 2005 (der de Vries/Suess-Zyklus hat, wie der im Mittel 11-jährige, „untergeordnete“ Schwabe-Zyklus, oft eine Doppelspitze in der Aktivität), als überall von den “Hitzetoten“ in der Presse zu lesen war, sagte der Chef-Sonnenforscher der NASA, Hathaway: “Das Solare Minimum explodiert – Das Solare Minimum sieht seltsamerweise aus wie ein solares Maximum.“ (http://science.nasa.gov/headlines/y2005/15sep_solarminexplodes.htm

Rahmstorf hebt bei seinen Aussagen auf die Satellitendaten ab, die seit 1979 die solare Aktivität messen. Satelliten sind hierfür sicherlich das geeignete Instrument, nur muss der Satellit auch die „Augen“ dafür haben, die solare Aktivität zu sehen. Genau die hat er aber nicht! Die solaren Aktivitätsschwankungen werden im sog. TSI (Total Solar Irradiance) zusammengefasst. Das Messsystem ist aber nur in der Lage, den Wellenlängenbereich von 200 nm – 2.000 nm aufzunehmen, also nur einen Bruchteil des solaren Aktivitätsspektrums. Von “Total“ kann demnach gar keine Rede sein. Insbesondere die Berieche, die zwischen Sonnenminimum und Sonnenmaximum am stärksten variieren, werden gar nicht aufgezeichnet. So variiert im Schwabe-Zyklus die solare Aktivität im sichtbarem Bereich lediglich um <1%, im UVB-Bereich (205 nm) um ca. 7%, bei 150 nm bereits um 15%, im EVU (<150 nm) sogar um 35% und im Röntgenbereich gar um das 100-fache. Über die kosmische Strahlung*) (Svensmark-Effekt) und die Stratosphäre, übt die Sonne jedoch auch in dem nicht aufgezeichneten Energiebereich indirekt, bzw. direkt Einfluss auf das irdische Wettergeschehen und somit auf das Klima.

*)    Die solare Aktivität im Hauptsonnenzyklus, dem de Vries/Suess-Zyklus lässt sich denn auch nicht im TSI erkennen.

Die wegweisenden Arbeiten von Svensmark und Veizer, die Rahmstorf, Schellnhuber und Marotzke offensichtlich nicht bekannt sind (Höhe der Zeit?) finden Sie z.B. unter (http://www.sciencedirect.com/..). Die Welt widmete  Svensmark im Dezember 2009 folgenden Titel “Ein Physiker erschüttert die Klimatheorie“ (http://www.welt.de/wissenschaft/umwelt/article5528858/Ein-Physiker-erschuettert-die-Klimatheorie.html). Weiteres wissenschaftliches Material findet sich unter (http://www.pro-physik.de/Phy/leadArticle.do?laid=2547). Die Liste lässt sich beliebig fortsetzen.

Fazit:   Bei den Sonnenaktivitätskurven des TSI handelt es sich um eine Mogelpackung, bzw. um eine irreführende Bezeichnung. Die solare Aktivität hat bis zum Maximum des Hauptsonnenzyklus, dem im Mittel 208-jährige de Vries/Suess-Zykluss (ca. 2003) kontinuierlich zugenommen, synchron zur leichten Temperaturerhöhung, die die Satellitendaten ausweisen. Seit dem Rückgang der solaren Aktivität (de Vries/Suess-Zyklus) und dem Ausbleiben des 24. Schwabezyklus, rauschten die Erdtemperaturen in den Keller, wie wir dies gegenwärtig in weiten Teilen auf der Nordhalbkugel anhand des strengen und langen Winters zu spüren bekommen.

6.) These lt. Focus der Klimarealisten

Alle Klimavorhersagen sind Quatsch

dazu im Text von Focus Redakteurin Christina Steinlein 

Die Wissenschaftler werden häufig als Klimapropagandisten oder Klimahysteriker beschimpft. Gleichzeitig wird ihnen vorgeworfen, nur ungenaue Aussagen zu treffen.

Zum Beispiel die, dass sich die Temperatur bis Ende des Jahrhunderts um einen Wert zwischen 1,1 und 6,4 Grad erwärmen wird. Konkretere Prognosen lassen sich nicht treffen, sie wären unseriös. Die genannte Spanne stellt keine Unsicherheit dar. Sie entsteht aus der Arbeit der Klimaforscher, die eine ganze Reihe von Szenarien durchrechnen, nach dem Prinzip: „Wenn wir soundso viel CO2 ausstoßen, dann wird es soundso warm.“ So kommen die Differenzen zustande.

Dass die Vorhersagen aber alles andere als Unsinn sind, kann man der Landkarte unten entnehmen. Die einzelnen Grafiken des Weltklimarats IPCC zeigen die Prognosen von Modellen, die nur natürliche Einflüsse auf die Temperaturentwicklung berücksichtigt haben (blau) und von Modellen, die auch den Einfluss der Menschen miteinbezogen (rot). Die Linie zeigt jeweils die tatsächlichen Messwerte an.

Häufig behaupten Skeptiker auch, dass die Prognosen bislang alle zu pessimistisch ausgefallen sind. Auch das widerlegt die Karte des IPCC.

EIKE zu Punkt 6

Klimamodelle sind grundsätzlich nicht dazu geeignet, dass Geschehen für die nächsten Jahre auch nur annähernd richtig wiederzugeben, wäre die treffende Bezeichnung. Da sich jedes Modell/Vorhersage an der Realität messen muss, zeigt die folgende Abbildung, dass selbst für kurze Zeiträume, Klimamodelle fulminant an der Realität vorbei gehen. Die globalen Temperaturen fallen deutlich, das IPCC meldet anhand seiner Modelle, steigende Temperaturen.

Die Abbildung zeigt die vier Temperatur-Szenarien (B1, A1B, A2, Commitment) aus dem IPCC-Bericht 2007, die aus den IPCC-Klimamodellen ermittelt wurden. Das Szenario “Commitment“, welches die niedrigste Temperaturentwicklung angibt, geht dabei von einer gleich bleibenden CO2-Konzentration wie im Jahr 2000 aus! Bei den drei anderen Szenarien steigt indes die CO2-Konzentration weiter an, wie dies z.B. die Mauna Loa-Daten wiedergeben, Quelle: S.d.W. 02/09, “Kalt erwischt“, S. 21 – 22. Die roten Punkte geben die tatsächliche Temperaturentwicklung bis 2008 wieder (schwarze Punkte, sind die gemessenen Temperaturwerte bis 2005). Seit 2005 fallen die Temperaturen deutlich. Die grün, blau und violett schraffierten Bereiche zeigen die Szenarien der früheren IPCC-Vorhersagen (FAR = First Assessment Report, 1990; SAR = Second Assessment Report, 1995; TAR = Third Assessment Report, 2001).

Schellnhuber zeigte jüngst in beeindruckender Weise, was von Klimamodellen zu halten ist: Wir sind in der Lage, mit Modellen zu erkennen, dass bei fortschreitender Erwärmung in 100 Jahren alle Himalaya-Gletscher verschwunden sein dürften.” Diese Aussage machte der „Forscher“ Schellnhuber kürzlich im Bayerischen Rundfunk (BR5). Seit Glaciergate wissen wir, was es mit dem vermeintlichen Ergebnis aus Klimamodellen zum Himalaya-Gletscherschwund auf sich hat. Herr Schellnhuber schiebt dieses "katastrophale" Abschmelzen auf den Rußausstoß chinesischer Kohle-Kraftwerke. Kein weiterer Kommentar zu den Klimamodellen!! 

Zum Abschluss lassen wir das IPCC selbst zu den Klimamodellen Stellung nehmen. Im dritten Sachstandsbericht 2001 Sec. 14.2.2.2., S. 774 heißt es: ”… we are dealing with a coupled non-linear chaotic system, and therefore that the long-term prediction of future climate states is not possible”

Fazit:   Wissenschaft ist die Lehre der Exaktheit. Klimamodelle haben nichts mit Exaktheit, sondern lediglich etwas mit Vermutungen und Gewichtungen zu tun, durch welche die Endergebnisse in beabsichtigter Weise getrimmt werden. Vermutungen, bzw. Vermutungswissenschaftler sollten in der seriösen Wissenschaft keinen Einzug halten. Darüber hinaus kennen die “Klimaforscher“ ganz offensichtlich nicht die eigenen Klimamodelle, was den Anschein hat, dass die befragten “Klimaforscher“ nicht wissen, wovon sie reden!

7.) These lt. Focus der Klimarealisten

Die IPCC-Berichte sind politisch beeinflusst

dazu im Text von Focus Redakteurin Christina Steinlein

Das IPCC sei eine interessengeleitete Kungelrunde, behaupten Klimaskeptiker. „Das ist eine Verschwörungstheorie. Den Verschwörungstheoretikern kann man Fakten vorlegen, so viel man will, das kommt bei denen nicht an“, sagt Marotzke. „In der gesamten Geowissenschaft gibt es nichts Vergleichbares zu den IPCC-Berichten, nichts, das so sorgfältig begutachtet wird. An der Erstellung sind über 1000 Wissenschaftler beteiligt. Es gehen nur Studien in die Berichte ein, die zuvor veröffentlicht und damit von der Fachwelt beurteilt wurden. Die IPCC-Berichte, die aus diesen begutachteten Studien entstehen, werden wiederum begutachtet, und das mehrfach.“ Davon auszugehen, dass über 1000 Wissenschaftler die Wahrheit unterdrücken und nicht ein einziger ausschert, ist abwegig. „Jeder Wissenschaftler, der stichhaltig belegen könnte, dass die menschengemachte Erwärmung kein Problem ist, wäre auf einen Schlag der berühmteste Wissenschaftler der Welt. Das würde sich doch kein Forscher entgehen lassen“, argumentiert Marotzke weiter. 

Außerdem werden die Berichte des IPCC-Berichts anschließend noch zusammengefasst – und von Regierungsvertretern verabschiedet. Dass all die Regierungen mit den unterschiedlichsten Interessenlagen der Zusammenfassung des IPCC-Berichts Satz für Satz einstimmig zugestimmt haben, ist nur „deshalb möglich, weil an den enthaltenen wissenschaftlichen Ergebnissen beim besten Willen nicht zu rütteln ist. Die Bush-Administration hätte dem Bericht sonst wohl kaum zugestimmt“, stellt Rahmstorf klar.

  

EIKE zu Punkt 7 

Das IPCC sollte sich nicht hinter Politikern verstecken, sondern für seine Arbeiten und die seiner Leader selbst gerade stehen. Climategate ist real. Glaciergate ebenfalls und beides ist nur die Spitze des Eisbergs, wie die Temperaturmanipulationen diverser Institute, darunter das GISS, schließen lassen. Die Herren haben, um zu steigenden Temperaturen zu gelangen, einfach die Stationen heraus genommen, die fallende Temperaturen ausweisen und die Stationen, die in Ballungszentren liegen und somit steigende Temperaturen ausweisen, betrachtet (SURFACE TEMPERATURE RECORDS: POLICY DRIVEN DECEPTION? by Joseph D’Aleo and Anthony Watts). Ein Beispiel für die Güte der verbliebenen Messstationen. 

Fazit:   Climategate ist traurige Gewissheit. Der Rücktritt von Phil Jones, der den Betrug zugibt oder der Himalaya-Betrug der zugegeben wurde, beweisen: Es gibt ein Klimakartell von korrupten Klimaforschern, die Daten fälschen, Kollegen diffamieren und Begutachtungen manipulieren.

8.) These lt. Focus der Klimarealisten

Der Mensch ist nur für drei Prozent des CO2 verantwortlich

dazu im Text von Focus Redakteurin Christina Steinlein

Wer das sagt, vergleicht Äpfel mit Birnen. Die 97 Prozent CO2-Emissionen, für die angeblich die Natur zuständig ist, gehören zu einem geschlossenen Kreislauf: Menschen, Tiere und Pflanzen atmen Milliarden von Tonnen CO2 aus. Allerdings stehen auf der anderen Seite Pflanzen, die das CO2 (zusammen mit anderen Stoffen) durch die Photosynthese wieder in Blätter und Holz umwandeln. Der biologische Kohlenstoffkreislauf ist geschlossen. „Die CO2-Konzentration in der Atmosphäre war jahrtausendelang praktisch konstant und steigt erst an, seit wir dem System riesige Mengen an zusätzlichem Kohlenstoff aus fossilen Lagerstätten zuführen“, erklärt Rahmstorf. 

Diese vom Menschen verursachten Emissionen machen zwar tatsächlich etwa die oben genannten drei Prozent aus – dabei handelt es sich aber um Milliarden Tonnen Kohlendioxid, die dem eigentlich stabilen Kohlenstoffkreislauf netto hinzugefügt werden.

EIKE zu Punkt 8

Es gibt in der Natur keine stationären Prozesse, allenfalls periodisch-instationäre, wie z.B. den Tag/Nacht-Rythmus. Dies sollte einem Physiker, wie Rahmstorf eigentlich bekannt sein. 

Mit zunehmender Erwärmung gast nicht nur zusätzliches CO2 aus, welches vorher, z.B. in den Weltmeeren gebunden war (siehe unter Punkt 2). Auch das Leben überall auf der Welt nimmt an Vielzahl und Vielfalt zu. Nicht umsonst gelten tropische Regenwälder als das Dorado der Artenvielfalt. Das CO2 wird per Photosynthese nur zum Teil in Biomasse umgewandelt, ca. 30 % gehen als Hydrogencarbonat in die Böden (siehe Fachbücher der Pflanzenphysiologie). Ein nicht weiter wachsender Pflanzenbestand scheidet durch die Photosynthese (O2) und durch Atmung (CO2) die gleiche Menge ab, wie er für die Atmung (O2) und Photosynthese (CO2) verbraucht. Während der Wachstumsphase der Pflanzen wird im Zuge der Photosynthese so viel CO2 in organische Substanz eingebaut und O2 an die Atmosphäre abgegeben wie die Konsumenten dieser organischen Substanz (Bakterien, Pilze, Weidetiere, der Mensch, ja die Pflanzen selbst durch ihre Atmung) im Zuge der Verdauung, Atmung, Verrottung, Verwesung an CO2 freisetzen und an O2 verbrauchen. Dieser Prozess ist jedoch dynamisch und somit nicht für einen betrachteten Zeitabschnitt, wie z.B. 30 Jahre (internationale Definition für Klimazeitraum) konstant oder geschlossen. Mit zunehmendem Wachstum, z.B. durch bessere klimatische Bedingungen, wozu auch CO2 gehört, nicht umsonst ist CO2 ein effektiver Pflanzendünger, ergibt sich bildlich gesprochen, eine Spirale nach oben. Es gibt in einer offenen Atmosphäre mit ständiger Durchmischung keinen geschlossenen Kreislauf. CO2 schwankt mit der Temperatur und war nie konstant. Mit zunehmender Artenvielfalt steigen z.B. auch deren CO2-Emissionen (siehe unter Punkt 9).

Wer behauptet, natürliches CO2 wäre konstant, also ohne Einfluss auf die atmosphärischen CO2-Schwankungen, der hat entweder sein Metier verfehlt, oder sagt bewusst Falsches, wie die folgende Abbildung zeigt. 

Die Abbildung zeigt, wie sich laut NASA/Science die Artenvielfalt seit Beginn der 1980-Jahre steigerte, was folglich auch Auswirkungen auf die C-Kreisläufe der Natur hat. Von konstanten Kreisläufen kann also gar keine Rede sein. 

Fazit:   Der atmosphärische CO2-Gehalt war in der Atmosphäre nie konstant, sondern schwankte ganz erheblich. Anhand der gigantischen natürlichen CO2-Mengen, die allein durch eine Erwärmung freigesetzt werden und den CO2-Pegel triggern (Abbildung unter Punkt 2), gehen die wenigen GT anthropogenem CO2 im Hintergrundrauschen natürlicher Fluktuationen unter.

9.) These lt. Focus der Klimarealisten

Wir atmen zu viel

dazu im Text von Focus Redakteurin Christina Steinlein

Es stimmt, dass Menschen, Pflanzen und Tiere Kohlendioxid ausatmen – und zwar die unvorstellbare Menge von etwa 120 Milliarden Tonnen pro Jahr. Aber: Wie schon erwähnt, gehören diese zum geschlossenen Kohlenstoffkreislauf. Das ausgeatmete CO2 entsteht bei der Verbrennung von Nahrungsmitteln im Körper. Es ist also Kohlendioxid, das vorher der Atmosphäre durch die Photosynthese entzogen wurde.

Das Gleiche gilt für die Verbrennung von Holz: Wird immer nur so viel Holz verbrannt wie auch wieder aufgeforstet wird, trägt die Holzverbrennung nicht zusätzlich zum Treibhauseffekt bei.

EIKE zu Punkt 9 

Mit jedem Atemzug CO2 (385 ppm), dass wir einatmen, atmen wir die fast 150-fache Menge CO2 wieder aus. Jeder Mensch fast 1 Tonne und Jahr. Die von allen Menschen im Atmungsprozess erzeugte CO2-Menge ist beispielsweise mit der des weltweiten automobilen Verkehrs in etwa gleich zu setzen. Gottlob ist beim IPCC noch niemand mit der Forderung an die Menschheit herangetreten, dass diese, oder bestimmte Länder, jetzt nicht mehr atmen dürfen oder zumindest der Sport einzuschränken ist, bei dem bekanntermaßen die Atmung auf Hochtouren läuft.

Wie bereits geschildert, gibt es in der Natur weder stationäre Systeme, noch geschlossene Kreisläufe. Die Menge CO2, die die Menschheit im Jahr durch den Citrat-Zyklus produziert und in die Umwelt abgibt, ist vernachlässigbar, mit der Menge, die Bakterien oder Insekten als Beitrag zusteuern. Nach Brefeld beträgt allein die Biomasse der landlebenden Mikroorganismen 400 Milliarden Tonnen, die alle über den Citratzyklus Energie gewinnen und dabei CO2 freisetzen. Die Biomasse der Menschen liegt mit 0,4 Milliaden Tonnen bei 1/1000 und ist damit vernachlässigbar klein. Wie ein jeder weiß, steigen mit zunehmender Temperatur die Lebensbedingungen für Bakterien und sonst. Mikroorganismen. Nicht umsonst werden in der medizinischen Forschung die dort ablaufenden Prozesse mit dem Katalysator Wärme beschleunigt. D.h. aber, dass mit zunehmender Erwärmung die Biomasse entsprechend, teils explosionsartig steigt und damit natürlich auch deren Stoffwechsel und mit diesem die Atmung, wodurch sich die ausgebrachte CO2-Menge dieser Spezies deutlich erhöht.

Die genannten 120 GT beruhen denn auch nur auf den Abschätzungen des IPCC: 60 GT/Jahre durch Respiration von Pflanze und Tier und 60 GT/Jahr durch Bodenatmung (Bakterien). Diese Schätzung ist mehr als fragwürdig (auch ohne Glaciergate). Allein die Insektenatmung einer Spezies (Drosophila) beträgt  im halben Jahr 2.000 GT!!! (Fachbücher der Tierphysiologie). 

Die folgende Abbildung zeigt, wie sich dynamische Populationsmodelle verhalten. Es sind keine stationären oder in sich geschlossenen Systeme, sondern stark dynamische und offene.

Die Abbildung links zeigt den Unterschied zwischen begrenztem und exponentiellem Wachstum bei optimalen Bedingungen. Populationen bewegen sich, je nach Rahmenbedingungen zwischen diesen Kurven. Die rechte Abbildung zeigt anhand der Räuber/Beute-Beziehung das Schwingverhalten der Individuenanzahl. Durch die derzeitige Schwingung nach oben, zu mehr CO2 (auf Grund von Ausdünstungen aus Ozean/Land und erhöhter Biodervisität) wird das Pflanzenwachstum angetrieben, wie es die Untersuchungen der NASA (Abbildung unter Punkt 8) zeigt. 

Der weltweit anerkannte Prof. Veizer sagt hierzu folgendes: "CO2 reitet quasi Huckepack auf dem Wasserkreislauf, denn bei der Photosynthese müssen Pflanzen fast 1000 Wassermoleküle ausatmen, um ein einziges CO2-Molekül aufzunehmen." Wenn es wärmer wird, beschleunigt sich der irdische Wasserkreislauf, die Bioproduktivität erhöht sich, Bodenorganismen atmen vermehrt CO2 aus (http://www.pro-physik.de/Phy/leadArticle.do?laid=2547). 

Fazit:   Die befragten “Klimaforscher“ wandern auf ihnen unbekanntem Terrain und kommen dabei mächtig ins Straucheln. Deren genannte Zahlenangaben sind reine Spekulation (Vermutungen). Die Aussage, es gäbe stationäre Systeme, die somit keinen Einfluss auf die atmosphärische CO2-Konzentration hätten, ist wissenschaftlicher Mumpitz.

  

10.) These lt. Focus der Klimarealisten

Das wird alles nicht so schlimm, weil es Rückkopplungen gibt

dazu im Text von Focus Redakteurin Christina Steinlein 

Seit der Industrialisierung hat die Menschheit die CO2-Konzentration in der Atmosphäre um ein Drittel erhöht. Werden unvermindert Treibhausgase emittiert, wird sich der CO2-Gehalt bis etwa 2050 verdoppelt haben. Der jüngste Bericht des Weltklimarats nennt eine Spanne von 1,1 bis 6,4 Grad Celsius, je nachdem, wie sich die CO2-Emissionen entwickeln.

Schon eine leichte Erwärmung kann Vorgänge in Bewegung setzen, die den Treibhauseffekt verstärken oder abschwächen. Die Klimaforscher sprechen von positiven oder negativen Rückkopplungen. Es sind etliche dieser Rückkopplungen bekannt – das Problem ist, dass es sich dabei fast ausschließlich um solche Effekte handelt, die den Klimawandel beschleunigen. Die Wissenschaftler nennen solche Ereignisse „Kipppunkte“.

Die Erfahrung der Erdgeschichte zeigt, dass schon für Laien recht moderat klingende Klimaveränderungen immer massive Auswirkungen gehabt haben. Sie lehrt, dass in der Vergangenheit die Veränderung des Meeresspiegels pro Grad Celsius globaler Temperaturänderung zwischen 10 und 30 Metern lag. Wenn die Temperatur sich bis 2050 um zwei Grad erhöht, im Vergleich zu der Zeit vor der Industrialisierung, werden die Meeresspiegel langfristig um 20 Meter ansteigen. Venedig, New York und Bombay können wir vergessen“, warnt Hans-Joachim Schellnhuber, Leiter des Potsdam-Instituts für Klimafolgenforschung.

Ob er die Folgen des Klimawandels als „nicht so schlimm“ einschätzt, bleibe jedem selbst überlassen, sagt Marotzke. „Die Fakten sprechen jedenfalls für sich.“

EIKE zu Punkt 10

Die meisten Rückkopplungen sind spekulativ und nicht nachgewiesen, bzw. widerlegt (siehe Punkt 4). Nie sind Kipppunkte in der Natur beobachtet worden. Dagegen verlaufen alle geophysikalischen Vorgänge in Zyklen. Es gibt keinen beschleunigten Klimawandel (siehe oben und folgende Abbildung). Die Konzentration  des CO2 hat sich gemäß direkten Messungen um  ca. 2,6 % seit 1826 erhöht. Dies liegt innerhalb der Fehlergrenze und somit im natürlichen Rauschen (Beck E.G., Reconstruction of atmospheric CO2 Background levels since 1826, unpublished manuscript; ww.realCO2.de). Der vorindustrielle CO2-Wert lag deutlich höher, als das IPCC dies angibt und war wesentlich volatiler, wie aus dem Bericht der amerikanischen Akademie der Wissenschaft zu entnehmen ist, dortige Figur 2 (http://www.ncbi.nlm.nih.gov/pmc/articles/PMC129389/).

Prof. Augusto Mangini (Universität Heidelberg) hat in Zusammenarbeit mit Prof. Spötl von der Universität Innsbruck Stalagmiten der Spannagel-Höhle (Tirol) ausgewertet. Mittels weiterer Proxys (13C, Strontium, Barium, Yttrium, Blei, Schwefel, sowie ins Meer gespülte Sedimente) hat das Forscherteam seine Temperaturreihen abgesichert, so dass aus Sicht von Mangini et al. die Temperaturwerte (blaue Kurve) genauer, als mit Vergleichsmethoden (rote Kurve) angegeben werden konnte, Quelle: Wiener Zeitung vom 19.04.2009. Es fällt auf, dass die Temperaturwerte sehr viel variabler sind, als vom IPCC ausgewiesen. So stiegt im 4. Jahrhundert nChr. die Temperatur um 1,6°C in 120 Jahren. Hierzu ist der heutige Temperaturanstieg von 0,7°C im 20. Jahrhundert (der teilweise noch gefälscht ist) vergleichsweise gering.

Fazit:   Wissenschaft ist die Lehre der Exaktheit! Marotzke, Rahmstorf und Schellnhuber sind dieser Maxime offensichtlich nicht verpflichtet und betätigen sich lieber als Vermutungswissenschaftler, deren Thesen mit schriller Propaganda und immer phantastischeren Horrorszenarien zum Sieg verholfen werden soll. Wir möchten den o.g. Professoren deshalb die Aussage eines gestandenen und seriösen Wissenschaftlers, Herrn Prof. Dr. John S. Mattick, der in Spektrum der Wissenschaft 03/05 (in einem anderen Zusammenhang, die Aussage aber umso passender beim Klimaspektakel) folgende Worte fand, zum Nachdenken entgegen und mit auf den Weg geben:

“Unbewiesene Annahmen entwickeln manchmal ein Eigenleben, was besonders in der Naturwissenschaft unangenehme Folgen haben kann. Am Anfang geht es gewöhnlich nur um eine plausible oder einfache Erklärung der Befunde. Wenn sie keine offensichtlichen Mängel aufweist und ihr Wahrheitsgehalt nicht unmittelbar überprüfbar ist, mutiert sie aber oft zum Glaubenssatz. Neue Beobachtungen werden dann passend interpretiert. Irgendwann aber muss das Dogma unter der Last eklatanter Widersprüche zusammenbrechen.“  

Zusammengetragen von Dipl.-Ing. Michael Limburg, Dipl.-Biologe Ernst Georg Beck, Dipl.-Ing. Raimund Leistenschneider – EIKE

image_pdfimage_print

144 Kommentare

  1. Hallo NB,
    offensichtlich reden wir aneinander vorbei! Zweifach haben sie mit „Das ist klar!“ und „Auch klar!“ Ihr Verstehen bekundet. Und genau deshalb besteht nicht die geringste Notwendigkeit hier ein wissenschaftliches „paper“ zu präsentieren. Und eben aufgrund dieser allzu trivialen Wahrheit kann nicht die Rede von einer ewigen oberen Grenze von 0,2 Gt/a sein!

    Was Sauerstoffisotopen betrifft, wollte ich mit „trivial“ nur ausdrücken, dass mich Ihre überhebliche Haltung zu Herrn Leistenschneiders Aussage in punkto 18O nur aufgeregt hatte. Es ist auch nicht schlimm, dass nicht jeder weiß, ob Isotopenmessungen nun trivial sind oder nicht. Ihnen muss deshalb nämlich keine Perle aus der Krone fallen, wenn Sie im Umgang mit dieser Problematik über keinerlei Erfahrung und schon gar nicht aus Ihrer Berufspraxis verfügen, was ich mit meiner Äußerung indirekt so ausdrücken wollte.
    Mfg
    B. Hartmann

  2. Lieber Herr Hartmann,

    „Die von Ihnen angegebene obere Grenze des vulkanogenen CO2 in der Atmosphäre ist natürlich eine indirekte Ableitung und nicht anhand permanenter Messungen einer repräsentativen Menge von Vulkanen, Vulkanfeldern, untermeerischen Vulkanen, mittelozeanischen Rücken usw. ermittelt worden. Das ist Fakt! “

    Das ist klar!

    „Die jährliche Menge dieser Art von Co2-Ausstoß kann auch mal leicht infolge katastrophaler vulkanischer Ereignisse überschritten werden, was auch z.B. in der unmittelbaren Vergangenheit und natürlich auch viel früher öfter passiert ist. Das ist auch Fakt!“

    Auch klar!

    Wo ist jetzt Ihre Erklärung und die belegenden wissenschaftlichen Artikel, dass Ihre Behauptung:
    „das ändert aber trotzdem nichts an dem fakt, dass dieser CO2-Anteil nicht zuverlässig als Teil des gesamten Kohlenstoffkreislaufs eingeschätzt werden kann.“
    ein Fakt ist?

    Bitte eine exakte Erklärung nebst wissenschaftlichem paper, was dagegen spricht, dass 0,2 GtC/Jahr nicht als obere Grenze für die gegenwärtige mittlere C02-Emission in die Atmosphäre aus geologischen Quellen zu sehen ist. Dass dies kein direkter Messwert ist und Isotopenmessungen nicht trivial sind, weiß jeder.

  3. Hallo NB,
    Sie machen es sich manchmal aber auch sehr einfach! Die von Ihnen angegebene obere Grenze des vulkanogenen CO2 in der Atmosphäre ist natürlich eine indirekte Ableitung und nicht anhand permanenter Messungen einer repräsentativen Menge von Vulkanen, Vulkanfeldern, untermeerischen Vulkanen, mittelozeanischen Rücken usw. ermittelt worden. Das ist Fakt! Die jährliche Menge dieser Art von Co2-Ausstoß kann auch mal leicht infolge katastrophaler vulkanischer Ereignisse überschritten werden, was auch z.B. in der unmittelbaren Vergangenheit und natürlich auch viel früher öfter passiert ist. Das ist auch Fakt!
    Der von Ihnen empfohlene geologische ist in der Hinsicht auch nicht bahnbrechend und auch nicht ganz frei von Emotionen, was nicht unbedingt in einen wissenschaftlichen Artikel gehört: (sinngemäß) „Gasfreisetzungen infolge von Vulkanausbrüchen sind überhaupt nicht gefährlich (einmal abgesehen von der Gefährlichkeit der Explosion als solcher)“ Stimmt natürlich so nicht ganz und als ob jemand im Moment des Verfassens dahinter gestanden hätte und das Gegenteil behauptet hätte.
    Auch mit dem O-18 ist es nicht ganz so trivial, wie man denken könnte: „Es wurden Veränderungen in dem Verhältnis des leichten (16-O) und schweren Isotops (18-O) des Sauerstoffes in Schalen von Foraminiferen festgestellt, die aus verschiedenen Tiefenintervallen von Ozeansedimenten entnommen wurden. Es wird vorausgesetzt, dass das 18-O/16-O-Verhältnis in den Schalen der planktonischen und benthischen Foraminiferen diese Gehalte im Meerwasser während ihrer Lebenszeit widerspiegelt. Durchgeführte Untersuchungen untermauern, dass sich das Karbonat der Foraminiferenschalen, das für die Isotopenuntersuchung verwendet wurde, unter Gleichgewichtsbedingungen aus dem Meerwasser in den Schalen ablagert. Des Weiteren wurde festgestellt, dass sich mit einer Temperaturverringerung im Wasser der Gehalt des Sauerstoffisotops 18-O erhöht und der Gehalt des Isotops 16-O verringert. dabei existiert eine weitere Annahme: Das leichte Sauerstoffisotop 16-O verdunstet in größeren Mengen (schneller) von der Ozeanoberfläche und trägt zum Aufbau der Eisbedeckung des Festlandes bei. Ganz folgerichtig ist dabei, dass sich dadurch im Meerwasser die Konzentration des schweren Sauerstoffisotops 18-O erhöht. Jedoch welcher Faktor bei der Formierung der Isotopenzusammensetzung der Meereswässer bestimmend war (die Wassertemperatur oder das Ausbringen des leichten Isotops 16-O über die Verdunstung großer Wassermassen) bleibt unklar.(Frage: Entweicht 16-O als H216-O oder als 16-O2?)Angesichts dessen wird gefolgert, dass die Veränderungen der 18-O-Gehalte in den Schalen der planktonischen und benthischen Foraminiferen bei konstanter Zusammensetzung und Salinität des Meerwassers die Temperaturänderungen der Wasserschichten widerspiegeln, in denen die Foraminiferen lebten. Dabei entspricht eine Änderung des 18-O-Gehalts um 1 % im Karbonat der Schalen einer Temperaturänderung des Wassers von 4 °C. Dementsprechend muss konstatiert werden, dass die methodologische Grundlage der Interpretation der Resultate der Sauerstoffisotopenuntersuchungen bei weitem nicht vollkommen ist, zumal die Rolle der Einflussfaktoren auf die 18-O-Konzentration in den Meeresorganismen unklar bleibt wie beispielsweise die primäre Isotopenzusammensetzung der Wässer, Temperaturschwankungen, Salinität, ökologische Lebensbedingungen der Mikrofauna und diagenetische Prozesse. Außerdem wirken sich Temperaturschwankungen der Ozeanoberfläche nicht im vollen Maße und nicht immer in den bodennahen Schichten aus. Und umgekehrt – die Temperaturparameter der Wassermassen als solche sind nicht immer abhängig von globalen planetaren Klimasituationen, sondern können durch das Bodenrelief, durch die konkrete Lage, den Charakter von Strömungen usw. bestimmt werden. Trotzdem sind die Schwankungen des 18-O/16-O-Verhältnisses in den Meeresbodenprofilen (in den Schalen der Foraminiferen) hinreichend deutlich ausgeprägt und werden als Zeugnis über Temperaturerhöhungen und Temperaturverringerungen der oberflächlichen und bodennahen Ozeanwässer angesehen. Daraus werden auch Rückschlüsse auf die Lufttemperatur gezogen.“
    Also, z. B. auch das – nämlich meistens ist doch nicht alles ganz klar. Für mich zumindest.

  4. #122: Raimund Leistenschneider sagte am Montag, 08.02.2010, 11:53:

    ‚Sehr geehrter Herr Müller, #93

    „Will der Autor ernsthaft behaupten, die klimatischen Schwankungen in der Größenordnung von Jahrzehnten seien die Zacken dieser Kurve??“‘

    Da ist mir beim editieren ein Lapsus unterlaufen und ich habe nicht bemerkt, dass ein Teil des Originaltextes im verdeckten teil des Editierfeldes stand. Und da überspringen Sie leider den teil mit meinen sachlichen Anmerkungen und Fragen und greifen eine Aussage auf, die Sie schonmal viel … hmm, wie nenne ich das jetzt, am besten gar nicht … beantwortet haben. Da sich zu meinen eigentlichen Fragen nichts getan hat und das Diagram immer noch auf eine falsche Quelle verweist, wiederhole ich einfach nochmal meinen text, auch wenn das nicht viel Sinn haben wird, da Sie sich ja anscheinend doch schon bewusst sind, dass DO-Events lokalen Charakter haben.

    Der Originaltext ohne die ausversehen mitzitierten Teile:

    64: Raimund Leistenschneider sagt:
    „Wissenschaft fängt bei Transparenz und Seriosität an.“‚

    Vielleicht können Sie Ihre Artikel ja in der Hinsicht verbessern, indem Sie die Quellen Ihrer Graphen angeben. Die Abbildung 1 ziehrt die Beschriftung „Rahmstorf 2003“. Ich nehme an, Sie meinen den Artikel „Timing of abrupt climate change: A precise clock“. Dort findet sich allerdings der Graph nicht. Wie Sie als Reaktion auf Klaus_V nachgereicht haben, stammt der Pfeil und das 6K Event aus einem Vortrag von jemand anderem. Dort ist allerdings auf der angegebenen Folie kein Bezug zur Zeit hergestellt. Das Diagram stammt also entweder aus einer anderen Quelle oder ist selbstgemacht.

    Was allerdings nichts daran ändert, dass es Dansgaard-Oeschger events mit ihrem steilen Anstieg der Temperatur gab. Nur ist anscheinend immer noch strittig, ob es ein globales Erwärmungsevent war oder ein lokales. Ich habe mal ein wenig rumgelesen, und da scheint es verschiedene Meinungen zu geben. http://preview.tinyurl.com/yftcf8p erklärt ab 2:45 das DO-Event, postuliert aber, dass eine Erwärmung der Nordhälfte von einer Abkühlung der Südhälfte begleitet wurde, also nur Energie umverteilt wurde. Wikipedia liegt ähnlich, Rahmstorf 2003 macht keine Aussage. Kennen Sie ein Papier, dass das als globale Erwärmung kennzeichnet? Nur dann wäre es eine Widerlegung der Originalthese…

    MfG, Marvin, nur mal kurz reinschauend

  5. Lieber Herr Leistenschneider,

    meine Frage bezog sich auf Ihre Gegenüberstellung von lokalen und globalen Temperaturtrends. Was a priori nicht vergleichbar ist, wie man leicht durch Vergleich rezenter Daten sehen kann. Und damit Ihre Behauptung nicht belegt.

    Was soll das mit CO2 zu tun haben? Sie sitzen damit immer noch auf „Start“.

  6. Lieber Herr Hartmann,

    „danke für Ihren Hinweis auf den CO2-Artikel. das ändert aber trotzdem nichts an dem fakt, dass dieser CO2-Anteil nicht zuverlässig als Teil des gesamten Kohlenstoffkreislaufs eingeschätzt werden kann.“

    Behauptungen sind kein Fakt?
    Bitte eine exakte Erklärung nebst wissenschaftlichem paper, warum 0,2 GtC/Jahr nicht als obere Grenze für die gegenwärtige mittlere C02-Emission in die Atmosphäre aus geologischen Quellen zu sehen ist.

  7. Lieber NB,
    danke für Ihren Hinweis auf den CO2-Artikel. das ändert aber trotzdem nichts an dem fakt, dass dieser CO2-Anteil nicht zuverlässig als Teil des gesamten Kohlenstoffkreislaufs eingeschätzt werden kann. Wie sie angesichts dessen glauben können, dass sie eine obere Grenze dafür kennen bzw. ihr Vertrauen schenken ist mir ein Rätsel (s. auch Leistenschneiders Kommentar).
    Gruß
    B. Hartmann

  8. Lieber NF,
    ich wollte den Tread schon abschließen, möchte aber auch nochmal betonen, dass mir sehr viel an einem respektvollen Umgang liegt, auch wenn in der Sache die Meinungen hart bleiben.
    In diesem Sinne bis zum nächsten mal, falls ich Zeit finde wie heute…

  9. Sehr geehrter Herr NB,

    „Was offensichtlich ein Trugschluß ist. Denn wie man wissen sollte (siehe die D-O-Literatur oder IPCC 2007), zeigen die von Ihnen in Ihrem „Widerlegungsversuch“ gezeigten D-O-Temperaturrekonstruktionen von Suenje/PIK den lokalen bis regionalen Temperaturverlauf an Grönlands Küste, aber Marotzke spricht explizit von Trends im global Mittel – womit Ihre Argumentation nicht valide ist, und es für Sie heißt: Zurück zum Start! – q.e.d. ;-)“

    da Sie durch vermeintliche subjektive Feststellungen, gerne Resultate vorgeben möchten, was in der Wissenschaft ein zutiefst untypisches Verhalten ist – aber dies kennen wir ja von der gesamten sog. IPCC-Klima“elite“, haben Sie sich und das ganze Fundament der CO2-basierten Erwärmung/anthr. CO2-Ausbringung, zurück zum Anfang gesetzt.

    Die CO2-Steigerung basiert auf den sog. vorindustriellen CO2-Pegeln. Diese stammen aus Eisbohrkernuntersuchungen. Iher Anmerkung zu den DO-Ereignissen heißt denn auch, angewendet auf den CO2-Pegel, dass die vorindustriellen Werte, mit denen IPCC und Co. agieren, lediglich von der Arktis stammen, keinen globalen CO2-Pegel wiedergeben, wiedergeben können, da in der Arktis bekanntlich weder CO2-Senken, noch CO2-Emissionen in nennenswertem Umfang vorhanden sind. Ergo diese Werte auch nicht als (unabdingbarer) Ansatzpunkt gewählt werden können, zu behaupten, der vorindustrielle CO2-Pegel wäre über Jahrtausende konstant und niedrig gewesen. Also, alle IPCC-Forscher und deren Anhänger, zurück zum Anfang.

    Viele Grüße RL

  10. Ausdrückliche Entschuldigung, Herr Georgiew.

    Ich war etwas irritiert, weil wir doch zuletzt einen so respektvollen Umgang pflegten. Ich finde, dabei sollte es bleiben.
    Deshalb nochmals: Sorry

  11. Lieber NF

    nun unterliegen Sie einer kleinen Verwechslung, in 44 haben Sie einem Herrn Eber geantwortet, ich kann selbst rechnen. Das O2 sich relativ und absolut kaum verändert (Sie in 44) bedeutet im Zusammenhang, dass sich auch CO2 absolut kaum verändert, auch wenn Sie es anders sehen wollen.
    An alle N..
    Ich bleibe dabei, die Verwendung von ppm anstatt der gebräuchlichen %-Rechnung und der relativen (33%) anstatt der absoluten (0,01%)Konzentrationserhöhung gehört zur bewussten Angstmache(Pachauri:“Man muss nur richtig Panik machen, dann fließt das Geld schon“), die ganz gut funktioniert. Noch..

  12. Jetzt bin ich (NF) es wirklich, Herr Georgiew.

    Ich wollte in #44 nur hilfsbereit auf ihre Frage antworten. Was habe ich dort „zugegeben“??
    Nie im Leben, dass eine Erhöhung von 280ppm um weitere 100ppm vernachlässigbar sei, ganz im Gegenteil, das wäre eine Steigerung um ca. ein Drittel!
    Ich fühle mich falsch interpretiert und werde künftig auf solche Akte der Hilfsbereitschaft verzichten.

  13. NB,
    die 114 bezog sich auf 44 von NF, Sie haben darauf reagiert, daher meine Verwechslung. Nicht Sie, sondern NF hat in 44 im Zusammenhang mit O2 Gehalt indirekt die vernachlässigbar geringen CO2 Anstieg zugegeben. Mit Ihnen wollte ich nicht diskutieren, Ihr Fall ist hoffnungslos. Sollten Sie beide Nicks benutzen ist mir das auch gleich.

  14. Lieber Herr Leistenschneider,

    „Nun, der von Ihnen in #118 gezeigte Stil, macht eine Antwort darauf überflüssig.“

    Suchen Sie nun nach Argumenten, um sich um Ihre eigenen längst überfälligen sachlichen Beiträge drücken zu können?

  15. Lieber Herr Georgiew,

    zu #124, meinen Sie wohl mich mit NF.

    „Der Bild-Leser kann es aber definitiv nicht.“ Falsch, ich habe auch schonmal Bild gelesen und kann es. Ich denke, sie wollen letztlich sagen, dass die Zahlen zwar matehmatisch äquivalent sind, aber unterschiedliche Folgerungen suggerieren.
    Nun, dazu kann ich nur sagen: wer aus den Zahlen alleine irgendwelche Folgerungen zieht, ist selber schuld, egal ob Akademiker oder nicht. Und wenn er meint, diese hingen von der Darstellungsform ab, ist ihm nicht mehr zu helfen.
    Ich denke, dass Sie an diesem auch bei Ihnen ausgeprägten Defizit („indirekt geben Sie zu, dass er vernachlässigbar gering ist!“) arbeiten sollten. Durch Anderen Aussagen in den Mund legen kommen Sie nicht weiter.

    Zu Ihren Fragen:
    „Ist die einzige O2-Quelle der Natur die Photosynthese auf Kosten des Abbaus von CO2?“ Nein, aber die wichtigste.

    „Ist das Leben vor 3,5 Mrd. Jahren in sauerstofffreien Umgebung entstanden“ Ja.

    „war daher in der Frühgeschichte der Erde der CO2 Gehalt exorbitant höher? Ja, bis 100mal höher als heute.

    „Hat sich das Leben auf der Erde in 3,5 Mrd. Jahren prächtig entwickelt?“ die biologische Evolution kennt den Begriff „prächtig“ nicht.

    „Sind die Fragen mit „ja“ beantwortet, folgt die logische Frage: Wieso soll, seitdem es das IPCC gibt, ein vergleichsweise geringer CO2 Anstieg (in absoluten Werten) auf einmal lebensbedrohlich sein?“

    Auch wenn ich Ihre Frage nicht logisch konsequent finde, gibt es eine einfache Antwort: Das Klima war schon immer ein Evolutionsfaktor, der zum Untergang und Aufstieg von Arten führte, also für einige Arten schon immer „lebensbedrohlich“ war. Die Aussage ist in so fern nichts grundlegend Neues.

  16. Lieber Herr Leistenschneider in #123,

    was hat Ihr elementarer O-18 Exkurs (der mir nichts Neues bietet) in #122 mit meinen Aussagen in NB #100 zum C-Zyklus zu tun?
    Aber da wir schon dabei sind: Sie schlußfolgern in Ihrem #122: „womit die Behauptung von Herrn Prof. Marotzke, durch die Arbeiten zu den DO-Ereignissen widerlegt ist.“

    Was offensichtlich ein Trugschluß ist. Denn wie man wissen sollte (siehe die D-O-Literatur oder IPCC 2007), zeigen die von Ihnen in Ihrem „Widerlegungsversuch“ gezeigten D-O-Temperaturrekonstruktionen von Suenje/PIK den lokalen bis regionalen Temperaturverlauf an Grönlands Küste, aber Marotzke spricht explizit von Trends im global Mittel – womit Ihre Argumentation nicht valide ist, und es für Sie heißt: Zurück zum Start! – q.e.d. 😉

    Nun zu Ihrer Äußerung:

    „“„Sind die wichtig? Die Summe aus den geologischen Emissionsprozessen ist pro Jahr unter 0,2 GtC. Damit haben Sie für die Einzelprozesse eine obere Grenze.“
    Aus einer willkürlichen Erhebung von 24 zu 1400 Vulkanen (Punkt 3) lässt sich nun einmal statistisch keine Aussage treffen, wie hoch der C-Ausstoß der Vulkane beträgt. Nicht einmal abschätzen. “

    Wer sagt, dass ich dies so getan hätte?

    „Hinzu kommen die gewaltigen Ausgasung auf Grund der Plattentektonik und die Angaben diverser Geologen und Institute wie Prof. Kopf, AWI und der Aussage des geologischen Instituts für angewandte Geologie der Uni Erlangen (siehe Punkt 3), um nur einige zu benennen und Sie werden mir bitte nicht böse sein, dass ich deren Fachkompetenz höher einschätze, als Ihre in die Welt gesetzten 0,2 GtC. “

    Ich lasse mich von Fachkompetenz gerne belehren. Dann zeigen Sie mir bitte, wo die Fachkompetenz explizit angibt, dass die geologisch bedingten Emissionen von CO2 in die Atmosphäre die letzten Jahrzehnte im Schnitt 0,2 GtC/Jahr überschritt. Darum geht es ja.

  17. Sehr geehrter Herr NB, #118

    Statement von Herrn Georgiew: „Die Nutzung der Einheit ppm ist dazu erfunden worden, um die mickrigen Mengen „Erhöhung des CO2 Anteils um 0,002%“ bedrohlicher erscheinen zu lassen.“

    Ihre Antwort: „Glauben Sie das wirklich?
    Sie sind doch ein technisch versierter Mensch uns sollten den Sinn von absoluten oder relativen Änderungen kennen. Einem technisch vorgebildeter Mensch sollte doch klar sein,…“

    Sie haben sich in #79 über eine nicht sachgerechte Beantwortung (Behandlung) beklagt. Nun, der von Ihnen in #118 gezeigte Stil, macht eine Antwort darauf überflüssig. Der Text spricht für sich!

    Viele Grüße RL

  18. Lieber NF,

    jeder, der hier schreibt, kann zwischen ppm und % umrechnen und zwischen absolute und relative Werte unterscheiden. Der Bild-Leser kann es aber definitiv nicht. Würde in der Bild wahrheitsgemäß und jedermann verständlich stehen, dass sich der CO2 Gehalt um 0,01% von 0,03% auf 0,04% erhöhen könnte, würde sich in hier kaum jemand um den Klimaschutz scheren. Mathematisch das gleiche, Erhöhung von 300 auf 400 ppm klingt bedrohlich, die Einheit verstehen 9 von 10 nicht, es wird 33% Erhöhung suggeriert, obwohl es absolut 0,01% sind. Propaganda funktioniert nun mal so. Im Krieg wurde der versenkte Schiffsraum in BRT monatlich gemeldet, zum Schluss in BRT pro U-Boot im Einsatz, durch die sinkende Zahl der U-Boote stieg sogar die Quote. Ein klassisches Beispiel für die Benutzung von relativen Werten um die absolute Wahrheit schönzureden.

    Der Mensch neigt dazu, die Fakten nach seinem Glauben auszurichten, daher blenden Sie den wichtigen absoluten Wert aus (indirekt geben Sie zu, dass er vernachlässigbar gering ist!) und richten Ihre Gedankenkonstruktion am relativen Wert aus.

    Sie könnten mir folgende Fragen beantworten:
    Ist die einzige O2-Quelle der Natur die Photosynthese auf Kosten des Abbaus von CO2? Ist das Leben vor 3,5 Mrd. Jahren in sauerstofffreien Umgebung entstanden, war daher in der Frühgeschichte der Erde der CO2 Gehalt exorbitant höher? Hat sich das Leben auf der Erde in 3,5 Mrd. Jahren prächtig entwickelt?

    Sind die Fragen mit „ja“ beantwortet, folgt die logische Frage: Wieso soll, seitdem es das IPCC gibt, ein vergleichsweise geringer CO2 Anstieg (in absoluten Werten) auf einmal lebensbedrohlich sein? Die Antwort führt automatisch auf das IPCC-Konto.

    Lassen Sie bitte die Physik dafür beiseite!

  19. Sehr geehrter Herr NB, #100

    Ihre Wissenslücke zu 18O und den Temperaturen habe ich in meiner heutigen Antwort an Herrn Müller schließen können. Nun zu Ihrer Äußerung:

    „Sind die wichtig? Die Summe aus den geologischen Emissionsprozessen ist pro Jahr unter 0,2 GtC. Damit haben Sie für die Einzelprozesse eine obere Grenze.“

    Aus einer willkürlichen Erhebung von 24 zu 1400 Vulkanen (Punkt 3) lässt sich nun einmal statistisch keine Aussage treffen, wie hoch der C-Ausstoß der Vulkane beträgt. Nicht einmal abschätzen. Hinzu kommen die gewaltigen Ausgasung auf Grund der Plattentektonik und die Angaben diverser Geologen und Institute wie Prof. Kopf, AWI und der Aussage des geologischen Instituts für angewandte Geologie der Uni Erlangen (siehe Punkt 3), um nur einige zu benennen und Sie werden mir bitte nicht böse sein, dass ich deren Fachkompetenz höher einschätze, als Ihre in die Welt gesetzten 0,2 GtC. Viele Grüße RL

  20. Sehr geehrter Herr Müller, #93

    „Will der Autor ernsthaft behaupten, die klimatischen Schwankungen in der Größenordnung von Jahrzehnten seien die Zacken dieser Kurve??“

    Die Abbildung 1 deckt einen Zeitraum von 50.000 Jahren ab und stammt von GISP 2. Die Spitzen signalisieren die starken Temperaturanstiege anhand des Isotopenverhältnisses 18O, bzw. dessen Anteil. Es ist in der Klimawissenschaft stand der Technik, aus dem Isotopenverhältnis, oder dem relativen Anteil des schweren Sauerstoffs (18O) die äquivalenten Temperaturen zu diesem Verhältnis abzuleiten. Grundgedanke ist dabei, dass bei Erwärmung zuerst die leichteren Sauerstoffisotope, allen voran 16O verdampfen und daher der relative Anteil des schweren Sauerstoffisotops 18O im Wasser ansteigt. Deshalb ist es in der Klimawissenschaft auch gängige Praxis, den Proxy-Verlauf des 18O direkt in Relation zum Temperaturverlauf zu setzen.

    Da sowohl die Glättung, als auch das Isotopenverhältnis zu Fragen Anlass gaben, inwieweit beides mit der Temperatur in Jahrzehnten in Zusammenhang steht, hatte ich in #21 auf die Arbeiten des PIK Potsdam hingewiesen (www.pik-potsdam.de/~boris/statcrew/suenje_12.01.06.pdf). Dort ist auf der Seite 7 (“Beispiele vom T-verlauf eines DO-Events“) zu sehen, wie der 18O-Gehalt mit der Temperatur in Einklang steht (links 18O und rechts die Temperatur in °C aufgetragen) und wie in Zeitskalen von Jahrzehnten die Temperaturen um bis zu 6°C schwankten, womit die Behauptung von Herrn Prof. Marotzke, durch die Arbeiten zu den DO-Ereignissen widerlegt ist. Die zweite Widerlegung ist in Abbildung 2 zu sehen.

    Zu der Quelle, lt. GEOPHYSICAL RESEARCH LETTERS, VOL. 30, NO. 10, 1510 (http://www.pik-potsdam.de/~stefan/Publications/Journals/rahmstorf_grl_2003.pdf) wird als Autor, Stefan Rahmstorf genannt. Viele Grüße RL

  21. Lieber Herr Hess,

    danke für den Link zum Segalstad-paper.
    Ich denke, das ist das paper, auf welches sich Herr Limburg bezieht.

    Das scheint ja eine „heilige Schrift“ der Klimaskeptiker zu sein. Die Urschrift, auf die sich Beck und andere hier beziehen. Bislang ging ich davon aus, dass wenigstens die Strategie zur Täuschung auf EIKEs eigenem Mist gebaut wurde. Aber selbst die wurde Ihnen noch vorgekaut…

  22. Lieber Herr Hartmann,

    zu vulkanischen Beiträgen der CO2-Emissionen hier eine Untersuchung des britischen geologischen Dienstes:
    http://tinyurl.com/yhrsff2
    Emissionsraten liegen im Bereich unter 0,1 GtC/Jahr (= 100 MtC/Jahr) und stimmen damit mit den Angaben vom IPCC überein.

  23. Zitat:

    /“Fakt ist jedenfalls, dass bei EIKE Kritik nicht erwünscht ist…“/

    „mit Verlaub, aber das ist nun wirklich Quatsch. Bisher wurde hier noch jeder kritische Beitrag veröffentlicht.
    Kritik ist EIKE hochwillkommen, denn sie belebt die Diskussion und diese steigert die Attraktiviät der Seite. Eine Zensur findet hier nicht statt. Nicht freigeschaltet werden lediglich allzu beleidigende und verfassungswidrige Kommentare.

    Grüße
    Admin“

    Hier ist dem Admin unbedingt Recht zu geben.
    Auch der freien Diskussion abträgliche enervierende Rabulistik-Nebelkerzen werden veröffentlicht.
    Wenn ich (u.A. Experte für kombinierte passive serielle und parallele Schaltungen bei 3 SC-Strats und Clones) Sätze beginne mit „Fakt ist …“, dann lese ich mir’s meist vorher noch mal durch, wg. GLAUBWÜRDIGKEIT.

    @ Ebenherz: Fakt ist, daß der CO2-Anteil ca. 800-1200 Jahre NACH einer (menschenfreundlichen) Erderwärmung stieg und steigt. Das ist wahr, bis das Gegenteil bewiesen ist. Und das kann dauern. :-))

    btw. ist das eine der offensichtlichsten Lügen bzw. Fälschungen in Gores Emissionshandel-Propagandafilm, von dem peinlichen Eisbär-Kindermärchen abgesehen.
    Die Eisbärpopulation hat sich seit den ’60ern verfünffacht.

  24. Lieber Herr Georgiew,

    „Die Nutzung der Einheit ppm ist dazu erfunden worden, um die mickrigen Mengen „Erhöhung des CO2 Anteils um 0,002%“ bedrohlicher erscheinen zu lassen.“

    Glauben Sie das wirklich?
    Sie sind doch ein technisch versierter Mensch uns sollten den Sinn von absoluten oder relativen Änderungen kennen. Einem technisch vorgebildeter Mensch sollte doch klar sein, dass es für die Sache und für die Zahlen keinen Unterschied macht, ob man relative oder absolute Änderungen betrachtet. Wollen Sie sagen, dass die Herstellung von 1 µg Dioxin ungefährlich ist, da sich die Menge des darin gebundenen Kohlenstoffs und anderer Elemente im Vergleich zur Gesamtmenge der Vorkommen klein ist?
    Meinen Sie nicht, dass Sie mit solchen Aussagen hinter Ihren Möglichkeiten bleiben?

  25. #113, Hallo Aktivposten

    ich bin da völlig bei Ihnen.

    Ich wollte nur auf die augenfällige Diskrepanz in Ihrem ersten Beitrag hinweisen. Sie äußern sich ja jetzt auch wesentlich differenzierter. Tatsächlich habe auch ich grob vereinfacht, denn die Atmosphäre zeigt beileibe keine konstante Zusammensetzung in Abhängigkeit von der Höhe, selbst die CO2-Isotope varieren ja.

  26. Herr Limburg, Herr Heß schreib doch, dass andere Werte gemeint sind und daher icht im Widerspruch stehen. haben Sie die Diskussion nicht gelesen?

  27. Lieber Herr Limburg,
    Auszug aus IPCC AR4 wg1 Annex 1:
    „Carbon dioxide
    (CO2) is an extreme example. Its turnover time is only about four
    years because of the rapid exchange between the atmosphere and
    the ocean and terrestrial biota. However, a large part of that CO2 is
    returned to the atmosphere within a few years. Thus, the adjustment
    time of CO2 in the atmosphere is actually determined by the rate
    of removal of carbon from the surface layer of the oceans into its
    deeper layers. Although an approximate value of 100 years may be
    given for the adjustment time of CO2 in the atmosphere, the actual
    adjustment is faster initially and slower later on.“

    Kein Widerspruch. Den Unterschied zwischen „adjustment time“ und „turnover time“ habe ich oben in #105 erklärt.
    Mit freundlichen Grüßen
    Günter Heß

  28. Lleber NF,
    es ist vielleicht verjährt, aber ich habe soeben Ihr Beitrag 44 gefunden:
    „“Wird reiner Kohlenstoff verbrannt (Stein- und Braunkohle), müsste die O2-Konzentration im selben Maß fallen, wie die CO2-Konzentration zunimmt.“

    Im Prinzip ja, aber dazu ein Zahlenbeispiel:
    20%-100ppm sind dann mit 19,9999% praktisch dasselbe. 100ppm Zunahme ist bei einem Spurengas wie CO2 eine deutliche relative Zunahme, bei O2 aber vernachlässigbar.““

    Der Schwindel oder Selbstbetrug ist im Ansatz der unterschiedlichen Prozentbasis versteckt (Erhöhung von CO2 wird mit CO2 Anteil verglichen, O2 mit Anteil an Atmosphäre). Bei sachlich richtiger Betrachtung müsste man zugeben „O2 ist praktisch konstant auf hohem Niveau, CO2 praktisch konstant auf niedrigem Niveau, selbst wenn sich der CO2 Gehalt verdoppeln sollte“. Die Menge, die an O2 verloren geht, ist natürlich die Gleiche wie die Zunahme an CO2.
    Die Nutzung der Einheit ppm ist dazu erfunden worden, um die mickrigen Mengen „Erhöhung des CO2 Anteils um 0,002%“ bedrohlicher erscheinen zu lassen.
    Gruß
    PG

  29. Sehr geehrter Herr Wanninger,
    vielen Dank für die Aufklärung, wobei das für mich unter der Brücksichtigung der Istopenanalyse und des daraus gefolgerten „menschlichen“ Anteils immer noch keinen Sinn ergibt, da 13C (ein stabiles Isotop im Gegensatz zu 14C) zu ungefähr 1,1% auch natürlich vorkommt. Woher will man denn da schon wieder wissen, welcher Anteil natürlich und welcher „menschlich“ ist ? Und wieder ziele ich auf den kleinen Anteil ab: wenn der Anteil C13 an der Gesamtmenge Kohlenstoff (nicht CO2 !) natürlicherweise 1,1% beträgt und angenommenerweise durch menschliches Zutun auf 1,3% oder 1,5% steigt, so what ?
    Wie kann das (unter Berücksichtigung des Jahrhundertwinters) zu einer „Unumkehrbarkeit des Klimawandels“ führen ?
    Das ist vollkommen lächerlich…

  30. Lieber Herr NB,
    ich habe ein Paper http://tiny.cc/pkbJx
    von Segalstad gefunden:
    „Carbon cycle modelling and the residence time of natural and anthropogenic atmospheric CO2: on the construction of the „Greenhouse Effect Global Warming“ dogma.“

    Er meint also die „residence time“ bzw. „turnover time“ und da ist dann wohl kein Widerspruch zum IPCC.
    Eine Tabelle zeigt:
    “Table 2. Atmospheric residence time (i.e. lifetime, turnover time) of CO2, mainly based on the compilation by Sundquist (1985; for references in brackets).”
    Mit freundlichen Grüßen
    Günter Heß

  31. Lieber Herr Hess,

    ich denke auch, dass Segalstadt die „Turnover Time“ des CO2s in der Atmosphäre meinte. Die ergibt sich ja leicht aus den bekannten Bilanzzahlen der Kohlenstoffbilanz. Wenn durch Vegetation und Meer pro Jahr ca. (120+90) GtC = 210 GtC umgewälzt werden (den keinen menschengemachten „Bonus“-Eintrag mal vernachlässigt), und z.Zt 817 GtC (entsprechend = 375 ppmv CO2) in der Luft sind, so dauert es im Schnitt alle 817/210 = 3,9 Jahre, dass ein CO2-Molekül einmal die Atmosphäre verlassen hat und durch die Decksicht oder die Biosphäre lief.

    Kennen Sie den von EIKE zitierten aber unverständlich erklärten Segalstadt-Artikel?

    Lieber Herr Hartmann,

    „Sie wissen also genau auch besonders in welcher quantitativen Hinsicht der CO2-Anteil aus dem Bereich der Plattentektonik von Wichtigkeit im Kohlenstoffhaushalt der Erde ist oder nicht! “

    Falsch, ich weiß es nicht genau. Wahrscheinlich weiß man es genauer als ich. Ich habe Ihnen ja nur eine obere quantitative Grenzen für die jährlichen geologischen Emissionsraten geben können. Eine noch gröbere obere Grenze können Sie ja selber aus den Messkurven des CO2s ablesen.

    „Vielleicht bin auch nur äußerst mangelhaft informiert…!?“

    Wenn’s nur das wäre, so freut es mich, dass ich Ihnen weiterhelfen konnte. Ich denke, ein Mangel an Information oder der Zugang dazu ist nicht Ihr eigentliches Problem.

    PS: über manche Leute amüsiere ich mich köstlich.

  32. Lieber Herr NF,
    mit dem Swimming Pool wollte ich nur die Definition klarstellen.
    Die „turnover time“ wäre die einfachere Frage gewesen, da sie einfach der Quotient aus Masse und Fluss ist.
    Die „turnover time“ beträgt für den Austausch Oberflächenozean (ca. 70 m) und Atmosphäre laut Fortunat Joos 8.5 Jahre. Für den Austausch Oberflächenozean plus Biosphäre und Atmosphäre 4 Jahre.
    Die „turnover time“ für den Austausch mit dem gesamten Ozean beträgt 530 Jahre laut Fortunat Joos.
    Die „response rime“ oder „adjustment tme“ ist am schwersten zu bestimmen, da man dafür eben das Experiment machen muss ,die Emissionen abzuschalten. Ansonsten kann man nur die einzelnen Prozesse untersuchen und simulieren.
    Für eine kleine Störung (Zunahme) im System Atmosphäre – Oberflächenozean gibt Fortunat Joos die „response time“ mit 1 Jahr an. Allerdings hat sich nach dieser Zeit ein stationärer Zustand mit einer höheren Konzentration in der Atmosphäre eingestellt. Der gesamte Ozean erreicht laut Fortunat Joos erst nach ca. 1000 Jahren seinen neuen stationären Zustand.
    Aus diesen Zeiten schätzt wohl das IPCC ab, dass die „response time“ des gesamten Systems eben bis zu tausend Jahren betragen kann.
    Herr Dietze hat auch ein Modell entwickelt und kommt auf 55 Jahre.
    Ich selbst habe bisher immer nur gelesen und keine eigenen Abschätzungen gemacht. Aber vielleicht lohnt es sich ja eine Diskussion zu starten.
    Hier meine Quellen:
    Diese Homepage ist interessant und repräsentiert wohl das IPCC Wissen:
    http://tiny.cc/W2PH4
    weiter geht’s mit Herrn Dietze
    http://tiny.cc/9hRcc
    und Hern Kehl
    http://tiny.cc/RwwWM
    habe ich alle mal über Herrn Gans gefunden, also müssen sie ihm danken.
    Mit freundlichen Grüßen
    Günter Heß

  33. #107 Balthasar Schmidt

    Sehr guter Beitrag. Das habe ich mir mit allem Ernst auch schon überlegt. Ich selbst finde, in letzter Zeit eindeutig zuviel Zeit bei diesem Thema verbracht zu haben. Aber manche Leute scheinen regelrecht mit professionellem Zeitaufwand in den diversen Foren zu verbringen, um (aus Ihrer Sicht und so äußern sie sich auch gelegentlich) die ‚Skeptiker-Dumpfbacken bzw. unentschiedenen Mitleser zu überzeugen‘.
    Nun ich will es nicht falsch verstanden wissen. Ich bin ergebnisoffen, und nehme gerne Fakten pro/contra wahr. Meine Motivation habe ich unter #32 schon geschildert. Natürlich habe ich eine Tendenz, und alles was ich höre und beobachte, verstärkte meine Zweifel bislang nur.

    Allerdings sehe ich die politische Wertung nicht ganz so negativ. Ich denke, die Politiker haben hier einfach einen schönen neuen Wachstumsmarkt entdeckt. Hier entstehen neue Industrien, die neue Produkte an den Mann bringen und das weltweite Arbeitslosigkeitsproblem kurzfristig, primär zum Nutzen der reichen Länder, etwas entspannen, z.B. PV-Module, meterdick in Styropor verpackte Häuser, neue E-Autos usw. usw. Aus marktwirtschaftlicher Sicht ist das natürlich Schwachsinn. Aber wer traut sich schon an das eigentliche Problem, Arbeitszeitverkürzung OHNE Lohnausgleich. Weniger Geld, weniger Wohlstand, weniger Probleme. Und was macht man mit den Gesellschaften, die dann noch mehr Freizeit hätten. Alles Sprengstoff. So verzögern sie lieber den nächsten Wirtschaftcrash und pumpen mit neuen Staatschulden weiter ungedecktes Geld in den Kreislauf gegen den imaginären Feind AGW.

  34. Sehr geehrter Herr Heß,

    vielen Dank.

    Verlassen wir das Beispiel des Schwimmbads und kehren zurück zum Kohlenstoffzyklus, so wäre für mich die „response time“ die interessantere Größe. Kennen Sie eine ungefähre Zahl dazu?

    MfG

  35. Hallo Aktivposten,

    das haben Sie gleich richtig erkannt. Eigentlich braucht man nicht mehr als einen gesunden Verstand um zu erkennen, wie in diesen Foren von einigen Teilnehmern (u.a. NB, NF) stetig versucht wird, von den eigentlichen Kernproblemen abzulenken, auf unwichtige Details umzuschwenken, um welche dann verbissen gerungen wird, als wenn es darum gehe, das Rad neu zu erfinden.
    Dies ist nichts anderes als ein Frage und Antwortspiel mit Ablenkungsmanövern, um die weniger fachlich kompetenten Forumsteilnehmer zu verunsichern.
    Die besagten Personen wenden sehr viel Zeit dafür auf, so daß man sich die Frage stellen muß, wer sitzt denn da im Hintergrund der Interesse daran hat, diesen Zeit- und Arbeitsaufwand zu finanzieren, um die Religion der Klimahysteriker so penetrant verteidigen zu lassen?
    Es ist gut, daß es immer mehr Menschen gibt, welche ihren Verstand dazu benützen die Klimahype in ihrer Gesamtheit (Qui Bono?) zu sehen und zu verstehen und auch in der Lage sind, sich darüber selbst eine Meinung zu bilden.
    Wer lange genug die entscheidenden Fakten gesammelt hat, kann auf kein anderes Ergebnis mehr kommen, als daß hier ein globaler Betrug stattfindet, bei welchem sich Wissenschaftler zu Helfershelfern von Politikern machen lassen und Politiker die Meinung von Wissenschaftlern dazu benützen, sich ein Alibi für ihre politischen Entscheidungen geben zu lassen. Dies kann dann nur noch als Verschwörung zwischen Wissenschaft und Politik bezeichnet werden. Der eine ist des anderen Steigbügelhalter, somit jeder seinen Teil des monetären Kuchens abbekommt.
    Die Profiteure und das Ziel sind allen bekannt und dies gilt es zu verhindern.
    Die Antworten zu Ihren Fragen kann Ihnen mit Sicherheit einer der seriösen Mitstreiter oder Herr Limburg selbst besser als ich hier geben.

  36. Lieber NB,
    danke für Ihre Antwort!
    Den Inhalt letzterer hatte ich mir , was meine Voraussage im Zusammenhang mit meiner Kenntnis über die Art und Weise Ihres Auftretens hier bei EIKE betrifft, auch genau so vorgestellt. Sie wissen also genau auch besonders in welcher quantitativen Hinsicht der CO2-Anteil aus dem Bereich der Plattentektonik von Wichtigkeit im Kohlenstoffhaushalt der Erde ist oder nicht! Na, dann publizieren das `mal schnell in der einschlägigen Fachpresse. Nach meiner Kenntnis wären Sie damit wohl der Erste. Sehr gut! Vielleicht bin auch nur äußerst mangelhaft informiert…!?

    P.S.: Der Teil Ihrer Antwort „Es ist mir selber unerklärlich, aber hier möchten inzwischen so viele nicht mehr von EIKE, sondern von mir aufgeklärt werden; Sie müssen sich für eine Antwort nun leider anstellen. Klar formulierte Fragen werden bevorzugt.“ hatte mit der von mir hier gemachten Bemerkung nicht das Geringste zu tun. Es zeugt aber auch davon, wie Sie Ihre eigene Person übermäßig wichtig nehmen und sich selbst dabei jämmerlich überschätzen. Rein menschlich gesehen, ist das nur bedauernswert.

    #99 Wissender
    Um im Sinne von NB zu reden: Ich kann hier wirklich nicht die Rolle eines Aufklärers übernehmen. Dafür habe ich zu wenig Zeit und bin auch, ehrlich gesagt, zu faul dazu. Vor allem aus Zeitgründen nehme ich auch nur sporadisch an den Diskussionen hier teil. Sich da selbst Informationen zu holen, ist bestimmt auch keine Zumutung, weshalb mir meine Reaktion auch nicht peinlich ist. Wenn es für Sie trotzdem tatsächlich eine Zumutung sein sollte, stehe ich natürlich zur Verfügung.

  37. Lieber Herr NF,
    ich glaube hier werden einfach immer zwei unterschiedliche Zeiten durcheinander geworfen. Die sogenannte „Residence Time“ oder „Turnover Time“. Das ist die Zeit die ein Molekül oder eine bestimmte differentielle Masse im Mittel in der Atmosphäre verbleibt, nachdem sie in die Atmosphäre emittiert wurde, bis sie wieder entfernt ist. Angenommen man hat ein Kreislaufsystem mit Wasserreservoir , Umwälzpumpe, Leitungssystem und Swimmingpool. Das Reservoir sei größer als der Swimmingpool, aber nur über eine dünne Leitung mit dem Kreislauf verbunden, um den Swimmingpool immer auf dem gleichen Wasserstand zu halten
    In diesem Modell ist die „Turnover Time“ die Zeit, die es dauert bis das Volumen im Swimmingpool komplett einmal durch die Umwälzpumpe ausgetauscht ist. Im Grunde beschreibt diese Zeit die Saugleistung der Pumpe im Kreislauf und die Querschnitte des Leitungssystems im Kreislauf, nicht die dünne Leitung zum Reservoir.
    Davon unterscheiden muss man die „Adjustment Time“ oder „Response Time“ auf eine Störung im System. Wir schütten eine zusätzliche Wassermenge in den Swimmingpool. Dann ist die „Response Time“ diejenige Zeit, die es dauert bis die Störung im Wasserstand über die dünne Leitung abgeklungen ist und das zusätzliche Wasser im Reservoir landet. Sie beschreibt den Querschnitt der dünnen Leitung zum Reservoir und den Durchfluß zum Reservoir.
    „Airborne Fraction“ konstant heißt in diesem Bild, dass wir die zusätzliche Wassermenge nicht komplett ins Reservoir fließen lassen und den Wasserstand konstant halten, sondern dass der Wasserstand im Pool eine prozentuale Funktion von der gesamten Wassermenge im gesamten Kreislaufsystem mit Reservoir ist.
    Ich würde sagen „Airborne Fraction“ konstant soll bedeuten, dass wir für die Erde noch in einem Bereich sind, wo die Aufnahmefähigkeit der Senken und der Prozesse die CO2 wieder aufnehmen zurzeit nicht schwächer werden.
    In dem Analogbild: Das heißt der Querschnitt des Schlauches zum Reservoir ist groß genug, um den neuen stationären Wasserstand schnell einzustellen den wir aufgrund der gesamten Wassermenge vorgeben.
    Mit freundlichen Grüßen
    Günter Heß

  38. #103 Aktivposten

    leider ist die Welt nicht so einfach, wie Sie sie beshreiben. Ich teile zwar Ihre Weltanschauung (es gibt keinen AGW), aber Ihre Überlegungen sind nur oberflächlich. Die tägliche Erfahrung reicht zur Analyse nicht aus.

    Zum Trockeneisnebel: Die Prozesse, die hier ablaufen sind sehr kompliziert durch Kälte, Anbindung von Wasser usw. Letztlich ist das Ergebnis aber, dass sich nach Erwärmung des CO2 nach vielleicht 10 Minuten dieses absolut gleichmäßig mit der Raumluft vermischt hat und Sie am Boden und unter der Decke genau den gleichen Anteil CO2 messen werden. Das gilt auch für den unteren Teil der Atmosphäre. Sie steht mit den Ozeanen und der Biomasse in einem GLeichgewicht hinsichtlich des CO2-Gehalts. Und zwar enthalten die oberen Meeresschichten, Atmosphäre und Biomasse ungefähr gleiche Mengen CO2. Fügen Sie der Atmosphäre durch Verbrennung CO2 zu, löst sich ein Teil im Meer, ein Teil in der nun besser wachsenden Biomasse, der Rest bleibt zunächst in der Atmosphäre, grob 1/3. Das hat nichts mit dem spez. Gewicht des Gases zu tun, das sich unten sammeln würde.
    Das Oberflächenwasser gibt dann sein CO2 aber an das Tiefenwasser ab und kann wieder neues atmosphärisches CO2 aufnehmen. Diese Prozesse sind höchst kompliziert und quantitativ wenig verstanden, insbesondere auch die zeitlichen Bilanzen. Und deshalb sind auch die Bilanzen natürlicher und menschlicher Quellen höchst umstritten. Und u.a. deshalb streitet man sich hier udn anderswo so trefflich.

  39. Sehr geehrte Diskutanten,
    ich mische mich jetzt mal als Nichtwissenschaftler und Nichtakademiker in die Diskussion ein. Vielleicht können Sie einem „Wissenschafts-Ketzer“, der lediglich sein Gehirn gebraucht, ja Einiges erklären.

    1. Ich weiß (gelernt), daß CO2 schwerer ist als Luft. Das kennt jeder, der in der Schule das Trockeneis-Experiment gemacht hat, alle Anderen haben mit Sicherheit im Fernsehen schon mal Bodennebel als Effekt gesehen. Dieser wird aus Trockeneis (=gefrorenes CO2) gemacht, der bei höheren Temperaturen den Aggregatzustand wechselt und dann aus Schalen deutlich zu Boden fällt.
    Frage: wie kommt also das „anthropogene“ CO2 in die Atmospäre ?
    Meine überwiegend (bis auf meine Frau) Pro-AGW eingestellte Umgebung erklärt das mit den Flugzeug-Abgasen, aber die sind doch immer noch schwerer als Luft und sinken auch zu Boden. Keine der gängigen Theorien hat das bisher auch nur annähernd plausibel erklärt.

    Zitat:“Von den sieben Gigatonnen jährlich, die die Menschheit verursacht, bleibt etwa die Hälfte in der Atmosphäre.“
    Wie entscheidet das CO2 das, was oben bleibt und was nicht ? Ziehen die Moleküle Hölzchen ? Oder würfeln die ? Wieso sinkt eine Hälfte – physiklich logisch – ab und die andere Hälfte nicht ? Ist die leichter geworden ?

    2. Aus den diversen Meßreihen und Behauptungen werde immer wieder Eiskernbohrungen (Punkt 2 und Punkt 8) erwähnt.

    a) Aus dieser Webseite (http://www.lost-squadron.org/cms/index.php) geht ganz klar hervor, daß eine 100 Meter dicke Eisschicht in ungefähr 75 Jahren entstanden ist. Da die Extrapolation vorwärts und rückwärts zum „wissenschaftlichen“ Geschäft gehört, ist die Eisschicht auf Grönland (ca. 2000 Meter im Durchschnitt) in ca. 1500 Jahren entstanden. Abgesehen davon, was diese Erkenntnis für einige andere „Wissenschaften“ bedeutet (Fossilien nur noch tausende statt Millionen Jahre alt usw.), sind ja wohl alle Annahmen über ein Klima vor 15 Millionen Jahren diskreditiert. Selbst die Ergebnisse vor hundert Jahren (anhand der „Jahresringe“ im Eis) sind äußerst fragwürdig, da eine genaue Bestimmung, wann welches Eis entstanden ist, überhaupt nicht möglich ist. Oder haben die Eiskernbohrer tatsächlich mindestens 135 Meter tief gegangen ?

    b) Noch viel gewichtiger ist allerdings die Frage wie sich der CO2-Gehalt der ATMOSPÄRE (!!!) aus dem Eis ablesen läßt ???????????
    Wie hat sich die ATMOSPÄRE im Eis manifestiert ?
    Kommt die Atmospäre regelmäßig auf dem Eis nieder, um einen Fingerabdruck für die Nachwelt zu hinterlassen ?
    Alleine diese Frage, aus gesundem Menschenverstand geboren, müßte jedem „Wissenschaftler“ die Schamröte ins Gesicht treiben.
    Zitat:„Die CO2-Konzentration in der Atmosphäre war jahrtausendelang praktisch konstant und steigt erst an, seit wir dem System riesige Mengen an zusätzlichem Kohlenstoff aus fossilen Lagerstätten zuführen“, erklärt Rahmstorf.
    Woher weiß er das ? Aus den „Eisbohrkernen“ ?

    3. Der CO2-Gehalt der Atmospäre beträgt 0,0385% (Wiki) und ist laut nicht beweisbaren Behauptungen in den letzten 100 Jahren durch anthropogene Ursachen um 10% gestiegen, also ist der CO2-Anteil in der Atmo. um 0,0035% gestiegen.
    Und das soll solche Auswirkungen haben ???? (siehe auch nächster Punkt)

    4. Als 1969 Geborener habe ich den „Jahrhundertwinter“ 1978/1979 ziemlich hautnah mitgekriegt. Selbst in den 80er Jahren auf dem Gymnsium wurde immer noch vor einer neuen Eiszeit gewarnt, witzigerweise mit der Erklärung desselben „Treibhauseffektes“ (Erklärung: der „Treibhauseffekt“ läßt das Polareis abschmelzen, dadurch kommt der Golfstrom zum Erliegen, der für das warme Klima sorgt, und das sorgt am Ende auf der Nordhalbkugel für eine neue Eiszeit), der jetzt für die globale Erwärmung verantwortlich sein soll.
    Innerhalb von nur 15 Jahren ist aus der „Eiszeit-Hysterie“ eine „Erwärmungs-Hysterie“ geworden. Und jetzt soll der Effekt schon in wenigen Jahren „umumkehrbar“ sein ?
    Merkt eigentlich keiner, was hier für ein Mumpitz veranstaltet wird ?
    Mein Punkt 3 wird dadurch noch deutlicher, da der „atmosphärische CO2-Anstieg“ ja zunächst eine deutliche Abkühlung verursacht hat.

    Der Sonneneinfluß dürfte also doch größer sein als der des CO2.

    Fazit: seriöse Wissenschaft ist das alles offensichtlich nicht, wenn man mit gesundem Nachdenken und wenigen echten Fakten das ganze Gebäude zum Einsturz bringen kann. Da können auch noch so viele Studien und „Peer-Reviewte“ Arbeiten nicht drüber hinwegtäuschen. Da es auch kaum noch echte disziplinübergreifende Forschung gibt und selbst innerhalb der Disziplinen nur winzige Teilausschnitte betrachtet werden, gibt es kaum noch ein Gesamtbild. Selbst wenn es heute einen „geschlossenen CO2-Kreislauf“ (Punkt 8) gibt, war das auch schon während der letzten Eiszeit so ? Oder vor 2000 Jahren ? Wer will das wissen ?

    Mit freundlichen Grüßen

    Ein kritischer Selbstdenker

    P.S: Das hier erwähnte Peer-Reviewing ist nur ein Mäntelchen für Pseudowissenschaftlichkeit. Denn das Peer-Review sorgt dafür, daß nur neue Forschungsegebnisse „peer-reviewt“ werden, die in den aktuellen Wissenschaftskonsens paßt. Denn abweichende Ergebnisse, selbst wenn diese noch so stichhaltig und plausibel sind, werden durch die Reviewer verhindert.
    Daher können echte neue Forschungergebnisse niemals peer-reviewed sein, weil das jeweils ein Umdenken in den betroffenen Disziplinen zur Folge hätte, was aber im heutigen Wissenschaftsbetrieb quasi nicht mehr passiert.

  40. Herr Gans (#97),

    ich verfolge den Disput mit NB mit Interesse und Verwirrung. Vielleicht helfen meine Fragen nicht nur mehr, sondern allen Lesern.

    1. Wie ist CO2-„Lebensdauer“ oder auch „Verweildauer“ definiert? Ich kenne den Begriff der Halbwertszeit z.B. für Methan, das durch chemische Prozesse in der Atmosphäre abgebaut wird. Ist bei CO2 (da stabil) gemeint, wie lange es dauert, bis ein Molekül wieder in den natürlichen Kohlenstoff-Zyklus eingebunden wird?

    2. Sie zitieren ein Papier von Knorr et al. mit „In contradiction to some recent studies, he finds that the airborne fraction of carbon dioxide has not increased either during the past 150 years or during the most recent five decades.“
    Was verstehen Sie unter „airborne fraction of carbon-dioxide“? Ich vermute, dass Sie an dieser Stelle etwas falsch interpretieren, denn ich kann beim besten Willen keinen Widerspruch zu einer Aussage von NB finden. Würden Sie dies bitte erläutern?

  41. Lieber Herr Gans,

    „Peter Dietze hat sich einem „open review“ unterstelt“

    das ist kein wissenschaftlicher publizierter Artikel.

    „Und auch das widerspricht Ihnen:
    http://tinyurl.com/ycjgl5x

    „In contradiction to some recent studies, he finds that the airborne fraction of carbon dioxide has not increased either during the past 150 years or during the most recent five decades.““

    Wieso?

  42. Lieber Herr Hartmann,

    „Ich habe Ihnen doch deutlich und verständlich genug in #57 gesagt, dass es große weiße Flecken im Wissen über den Kohlenstoffkreislauf gibt und wo sie zu lokalisieren sind.“

    Sind die wichtig? Die Summe aus den geologischen Emissionsprozessen ist pro Jahr unter 0,2 GtC. Damit haben Sie für die Einzelprozesse eine obere Grenze.

    „Ich glaube aber, es sind die Scheuklappen.

    Es ist mir selber unerklärlich, aber hier möchten inzwischen so viele nicht mehr von EIKE, sondern von mir aufgeklärt werden; Sie müssen sich für eine Antwort nun leider anstellen. Klar formulierte Fragen werden bevorzugt.

    Kommentar:in #96

    „Der bekannte Geologe Prof. Segalstadt und mit ihm die Mehrheit der Geologen,
    bestimmt die Verweildauer nur mit 5-10 Jahre (1).“ Etc.

    Wie ist die Verweildauer bei Segalstadt definiert, wo verweilt es und wo geht es hin?
    Ihre ESEF-Referenz habe ich nicht. Ist die wissenschaftliche Diskusssion über die Diskrepanzen nicht schon seit Jahren abgeschlossen?

  43. Wenn die meisten Geologen dieser Meinung sind, dannn bringen Sie doch bitte die eine oder andere Studie, die zu diesem Schluss kommt. Oder ist das so wie bei Gerlich, dass zwar eigentlich alle Physiker wissen, dass der Treibhauseffekt quatsch ist, aber alle Angst haben dies zuzugeben?

  44. Lieber NB,
    mir reicht es wirklich langsam! Um einmal in Ihrem Stil zu reden: „Ich habe Ihnen doch deutlich und verständlich genug in #57 gesagt, dass es große weiße Flecken im Wissen über den Kohlenstoffkreislauf gibt und wo sie zu lokalisieren sind. Entweder sind Sie intellektuell nicht in der Lage, das nachvollziehen zu können oder ist Ihnen das einfach nur zu stressig? Für den letzteren Fall müssen sie sich eben einfach nur mal ein wenig anstrengen!“
    Spaß bei Seite! Mit #57 hatte ich eigentlich bzw. indirekt um eine Gegenrede gebeten. Gut, das traf nicht ein. Vielleicht habe ich es nicht deutlich genug, nicht provokativ genug ausgedrückt. Kann sein! Ich glaube aber, es sind die Scheuklappen.

  45. @NB
    Peter Dietze hat sich einem „open review“ unterstelt und über die CO2 „Lebensdauer“ geschrieben:
    http://tinyurl.com/y9vp4le

    Und auch das widerspricht Ihnen:
    http://tinyurl.com/ycjgl5x
    Zitat:
    —–
    „In contradiction to some recent studies, he finds that the airborne fraction of carbon dioxide has not increased either during the past 150 years or during the most recent five decades.

    The research is published in Geophysical Research Letters.“
    —–

  46. Lieber Herr Ebenherz,

    Zu Antwort 1: „Wo ist also das Problem?“
    Das Problem ist, dass der Einschwingvorgang – wie beschrieben – lange dauert. Wenn Sie heute 1000 ppm CO2 emittieren (und dann stoppen), so hat sich erst nach vielleicht 500 Jahren das neue Gleichgewicht zwischen CO2 und Temperatur ausgebildet. Das ist dann bei vielleicht (müsste ich mal genauer nachrechnen) 4°C höherer Temperatur und 600 ppm, der Rest wanderte in der Zwischenzeit ins Meer (zur Erreichung des neuen chemischen Gleichgewichts).

    zu Antwort 2: Falsch, ich schrieb „man erwartet“. Wenn keine anderen Klimafaktoren wirken würden außer CO2, erwartet man aufgrund der Rechnungen zur Klimaphysik des CO2s diese Steigerungsrate. Mit Einrechnung der anderen Klimafaktoren erwartet man etwas anderes, nämlich, dass sich das Klima entsprechend der damit induzierten Dynamik schwankend ändert. Diese zusätzlichen Klimafaktoren sind mehr oder weniger periodisch. Kurzfristig führen Sie zu starken Schwankungen, die den Trend verdecken (Signal-zu-Rausch-Verhältnis). Langfristig mitteln sie sich raus, der CO2-Trend bleibt übrig. D.h. langfristig steigen die Temperaturen. Die Erwartungen kann man inzwischen mit den Messungen vergleichen und
    langfristig gibt es Übereinstimmung. Nur kurzfristig hat man Probleme, die natürlichen Klimafaktoren richtig zu berechnen, darum geht es z.B. auch in der Wette zwischen Latif und Rahmsdorf und in der Aussage Ihres IPCC Manns. In Ihrer Erwiderung falsifizieren Sie mit den falschen Prämissen.

  47. Lieber Herr NF,

    in #81
    „ich fasse es einfach mal als Kompliment auf, wenn Herr Leistenschneider uns für identisch hält.“

    Ich glaube eher, er kann es nicht fassen, dass es so viele Kritiker seines „Kunstwerks“ gibt.

    Naja, die meisten haben es verständlicherweise ja auch aufgegeben, sich mit Holzköpfen weiter auseinanderzusetzen.

    Ich bin mir ziemlich sicher, wie die Geschichte weitergeht, nämlich so wie immer bei EIKE.

    Sie haben ja schon die Erfahrung gemacht.

    Und auch hier wird Herr Leistenschneider auf die Kritik nicht konstruktiv eingehen, sondern die unzulänglichen Vergleiche, selbstgefälligen Interpretationen und Unterstellungen unverändert drinlassen. Selbst wenn es auch für manche EIKE-Mitglieder evident ist, dass sie falsch sind.

    Denn dies ist Programm von EIKE.

    Nachdem der Artikel nun nicht so kritiklos wie erhofft durchging, setzt EIKE auf Zeit.

    Ich schätze, man hofft, dass die Kritik in dem Wust von mehr oder weniger sinnvollen Kommentaren untergeht. Möglicherweise sind diese „allergische Reaktionen“, auf Kritik mit wüsten Unterstellungen zu kontern und abseitige Themen aufzumischen, auch abgesprochene Taktik.

    Fakt ist jedenfalls, dass bei EIKE Kritik nicht erwünscht ist, damit die EIKE-Argumente keinen öffentlichen Schaden nehmen und weiterhin als Pseudowissenschaft getarnt unter die Leute gestreut oder als „Propaganda- oder Erbauungsliteratur für Gesinnungsgenossen“ genutzt werden kann.

  48. Das ist mir wirklich neu!
    Und das scheint auch Jones, Latif, Trenberth u.a. neu zu sein. Wie sind sonst öffentliche Aussagen bzw. ausgespäte eMails zu verstehen, wonach Modelle und beobachtete Temperaturen nicht miteinader korrellieren?

  49. Werter Herr NB,
    Ich danke Ihnen fuer Ihre Antworten.

    Zu Antwort 1:
    Sie sagen also, dass die Schwingbewegung und das sich gegenseitig beeinflussen von Temperatur und CO2 ein natuerlicher Vorgang ist, den es so immer schon gab. Wo ist also das Problem?

    zu Antwort 2:
    Sie meinen also, man koennte gar nicht berechnen welcher Temperaturanstieg von wie viel Zuwachs CO2 zu erwarten ist. Man schaetzt 0,2 C/10.

    Dann muss ich erwidern: Fuer die letzten 10 Jahre hat es schon mal nicht gestimmt.

    Desweiteren sagt ein Mitarbeiter des IPCC, man haette keine Erklaerung fuer das Ausbleiben des Temperaturanstieges (und dies waere eine Schande). Sie scheinbar eine. Wie kommt das?

    mit freundlichen Gruessen

    E.

  50. #64: Raimund Leistenschneider sagt:
    „Wissenschaft fängt bei Transparenz und Seriosität an.“

    Vielleicht können Sie Ihre Artikel ja in der Hinsicht verbessern, indem Sie die Quellen Ihrer Graphen angeben. Die Abbildung 1 ziehrt die Beschriftung „Rahmstorf 2003“. Ich nehme an, Sie meinen den Artikel „Timing of abrupt climate change: A precise clock“. Dort findet sich allerdings der Graph nicht. Wie Sie als Reaktion auf Klaus_V nachgereicht haben, stammt der Pfeil und das 6K Event aus einem Vortrag von jemand anderem. Dort ist allerdings auf der angegebenen Folie kein Bezug zur Zeit hergestellt. Das Diagram stammt also entweder aus einer anderen Quelle oder ist selbstgemacht.

    Was allerdings nichts daran ändert, dass es Dansgaard-Oeschger events mit ihrem steilen Anstieg der Temperatur gab. Nur ist anscheinend immer noch strittig, ob es ein globales Erwärmungsevent war oder ein lokales. Ich habe mal ein wenig rumgelesen, und da scheint es verschiedene Meinungen zu geben. http://preview.tinyurl.com/yftcf8p erklärt ab 2:45 das DO-Event, postuliert aber, dass eine Erwärmung der Nordhälfte von einer Abkühlung der Südhälfte begleitet wurde, also nur Energie umverteilt wurde. Wikipedia liegt ähnlich, Rahmstorf 2003 macht keine Aussage. Kennen Sie ein Papier, dass das als globale Erwärmung kennzeichnet? Nur dann wäre es eine Widerlegung der Originalthese…

    Ihre verschiedenen Pseudonyme sind denn auch wenig geeignet, Seriosität zu unterstreichen. Ihre Einleitung:
    “ In #4 war ich wohl doch zu optimistisch, was die Urteilskraft mancher Leser angeht“ unterstreicht, dass es sich möglicherweise um ein Überbleibsel pubertärer Selbstüberschätzung handelt, seine Person zu vervielfältigen. Wie aus der Datenverarbeitung bekannt, werden Kopien jedoch niemals besser, sondern, je mehr davon hergestellt werden, immer schlechter.

    Zu Ihren Fragen, Entschuldigung, Äußerungen.

    „Wer kann in dem genannten Diagramm einen Klimawandel von 6° erkennen?“

    Suggestivfragen sind die Argumente von Unwissenden, die in einer Thematik gerne mitreden möchten, aber das Basiswissen überschaubare Dimensionen verzeichnet. Zur Betrachtung empfehle ich Ihnen den Link, den ich Herrn Klaus V in #21 gab.

    „Will der Autor ernsthaft behaupten, die klimatischen Schwankungen in der Größenordnung von Jahrzehnten seien die Zacken dieser Kurve??“

    Es wäre zur Abwechslung einmal schön zu sehen, wenn Sie die Diskussion mit Hinweisen oder Belegen bereichern, anstatt mit Rhetorik. Dafür bedarf es allerdings Verständnis zur Materie.

  51. Lieber Herr Wanninger,

    #85 & #89
    Die Erforschung der verschiedenen Einflüsse auf das Klima und die dadurch bedingte Klimavariabilität ist der eigentliche Inhalt gegenwärtiger Forschung zum Klimawandel. Aber die Forschung ist da weiter als Sie. Denn man weiß inzwischen quantitativ hinreichend genau, wieviel die einzelnen Faktoren und darunter die durch menschliche Aktivitäten ausmachen.

    Wenn das für Sie neu ist, müssen Sie sich nur schlau machen.

  52. #88 Marvin Müller

    Vielleicht ein stärkerer El Nino 2010?

    Übrigens im Blog ‚Treibhauseffekt‘ wartet ‚Besso‘ noch auf eine Antwort von Ihnen…

  53. #88 Marvin Müller,

    das ist eine erstaunliche Spitze, in der Tat. Denn erdgebunden fällt der Winter bislang auf der Nordhablkugel eher strenger aus.
    Das bestätigt aber letztlich genau, was ich oben geschrieben habe. Geht man an die Forschung, ohne zu meinen, das Ergebnis schon vorher zu kennen, sammelt man möglichst viele Daten um etwaige singuläre Einzelereignisse zu minimieren. Letztesres kann man auch statistisch – mit den bekannten möglichen Folgen -> Der Hund…

  54. „Seit dem Rückgang der solaren Aktivität (de Vries/Suess-Zyklus) und dem Ausbleiben des 24. Schwabezyklus, rauschten die Erdtemperaturen in den Keller, wie wir dies gegenwärtig in weiten Teilen auf der Nordhalbkugel anhand des strengen und langen Winters zu spüren bekommen.“

    Ich meine mich an Stellen erinnern zu können, an denen es Beschwerden gab, die Leute könnten Klima nicht von Wetter unterscheiden. Eike griff hier genau zum Wetter, um einen Punkt zu machen. Andere wünschen sich eine Fortdauer der Erwärmung, um weniger Schnee schippen zu müssen. Und nun gibts die Januar Anomalie: http://preview.tinyurl.com/yauk9bo von UAH, den zuverlässigen Sattelitendaten. Ich zitier mal: „The global-average lower tropospheric temperature anomaly soared to +0.72 deg. C in January, 2010. This is the warmest January in the 32-year satellite-based data record.“
    Da hatten wir also global gesehen den wärmsten Januar seit 32 Jahren und wenn ich mich nicht verkuckt habe, sogar der drittwärmste Monat überhaupt (0.753 2/98, 0.770 4/98).

  55. Werter Herr Wanninger,
    das Klima ist ja auch in gewisser Weise über große Zeiträume konstant, selbst in seinen Schwankungen. Schließlich geht es um riesige Energiemengen, die notwendig sind, um plötzliche Änderungen hervorzurufen. Dafür bedarf es anscheinend extremer singulärer Ereignisse (Vulkanausbrüche, Asteroideneinschläge…). Ansonsten reagiert das Klima bedächtig auf Sonnenaktivitätsunterschiede, angetrieben dann auch durch Rückkopplungseffekte und die Änderung von Meeresströmungen. Wäre das Klima tatsächlich in hohen Größenordnungen chaotisch, gäbe es keine empfindlichen Ökosysteme.
    Das Wetter außerhalb der Tropen ist da natürlich anders, deshalb auch so schwer über Wochen vorherzusagen. Aber die einzelnen lokalen Wetterlagen, so sprunghaft (chaotisch) sie sein mögen, hängen ursächlich und regional voneinander ab. Soll heißen, wird es irgendwo plötzlich kalt, muss es in angrenzenden Gebieten warm werden, da es sich um Austauschvorgänge handelt. Wie verrückt das Wetter auch spielt, es sind Prozesse, die auf großen Skalen gut abgebildet werden können. Und selbst wenn es innerhalb dieser zu überraschenden regionalen Abweichungen kommt, gesamtenergetisch sollte das keine Rolle spielen, wenn ich die Physik richtig verstanden habe…:-)

  56. @NF

    Hallo Herr Fuchs,

    damit nicht wieder Verwechselungen passieren:

    NF = Herr Fuchs
    NB = Herr Bäcker

    …ich finde es nämlich ziemlich blöd sich hinter Kürzeln zu verstecken.

    ….allerdings finde ich ihre Beiträge zum Teil auch ziemlich gewöhnungsbedürftig.

    MfG

  57. #82 Lobodelrio

    Ihre Ansicht teile ich nicht. Wäre das Klima konstant, hätten Sie Recht. Will ich aber dessen Variabilität untersuchen, brauche ich möglichst viele Stichproben, zeitlich wie räumlich, denn das Wetter (und damit auch das Klima) zeigt in Ort und Zeit chaotisches verhalten. Nur mit vielen Stichproben kann ich Abweichungen durch extreme Einzelereignisse minimieren.
    Aktuelles Beispiel: Nehmen wir diesen Januar, Norden, Mitte und Osten waren dreimal extrem kalt, nur der Südwesten ist bei allen Kältewellen überdurchschnittlich verschont geblieben. Das ist nicht immer so. Deshalb würde, nehmen wir einfach einmal FFM, als einzelnes Thermometer ein kälteres Monatsamittel liefern, als ein Mittelwert von fünf über Deutschland gleichverteilten Stationen. Das hat nichts mit Überheblichkeit zu tun sondern ist einfach so.

    Warum unterscheiden Sie zwischen Politikern und Nichtpolitikern. Die Politiker kommen aus unserer Mitte. Sie sind im Durchschnitt nicht besser und nicht schlechter als jeder Arbeiter, Angestellte, Selbständige UND eben auch Wissenschaftler. Genauso ehrlich – oder das Gegenteil. Und wenn man sich den Zweck des IPCC anschaut, kann man keine reine und unabhängige Forschung erwarten. Das zu glauben, wäre reichlich naiv.

  58. Herr Eberherz,

    meinen Sie diese Frage:
    „Niemand bezweifelt, dass der CO2 Gehalt der Atmosphaere seit mehreren Jahrzehnten steigt.
    Wenn ich das selbe Verhaeltnis wie oben erwaehnt zu Grund lege, wie kann es dann sein, dass die Temperatur seit 10 Jahren gleich bleibt bzw. faellt?“?

    Differenzierte Antworten von mir dazu finden Sie in climategate-update-17. Durch den CO2-Anstieg erwartet man gegenwärtig global gemittelt +0,2 C/10 Jahre, was mit den Beobachtungen übereinstimmt. Trends über kürzere als 10-20 Jahre sind statistisch unsinnig, da das Signal-zu-Rausch-Verhältnis (CO2-Trend-zu-Klimavariabilität) signifikante Trends auf kürzerer Zeitskala nicht zulassen. Und ohne Signifikanz ist jeder Trend Kaffeesatzlesen aber keine Wissenschaft.

  59. Lieber Herr Wanninger,

    ich kann mich #82: LoboDelRio nur anschließen. Ich bin gerne bereit auf ehrlich gemeinte Verständnisfragen zu antworten, aber für kindische Versteckspielchen mit verbitterten Zeitgenossen verzichte ich.

    „Mir ist jedenfalls keine Disziplin in der Grundlagenforschung bekannt, in der man gezielt auf Daten verzichtet.“

    Sie haben wohl noch nie einen Forschungsantrag gestellt und mußten Streichungen hinnehmen? Bei meinem nächsten fordere, meine Kosten direkt durch die Ökosteuer einzutreiben…

    Lassen Sie sich mal von Meteorologen erklären, warum Stationen zusammengestrichen werden und
    dort sind, wo sie sind, sie Schlauberger.

    „Die Lösung scheint schon bekannt, obwohl wir noch mitten im Experiment stecken.“

    Wenn man die Augen vor der Lösung verschließt und dumm tut, kann man Ihnen tatsächlich glauben.

    Das war’s mit Ihnen.

    Lieber NF,

    ich freue mich, dass ich hier nicht mehr als einziger mit Dreck beworfen werde von Leuten, denen meine Texte inhaltlich zu hoch sind.
    Aber diese Leute haben immerhin ein Gespür für die Schaffung von Fronten, wo meiner Meinung nach gar keine bestehen müssten, es geht schließlich nur darum, wie das Klima funktioniert.

  60. Werter Herr Manninger,
    wenn sie diese Antwiort schon kennen, was ist Ihnen dann unklar? Natürlich kann man mit einer Station kein genaues Bild zum Klimaverlauf in verschiedenen Teilen Deutschlands machen, das will man ja auch nicht. Man möchte die globale Temperaturentwicklung erkennen und dazu scheint’s zu reichen. Wie geschrieben, es lässt sich relativ leicht berechnen, man hat ja die Daten. Also reduziert man sukzessive und schaut, ab wann die Ergebnisse signifikant abweichen. So hat man es wohl gemacht. Und ich gehe weiterhin davon aus, dass trotz Fehler einzelner Wissenschaftler, die NASA kompetente Statitsiker an Bord hat. Jedenfalls bessere als wir sie darstellen. Und ich gehe auch weiterhin davon aus, dass es an erschreckender Überheblichkit grenzt, hunderte Fachleute zu Lügnern und Fälschern zu degradieren. Sie finden in der Wissenschaft sicher den gleichen Anteil menschlicher Schwächen, wie in allen anderen Lebensbereichen (Politik mal ausgenommen), daraus eine Verschwörung zu konstruieren ist meiner Meinung nach sehr gewagt. Damit meine ich nicht speziell Sie, sondern einige der besonders aktiven Klimarealisten…

  61. Verehrter NB,

    ich fasse es einfach mal als Kompliment auf, wenn Herr Leistenschneider uns für identisch hält. So schlecht waren meine Argumente wohl nicht, auch wenn ich ihnen (noch!) nicht das Wasser reichen kann.

    MfG

  62. Hallo Tobias (#63),

    warum so humorlos? Ist es Ihnen evt. entgangen, dass in dem Film (http://tinyurl.com/ykzx4jj) die seriöse Klimaforschung veräppelt wird?
    Ich hätte gedacht, den Leuten hier gefällt’s und Herr NB schüttelt fassungslos den Kopf.

    Mit karnevalistischen Grüßen

  63. Herr Leistenschneider,

    Sie werden in #64 unverschämt, ich muss doch wirklich bitten, die Grundregeln dieses Forums einzuhalten.

    Sie antworteten:

    „Zu Ihren Fragen, Entschuldigung, Äußerungen.
    „Wer kann in dem genannten Diagramm einen Klimawandel von 6° erkennen?“
    Suggestivfragen sind die Argumente von Unwissenden, die in einer Thematik gerne mitreden möchten, aber das Basiswissen überschaubare Dimensionen verzeichnet. Zur Betrachtung empfehle ich Ihnen den Link, den ich Herrn Klaus V in #21 gab.“

    Herr Leistenschneider,
    warum gehen Sie überhaupt nicht auf meinem Hauptvorwurf ein sondern zitieren (wie so gerne) selektiv? Der Vorwurf war, dass das von ihnen verwendete 2.Diagramm keine Temperaturen anzeigt, sondern die relative Häufigkeit eines Sauerstoffisotops. Statt 6°C also eine Zunahme von 6 Promille von O-xx. Möge jeder Leser PUNKT1 lesen und selbst entscheiden.

    Dann zitieren Sie mich mit
    „Will der Autor ernsthaft behaupten, die klimatischen Schwankungen in der Größenordnung von Jahrzehnten seien die Zacken dieser Kurve??“
    und antworten
    „Es wäre zur Abwechslung einmal schön zu sehen, wenn Sie die Diskussion mit Hinweisen oder Belegen bereichern, anstatt mit Rhetorik. Dafür bedarf es allerdings Verständnis zur Materie.“
    Ich empfinde den Hinweis auf den zuvor genannten Fehler durchaus als Bereicherung. Auch der Hinweis, dass Sie die natürliche Variabilität von Jahrzehnten nicht mit den in der Abbildung gezeigten großskaligen Klimawandeln (Größenordnung: Jahrtausende!) verwechseln sollten, ist durchaus ein bedenkenswerter Fakt und keine Rhetorik.

    Ach ja, und als Beleg habe ich das von Ihnen benutzte Zitat aus SdW sogar noch ausführlicher angegeben, erinnern Sie sich?

    Außer Beleidigung und Diffamierung ist Ihnen leider überhaupt nichts eingefallen, was Sie meinen Argumenten entgegensetzen können.

    Es bleibt also beim 0:1, mein Angebot einer Widerlegung eines von Ihnen benannten weiteren Punktes schlagen Sie aus, also auch Endstand.

    Die Qualität naturwissenschaftlicher Forschung zeigt sich besonders im Umgang mit Fehlern, sagte einmal ein kluger Mensch. Und da ist das IPCC Ihnen um Längen überlegen.
    Anstatt Fehler zu berichtigen oder zuzugeben, verweigern Sie schon die Diskussion darüber.

    PS:
    Ich schätze Herrn Lüdecke als Physikkollegen als kompetent ein, sein Artikel zum Treibhauseffekt war wirklich gut. Er wird Ihnen meine Punkte bestätigen können.

  64. #75 NB

    Die Frage war natürlich rhetorisch!
    Mir ist jedenfalls keine Disziplin in der Grundlagenforschung bekannt, in der man gezielt auf Daten verzichtet. Mit jedem folgenden Experiment vergrößert man (aus der Erfahrung der vorangehenden) gewöhnlich gezielt die Anzahl Messpunkte dort wo’s am interessantesten ist. Bei Klima interessieren beispielsweise möglichst fein nach Ort und Zeit aufgelöste Temperaturpunkte. Es sei denn, man glaubt, die Natur schon entschlüsselt zu haben und will nur noch längst bekannte Trends bestätigen. Und damit wären wir ja dann beim Punkt. Die Lösung scheint schon bekannt, obwohl wir noch mitten im Experiment stecken.
    Und weil das eigentlich jedem klar ist, war das deshalb ja auch eine rhetorische Frage!
    Man sollte es auch immer im Zusammenhang mit dem Bild in Punkt 7 sehen und dass das wohl kein Einzelfall sondern womöglich (d’Aleo) die Regel ist.

  65. Werter Herr NB,
    Ich denke Ihnen sehr fuer Ihre Antwort. Bevor ich darauf eingehe, moechte ich Sie bitten auch auf meine zweite Frage zu antworten.

    Danke sehr.

    mit freundlichen Gruessen

    E.

  66. Lieber Herr Leistenschneider in#64

    es geht hier ums Klima. Anstatt hier über irrelevante Nebensächlichkeiten zu mokieren, wie wäre es, wenn Sie einfach sicherstellen, dass Sie Ihren Ansprüchen an Transparenz, Korrektheit und erschöpfender Recherche selber gerecht werden.
    Denn hier gibt es auch genug stille Leser, die also noch anonymer als Pseudonyme sind. Ich wäre Ihnen dankbar, wenn Sie das gleich mal bei der r Beantwortung der Fragen unter #50, #52, #55 umsetzen würden. Danke!

  67. Lieber Herr Ebenherz,

    Die Beobachtung ist richtig: die CO2-Emissionen folgen zeitlich der Temperatur. Was man dort sieht, ist der hier öfter schon beschriebene und stets – heute wie in der Vergangenheit – ablaufende Prozeß: Ohne anderweitige Störungen durch CO2-Quellen und -Senken bildet sich zwischen dem Gehalt von CO2 im Ozean und in der Atmosphäre ein Verhältnis (chemisches Gleichgewicht) aus. Im Wasser ist pro Liter viel mehr CO2 gelöst als in einem Liter Luft. Dieses Verhältnis (Henrysches Gesetz) ist aber von der Temperatur abhängig. Wenn die Temperatur der Ozeanoberfläche steigt, so tritt CO2 aus dem Ozean in die Atmosphäre über. Die Konzentration in der Luft steigt, während sie sich im Ozean kaum ändert (wegen der ohnehin höheren Konzentration).

    Der Zeitverlauf von CO2 und Temperatur zeigt, dass das CO2 der Temperatur folgt. Damit wird der kausale Zusammenhang, eine Temperaturerhöhung bewirkt eine CO2-Erhöhung suggeriert. Dieser Zusammenhang ist richtig und erklärt sich nach obigem Zusammenhang.
    Das Disgramm erklärt aber nicht, warum sich die Temperatur eigentlich ändert. Das kann i.a. viele Gründe haben, die quantitativen Anteile der Gründe bekommt man nur durch quantitative Klimamodelle heraus. Damit kommt raus, dass die letzten Eiszeiten durch Änderungen der Einstrahlung aufgrund von Erdbahnänderungen ausgelöst wurden, aber dies alleine erklärt nicht die Beobachtungen. Diese Einstrahlungsänderung war nur der Auslöser für eine interne Kettenreaktion des Klimasystems (Eis-Albedo-feedback), die zur Glazial-Interglazialdynamik führte. Die Temperaturschwankungen kommen in absteigender quantitativer Wirkung durch das Eis-Albedo-feedback, durch Änderungen der Treibhauskonzentration und durch Aerosole zustande.

    Die Treibhauswirksamkeit des CO2 führt also dazu, dass die Temperatur- und CO2-Änderungen wechselseitig abhängig sind: eine höhere Temperatur führt zu mehr CO2 in der Luft, dies führt via Treibhauseffektverstärkung zu einer weiteren Temperaturerhöhung etc. So schaukeln sich beide Parameter hoch bis ein neues Gleichgewicht erreicht ist. Dies sehen Sie in den Kurven. Dabei kann man ablesen, dass etwa pro 1 C Temperaturerhöhung 10 ppm CO2 ausgasen. Aber dieser Prozeß benötigt Zeit, dies geschieht über 100e von Jahren.

    Zur Zeit ist diesem Prozeß jedoch überlagert, dass CO2 neben dem Ozean noch zusätzlich vom Menschen freigesetzt wird. Das chemische Gleichgewicht muss sich darauf einstellen. Wenn also erstmal CO2 in die Atmosphäre geht, so wird nachfolgend das CO2 aufgrund des Gleichgewichts solange ins Wasser wandern, bis zwischen Wasser und Luft das Gleichgewicht wieder eingestellt ist. Momentan haben wir kein Gleichgewicht, aber durch den Prozeß reduziert sich die anthropogene Menge in der Luft auf ca. die Hälfte der emittierten Menge.

    Fazit: Es gibt keine einseitige Abhängigkeit von CO2 zur Temperatur. Daher läßt sich aus der Beobachtung Temperatur vor CO2 nicht folgern, dass die umgekehrte Abhängigkeit nicht existiert. Im Gegenteil, die Kurve läßt sich ohne die wechselseitige Abhängigkeit gar nicht erklären. Davon abgesehen ist ja evident, dass man CO2 auch ohne das Klima in der Atmossphäre anreichern kann (das machen wir ja gerade).

    Lieber Herr Wanninger,

    haben Sie schon einmal zur Beantwortung Ihrer Fragen gegooglet?
    Ihre Fragen sind ja nun alles andere als vorurteilsfrei. Was legt z.B. fest, was eine „lächerlich kleinen Zahl von Messpunkten“ ist, und was heißt „GROB FEHLERANFÄLLIG“ oder wieviel würde sich am Ergebnis verbessern, wenn man 50mal mehr Stationen hätte?

    Glauben Sie wirklich, Sie sind der Erste, der über so etwas nachdenkt? Glauben Sie, dass Messfehler und die Anzahl der Stationen bei der Ermittlung der globalen Temperatur nicht berechnet wurden? Glauben Sie, die Leute, die das berechnen, sind allesamt Vollidioten oder Betrüger?

    Wenn Sie so wie ich mal Physik studiert haben, so setzten Sie sich hin, lesen Sie die Literatur und rechnen Sie die Fehlerintervalle nach, anstatt eine nach der anderen These auf dem Bauch heraus zu postulieren und dann noch Ihren Kram von anderen bewiesen haben zu wollen. Erarbeiten Sie sich Ihre Antworten selber, oder haben Sie Angst vor der Wahrheit?

  68. #73 Lobodelrio
    Diese Antwort kenne ich. Sie kann aber doch nicht ernsthaft befriedigen! Haben wir nicht gerade in Deutschland erlebt, wie unterschiedlich das Wetter sein kann? Um das mal auf die USA zu beziehen, hätte also für Deutschland ein einzelnes Thermometer in Frankfurt/Main oder sonstwo ausgereicht, um den Temperaturverlauf repräsentativ aufzunehmen?
    Um aber allein die ‚deutsche Mitteltemperatur für 2010‘ einigermaßen genau zu bestimmen, reicht das wohl kaum aus -> Abtastungstheorem!
    Und was man mit Statistik machen kann, gewollt oder ungewollt, weiss wohl jeder hier. Wenn nicht, lese man ‚Der Hund, der Eier legt‘.

  69. Werter Herr Wanninger,
    ihre Frage wird von offiziellen Seiten (NASA)beantwortet. Man hat einfach durchgerechnet, wieviele Stationen man braucht, um sichere statistische Auswertungen zu machen. Z.B. kann man das Klimageschehen der USA mit 10 000 Meßstationen oder auch mit 50 gut Verteilten bestimmen, und es kommt nahezu das selbe raus (so zumindest die Angaben). Das liegt sicher daran, dass es bei Klima-Temperaturanomalien vor allem auf Großwetterlagen und nicht lokalen Einzelwerten ankommt. So wie ich das verstanden habe, kann dass jeder privat nachrechnen, so er denn möchte. Für mich macht das Sinn. Natürlich kann ich auch Betrug wittern, aber dass dann bei jeder Messung, die ich nicht selber in meinem Garten gemacht habe. Da man nur Vergleiche macht, die Absoluttemperatur keine Rolle spielt, könnte man sogar auch zahlreiche kalte Orte weglassen, am Trend sollte sich nichts ändern, solange man nicht ausschließlich Tendenzen nach unten eleminiert hat.
    Grüße LoboDelRio

  70. #71 Marvin Müller,

    das werde ich.
    Aber das ändert grundsätzlich nichts daran, dass 1000 Mess-Stellen sind, ja selbst 10000 völlig unbefriedigend sind, egal wie gut die erfassten Daten sein mögen. Und das ist wohl eine Kernfrage bei dem völlig kontrovers diskutierten Punkt, ob die globale Erwärmung nun stagniert oder nicht. Und hier hat sich in 20 Jahren offensichtlich nichts getan??

  71. #67: Jürgen Wanninger sagt:

    „Wenn es meteorologische Mess-Stationen wie in Punkt 7) abgebildet gibt und das, wie vielfach publiziert, nicht nur Einzelfälle sind, würden wir die ‚offizielle‘ Temperaturmessung dann als

    a) seriöse Wissenschaft
    b) Junk-Science

    bezeichnen?“

    Wollten Sie nicht erst mal das Menninger Papier lesen, bevor Sie sich weiter zu den Stationen äußern? Die gehen genau auf die Güte der Messwerte ein.

  72. Werter Herr NB,
    Das missverstehen Sie. Ich bin an einer Antwort sehr interessiert, da die Fakten, die hier ansonsten diskutiert werden, durch mich nicht beurteilt werden koennen.

    Meine Definition gilt weiterhin. Dies allerdings nur als Bekraeftigung, dass es mir wirklich ernst ist. Ich wollte Sie nicht zur Eile draengen.

    Mit freundlichen Gruessen

    E.

  73. #68, NB

    Schade, Ihre Antwort hätte mich auch interssiert, wie auch die auf meine oben #66, #67 gestellten zwei Fragen.

    Sein Sie mit Herrn Ebenherz nachsichtig. Ich denke er war deshalb ungeduldig, weil Sie zwischenzeitlich ein längeres Posting eingestellt haben.

  74. Lieber Herr Ebenherz,

    wie Sie meinen. Dann weiß ich zumindest, dass ich an meinen Text an Sie nicht mehr weiterarbeiten muss, denn Sie sind daran offenbar eh nicht ernsthaft interessiert gewesen.

    Was sind Sie nur für Menschen? Stellen rhetorische Fragen und rechtfertigen Ihre vorgefertigte Meinung, weil Sie nach ein paar Stunden noch keine gegenteilige Antwort haben?

    Geht’s noch?

  75. und noch ’ne Frage an AGW-Befürworter:

    Wenn es meteorologische Mess-Stationen wie in Punkt 7) abgebildet gibt und das, wie vielfach publiziert, nicht nur Einzelfälle sind, würden wir die ‚offizielle‘ Temperaturmessung dann als

    a) seriöse Wissenschaft
    b) Junk-Science

    bezeichnen?

  76. und nachmals die Frage an NF, NB, Marvin Müller, Wissender u.a. Befürworter des AGW:

    Wieso wird eine globale Mitteltemperatur aus einer lächerlich kleinen Zahl von Messpunkten und damit aufgrund der Lokalität von Wetter GROB FEHLERANFÄLLIG bestimmt? Wieso hat man in den letzten 20 Jahren kein Interesse, die Datenbasis, auf das 10, 20 oder 50fache zu stellen, was höchst wünschenswert sein müsste. Und es würde praktisch nichts im Vergleich dazu kosten, was an Geld in der Klimaforschung umgesetzt wird. Warum?
    Vielleicht lässt sich ein Befürworter des AGW einmal herab und gibt mir eine Antwort oder bestätigt die Sinnhaftigkeit meiner Kritik.

    P.S. Danke an den Admin für die schnelle Korrektur!

  77. #NF
    ziemlich geschmackloser Beitrag, das Filmchen.

    Und darauf läuft es letztendlich, soweit ich beobachte, immer heraus. Diskreditieren, Polemisieren, andere Meinungen durch Disqualifizieren des Autors unglaubwürdig machen.

    Die größte Leistung unserer ‚ehemals‘ freien Gesellschaften ist der freie Geist, das freie Wort und die freie Wissenschaft. Ob etabliert oder nicht. Auf wessen Geist wandelt der Erzeuger dieses Video-Pamphlets? Indem man ‚Skeptiker‘ in eine Ecke mit Faschisten stellt macht man sich selbst zum Faschisten!

    Ergänzend kann ich nur auf diesen glänzenden Beitrag von vor zwei Wochen in 3SAT hinweisen:

  78. Sehr geehrter Herr NF, alias NB oder wie auch immer,

    Wissenschaft fängt bei Transparenz und Seriosität an. Ihre verschiedenen Pseudonyme sind denn auch wenig geeignet, Seriosität zu unterstreichen. Ihre Einleitung:
    “ In #4 war ich wohl doch zu optimistisch, was die Urteilskraft mancher Leser angeht“ unterstreicht, dass es sich möglicherweise um ein Überbleibsel pubertärer Selbstüberschätzung handelt, seine Person zu vervielfältigen. Wie aus der Datenverarbeitung bekannt, werden Kopien jedoch niemals besser, sondern, je mehr davon hergestellt werden, immer schlechter.

    Zu Ihren Fragen, Entschuldigung, Äußerungen.

    „Wer kann in dem genannten Diagramm einen Klimawandel von 6° erkennen?“

    Suggestivfragen sind die Argumente von Unwissenden, die in einer Thematik gerne mitreden möchten, aber das Basiswissen überschaubare Dimensionen verzeichnet. Zur Betrachtung empfehle ich Ihnen den Link, den ich Herrn Klaus V in #21 gab.

    „Will der Autor ernsthaft behaupten, die klimatischen Schwankungen in der Größenordnung von Jahrzehnten seien die Zacken dieser Kurve??“

    Es wäre zur Abwechslung einmal schön zu sehen, wenn Sie die Diskussion mit Hinweisen oder Belegen bereichern, anstatt mit Rhetorik. Dafür bedarf es allerdings Verständnis zur Materie.

    Aber Selbstüberschätzung zeigt sich bereits in der Vervielfältigung einer Persönlichkeit.

    Viele Grüße RL.

  79. #47: NF sagt:
    am Mittwoch, 03.02.2010, 19:07

    Jawoll, lieber Tobias.

    Und für alle, die es nicht glauben:
    Hitler war der erste ÖKODIKTATOR und außerdem Komplize von Michael Mann.

    Beweis auf Youtube gefunden:
    http://tinyurl.com/ykzx4jj

    (Unbestätigten Quellen zufolge war sogar ein Tempolimit auf der Autobahn geplant, die Alliierten konnten dies aber im letzten Moment abwenden.)

    Toll Lieber Herr „NF“!! Wirklich toll! Sie versuchen wohl Geschichtsverfälschung zu betreiben? Stellen Sie sich damit nicht auf eine Stufe mit Verschwörern, Lügnern und den Datenfälschern des IPCC?? (Hockeydiagramm, Klima-, Gletschergate uvam)Als „Wissenschaftler“ gehören die geächtet!
    Aber abwarten! In absehbarer Zeit werden sich vermutlich die Herren Staatsanwälte damit beschäftigen. Mit vorsätzlich gefälschten Daten wurden Milliarden vernichtet!

  80. #57 Hartmann

    Klasse Beitrag! Mal die Größenordnungen zurechtgerückt!
    Die Wohlstandsgesellschaften haben ein Problem – sie nehmen sich zu wichtig! Religion und Gesellschaft sind in den Hintergrund gerückt – das Individuum erhebt sich selbst zum Gott!

  81. Werter Herr NB,
    Ich wollte Sie noch ueber folgende Definition in Kenntnis setzen:

    Keine Antwort ist hier eine eindeutige Antwort.

    mit freundlichen Gruessen

    E.

  82. #41 Marin B

    richtig, und die Gletscher sind das gleiche Ding. Man braucht keinerlei Naturwissenschaft zu betreiben. Allein aus Chroniken und kulturhistorischen Betrachtungen heraus sieht man, wie stark die Ausdehnung der Alpengletscher in den letzten 2000 Jahren geschwankt haben MUSS. Stellt man dann Aussagen von Schellnhuber u.a. dagegen, kann man letztere doch nur für Lügner halten.

  83. in meiner letzten Meldung muss es richtig heißen:

    Erdvolumen: 1.083?10hoch12 km3,
    Volumen der Ozeane: 1.37?10hoch9 km3

    Bitte um Verzeihung!

  84. Danke an die EIKE-Experten! Sehr guter Artikel was die direkte Gegenüberstellung der Argumente angeht: hier die von Rahmstorf, Schellnhuber, Marotze und Co praktizierte Verfälschung AGW-kritischer Argumente – und dort die Demontage eben dieser Ansammlung von Halbwahrheiten.

    Deja vu !!! Erinnert mich sehr positiv an die Auseinandersetzung des Herrn Rahmstorf mit Dipl.Physiker Alvo von Alvensleben. Auf die zweite Replik von Herrn von Alvensleben, welche Rahmstorfs Argumenten endgültig den Boden entzog, bekam letzterer keine Argumente mehr zusammen und blieb jegliche Antwort schuldig.
    Zur Erinnerung: http://www.schulphysik.de/klima/alvens/antwort.html

    Die dezidierte, sachorientierte Punkt-für-Punkt Auseinandersetzung ist also nicht neu. AGW-Oberkleriker wie Rahmstorf dürfte klar sein, dass sie unter wirklichen Fachleuten schnell Gefahr laufen, den kürzeren zu ziehen. Deswegen haben sie sich ja bisher auch lieber gegenübern Kritikern eingemauert. Aber die Realitäten zwingen diese Herrschaften nun mehr und mehr ans Licht. Und das ist gut so!

  85. Es spricht nicht gerade für Schellnhuber & Co., dass sie sich den FOCUS als populistische Zeitschrift zweifelhaftester Qualität für ihre Debatte ausgesucht haben. Polemik, die EIKE vorgeworfen wird, ist eben dort so wie so schon vorprogrammiert. Aber wer ist schon frei von Polemik. Sie ist nur allzu menschlich und vor allem in wohldosierter Form auch akzeptabel, besonders dann, wenn sie auf der Seite der Wahrheit steht. Aber wer kennt die schon? Und genau an dieser Stelle kommt einem schon manchmal das große Erbrechen, wie sich hier so mancher als all“wissend“ aufspielt. Offensichtlich fehlt es dort an einer gewissen Lebenserfahrung und auch „Wissenschaftserfahrung“, um zu dem Schluss zu gelangen, dass viel mehr Gewicht in der Erkenntnis liegt, zu wissen, dass man eigentlich zu wenig oder gar nichts weiß bzw. auch an welcher Stelle man nichts weiß. Diese Erkenntnis kann unschätzbaren Wert haben und vor schmerzhaften Beulen schützen, die man sich bei ihrem Fehlen beim Versuch, eine Wand einzurennen, holen kann. Ich hoffe, dass diese Dialektik gut verständlich ist!

    Der Versuch der wirklich sehr polemischen Kritik der „Wahrheitspächter“ zu entgegnen ist äußerst ehrenhaft und deshalb auch ganz ohne Polemik unmöglich.

    Wenn hier einige enttäuscht vom Artikel sind, zeugt das für mich nur von Voreingenommenheit. Was wurde denn erwartet. Die Darbietung einer Weltformel oder wie? Andere wiederum sprechen von „Klimmzügen“, die EIKE hier macht. Sind zum Beispiel die, physikalisch zwar plausiblen, postulierten Rückkopplungseffekte u. a., die nie gemessen wurden, etwa keine Klimmzüge? Also..!

    Die Kenntnisse beispielsweise über den Kohlenstoffkreislauf im System Erde weisen objektiver Weise erhebliche Lücken auf. Wer etwas Gegenteiliges behauptet, weiß nicht wovon er spricht bzw. disqualifiziert sich wenigstens in der mich umgebenden Fachwelt (natürlich keine Klimaspezialisten). In dieser Hinsicht kann ich die EIKE-Entgegnung zu Pkt. 3 nur unterstützen. (Diese zeugt für mich übrigens auch davon, dass einige Wissenschaftszweige aus der Klimaforschung ausgeblendet sind beispielsweise die Geologie/Geophysik (oder irre ich mich?))
    Bestätigend und ergänzend kann ich bezeugen, dass keine repräsentativen, verallgemeinungswürdigen, global anwendbaren Daten über Ausgasungen in Vulkangebieten (nicht nur Vulkanschlote, ihre vulkanisch aktive Umgebung u. a. mit Schlammvulkanen, Geysiren, Solfataren ist um ein Vielfaches größer)und z. B. aus den Räumen der zigtausende km langen mittelozeanischen Rücken einfach noch nicht zur Verfügung stehen, nicht ermittelt sind. Andererseits sind dort auch die Subduktionszonen (wo die tektonischen Platten „untertauchen“), wo massenweise Kohlenstoff in den oberen Erdmantel gelangt. Es gibt all zu dem keine großflächigen weltweit angelegten Messfelder, keine langzeitlichen, keinerlei kontinuierlichen Messreihen. An dieser Stelle hat unser Wissen schlichtweg einen weißen Fleck. Das ist einfach nur Fakt! (Und da wird sich hinter der fünften Stelle nach dem Komma gestritten, lachhaft). Wer das ernsthaft bestreiten möchte, sollte sich sehr warm anziehen bzw. schon `mal sicherheitshalber eine Zwangsjacke bestellen (sehen Sie die Polemik, um die ich auch nicht herum komme!).

    Zum Schluss hier noch ein paar interessante Informationen usw. zur Veranschaulichung:

    „…Zu früheren Zeiten (der Erdgeschichte), als zweifellos keine Gletscher existierten, sind Meeresspiegelabsenkungen um 300 – 400 m erreicht worden. Der wahrscheinlichste Grund für höhere Tempi von Meeresspiegelschwankungen gegen Ende des Känozoikums besteht in der schnelleren Aktivierung tektonischer Prozesse, die eine intensive Gebirgsbildung sowie andererseits eine Ozeanisierung bedingten. Außerdem gibt es keinerlei Erkenntnisse über Kaltzeiten im Mesozoikum und dem Frühkänozoikum… Megastrukturen der Erde: Bei einem mittleren Erdradius von 6371,032 km beträgt die Mächtigkeit der Erdkruste nur einige Dutzende Kilometer (ca. 0,5 % des Erdradius). Das Erdvolumen beträgt 1.083?1012 km3, das Volumen der Ozeane 1.37?109 km3 (0,1 % des Erdvolumens), das Gesamtvolumen des rezenten Gletschereises ca. 30 Mill. km3 (ca. 0,002 % des Erdvolumens).“

    „Die Größenordnungen der aufgezählten Größen liefern den Grund zu schlussfolgern, dass die Änderungen des Volumens der Ozeane in erster Linie von Prozessen kontrolliert werden, die unterhalb der Mohorovi?i?-Grenze ablaufen, die die Trennlinie der Erdkruste (20 – 40 km) vom oberen Erdmantel markiert und eine Masse umschließt, welche tausendfach die Masse der ozeanischen Wässer und der übrigen Wässer auf der Erdoberfläche einschließlich aller Gletscher übersteigt. Des Weiteren sind innerhalb von mehr als 90 % der Zeit der geologischen Geschichte der Ozeane Meeresspiegelschwankungen und Volumenänderungen der Ozeane ohne jegliche Beteiligung irgend welcher Gletscher von statten gegangen. Offensichtlich sind diese Schwankungen u. a. mit Prozessen der Ausgasung und Dehydrierung von aus dem Erdinneren ausgetretenen Material zu begründen.“

    „Unter Nutzung der Telealtimetrie (auch mittels Satelliten) konnte gezeigt werden, dass ein Null-Niveau der Ozeane eine Art gemittelter Abstraktwert ist. Die von ihm größten positiven Abweichungen wurden im Nordatlantik ermittelt (bis 68 m), maximal negative (112 m) in der äquatorialen Zone des Indischen Ozeans südlich von Shri Lanka… Noch deutlicher sind die Angaben vom Kosmischen Zentrum von Harvard gemäß derer die Geoidform und die Schwereanomalien in keiner Weise weder mit der Platznahme rezenter Gletscher, noch mit der Lage vermuteter früherer Gletscher, noch mit der Topografie der Meeresoberfläche, noch mit der Geografie der Gebirge korrelieren. Nicht die dünne „Haut“ der Lithosphäre übt Druck auf die tiefer liegenden Sphären aus, bestimmt das Relief der Ozeanböden, ist verantwortlich für das Ozeanvolumen und das Niveau des Meeresspiegels sowie die Abläufe tektonischer Prozesse, sondern magmatische Tiefenprozesse. Die endogenen Faktoren sind ungleich stärker…“

  86. In #4 war ich wohl doch zu optimistisch, was die Urteilskraft mancher Leser angeht. Deshalb möchte ich bei Punkt 1 doch etwas detaillierter argumentieren. Dort wurde geschrieben:

    „Dies ist definitiv eine Falschaussage, wie die Untersuchungen von Rahmstorf (2003, siehe Abbildung) zeigen. In dieser Untersuchung ist zu sehen, dass es Klimawandel mit einer Temperaturänderung von mehr als 6°C innerhalb von wenigen Jahrzehnten gab. In Spektrum der Wissenschaft, 11/09, S. 70 steht zu diesen krassen Klimawandel: “Die Schwankungen vollzogen sich bisweilen so schnell, dass einzelne Individuen sogar im Lauf ihres Lebens beobachten konnten, wie Pflanzen und Tiere, mit denen sie noch aufgewachsen waren, verschwanden und durch eine ihnen unvertraute Flora und Fauna ersetzt wurden.“ Und da die damaligen Individuen ein sehr kurzes Leben hatten (mit 30 Jahren war man alt), fand dieser Klimawandel von 6°C und mehr innerhalb von 2-3 Jahrzehnten statt.“

    Wer kann in dem genannten Diagramm einen Klimawandel von 6° erkennen? Wohl nur der Autor (Herr Leistenschneider vielleicht?), dem wohl vollkommen entgangen ist, dass auf der y-Achse überhaupt keine Temperaturen stehen, sondern die Abnahmen eines Sauerstoffisotops in Promille angegeben sind. Okay, jetzt sehe ich einen Klimawandel mit 6 Promille Zunahme dieses Isotops, aber warum dies jetzt Marotzke et al. widerlegen soll, bleibt mehr als unklar.

    Genau so kühn und kreativ geht es dann weiter:
    „In Spektrum der Wissenschaft, 11/09, S. 70 steht zu diesen krassen Klimawandel“. Will der Autor ernsthaft behaupten, die klimatischen Schwankungen in der Größenordnung von Jahrzehnten seien die Zacken dieser Kurve??
    Der Leser, der in diesem Diagramm Änderungen binnen Jahrzehnten erkennen kann, muss übersinnliche Adleraugen besitzen.

    In heutiger Fachsprache würden wir sagen, der Bericht in SdW bezieht sich auf die natürliche Variabilität, die dem Klimawandel (Eiszeit) aufgeprägt war. Es ist nicht seriös, Anstiege der natürlichen Variabilität mit durchschnittlichen Anstiegen eines langfristigen Klimawandels zu vergleichen (Äpfel und Birnen vergleichen).

    Übrigens war den Autoren des SdW-Artikels dieser Unterschied durchaus bewusst, ich zitiere einfach mal etwas umfassender:

    „Die Isotopenanalysen zeigen aber auch, dass das Klima sich keineswegs stetig von milden zu kalten Temperaturen veränderte; vielmehr wurde es auf dem Weg zur maximalen Vereisung zunehmend instabil und schwankte stark. Diese Oszillationen lösten tief greifende ökologische Veränderungen aus: Die Wälder wichen baumlosen Graslandschaften, Rentiere ersetzten Nashörner. Die Schwankungen vollzogen sich so schnell, dass einzelne Individuen…“ und jetzt weiter wie vom Autor zitiert.

    Damit steht es nach Punkt 1 schon 1:0 gegen EIKE.

    Mir fehlt die Zeit, jetzt die übrigen 9 Punkte von EIKE zu widerlegen, deshalb biete ich folgendes an:
    Sie, Herr Leistenschneider, bestimmen einen einzigen, weiteren Punkt, den ich widerlegen soll. Antwort folgt dann.

    PS: Wer so viele Fehler verbreitet, wirkt wenig überzeugend in der Kritik an einem Zahlendreher in einem IPCC-Bericht.

  87. zu Punkt 2

    „Hier liegt ein einfacher Rechenfehler vor (siehe E. Schulze), wie auch die Arbeit von Knorr bestätigt. Anthropogenes CO2 ist seit 200 Jahren gleichbleibend und nicht vom Rauschen zu unterscheiden,“

    E. Schulzes Artikel hätte ich auch gerne sowie die übrige wissenschaftliche Literatur, die bestätigt, dass H. Suess ein Rechenfehler unterlaufen ist. Was besagen moderne Isotopenmessungen (links zu papers bitte)

    Haben Sie das paper von W.Knorr überhaupt angeguckt oder rechnen Sie damit, dass Ihre Leser Ihnen sowieso alles glauben oder Ihre Referenzen eh nicht vestehen?
    In Abbildung 1 von Knorr werden die anthropogen Emissionsraten von CO2 (in GtC/Jahr) und die atmosphärischen Anstiegsraten von CO2 ab 1850 abgebildet.
    Sie sehen, dass die Kurven parallel laufen, und die daraus durch Division abzuleitende air borne fraction ist ca. 46% (die Abweichungen davon sind der Untersuchungsgegenstand des papers). Nun integrieren Sie die atmosphärische Kurve und bekommen raus, dass von 1850 bis 2000 der CO2-Anteil in der Luft um ca. 80 ppm gestiegen ist, bzw. entsprechend um 170 GtC. Fragen Sie mich, wenn Sie bei der Überprüfung meiner Ergebnisse nicht weiterkommen, man muss dafür schon Mathe und Chemie Abiniveau drauf haben.

    Zu Punkt 3:

    Können Sie Ihre eigenen Zitate nicht lesen? Sie zitieren:
    „Zu dieser Behauptung verweisen die Autoren auf die Arbeiten des Instituts für Angewandte-Geologie der Uni-Erlangen: “Insgesamt stammt der Hauptteil des CO2 in der Erdatmosphäre aus dem Erdinneren, kommt aber nicht nur aus den Vulkanen, sondern tritt als sog. „juveniles Kohlendioxid“ aus einer Vielzahl von Stellen, die teilweise recht unspektakulär aussehen. Es ist dies eine natürliche Folge der Plattentektonik,… Eine exakte Quantifizierung der gesamten, aus dem Erdinneren ausströmenden CO2-Mengen ist heute noch nicht möglich. Dies liegt einfach darin begründet, daß es unmöglich ist, an einer genügend großen Anzahl von Stellen (so z.B. an den Ozeanböden) kontinuierlich zu messen“ (http://www.angewandte-geologie.geol.uni-erlangen.de/klima1.htm).
    Noch ein paar weitere Zahlen, die die Dominanz der durch vulkanische Aktivität austretenden sog. Treibhausgase zeigt.“
    Das Institut schreibt wissenschaftlich korrekt, dass das meiste CO2 in der Erdatmosphäre aus dem ursprünglich Erdinneren stammt und dieser Prozeß (=Emissionsprozeß des langsamen geologischen Kohlenstoffzyklus) heute noch existiert.

    Dieses so emittierte CO2 zirkuliert heute im sog. schnellen Kohlenstoffkreislauf zwischen Biosphäre, Ozean und Atmosphäre herum mit der Rate der genannten 100 Gigatonnen Kohlenstoff zwischen Ozeanen und Atmosphäre und ca. 120 GtC/Jahr mit der Biosphäre. Ein Teil zirkuliert mit einer Rate von 0,2 GtC/Jahr auch in dem oben genannten geologischen langsamen Kreislauf herum (Sedimentation versus Verwitterung). Die sehen, dass die Emissions- und Abbaurate von 0,2 GtC viel geringer ist als die Raten des schnelllen Zyklus sowie die Emmissionsrate durch den Menschen.

    Wer so wie Sie 0,2 GtC/Jahr als dominant gegenüber 7 GtC/Jahr oder 100 GtC/Jahr sieht, hat da offensichtlich den Kohlenstoffzyklus grundlegend nicht verstanden!

    Zu vulkanischen CO2 Mengen schreiben Sie nichts, weil man nichts genaues weiß. Wenn sie aber mal kapieren würden, was man weiß, so würden Sie nicht ständig von Spekulationen sprechen und kämen selber darauf, dass die Rate sicher kleiner als 0,2 GtC/Jahr beträgt. Stattdessen geben Sie natürliche Emissionsraten von Methan an. CO2 war aber das Thema!

    Dann möchte ich wissen, wie die Emissionsraten („1,97 . 1011 bis 1,23 . 1014 m³ Methan/Jahr, wovon 4,66 . 107 bis 3,28 . 1011 m³ aus Oberflächenvulkanen stammen. Dies entspricht einer Gewichtsmenge von 141 – 88.000 Tg/Jahr (1Tg = 1 Million metrische Tonnen)“) mit der Gesamtmasse von Methan in der Atmosphäre zusammenpasst.
    Die Erdatmosphäre enthält z.Zt. 1,7ppm Methan, was einer Gesamtmasse von 5000 Millionen Tonnen (5 Gt = 5000 Tg) entspricht! Nun?

    Auch Ihren anderen Entgegenhaltungen folgen wieder dem EIKE-Schema unter der Fuchtel von Limburg und seinen willigen Verschwörungsgenossen Ernst Georg Beck und Dipl.-Ing. Raimund Leistenschneider.

    FAZIT: Eine nette Dokumentation, die die fachliche Unfähigkeit von EIKE-Mitgliedern unzweifelhaft offenlegt.

    PS: Wenn kommt es zur Aufspaltung EIKEs in die Lager um Lüdecke und Limburg? Es gibt ja offensichtlich gravierende Unterschiede in der fachlichen Kompetenz zu dem Thema zwischen Physikern und dem Rest. Das kann doch nicht lange gut gehen?

  88. Hier noch was aus dem Beipackzettel von Dennerle Planta Gold 7

    Dennerle ist ein Renomierter Herseller von Aquariestik Zubehör.
    Panta Gold ist ein Präperat zur Pflanzenernährung im Aquarium.

    Zitat: Co² (Kolendioxid) Ist der wichtigste Pflanzendünger Überhaupt. Z.Ende

    Und wenn es jemand nicht glauben will schickt ihn ins nächste Zoofachgeschäft.

  89. Hier mal meine ganz eigene Geschichte.

    Es war im Jahr 2005 da hab ich den Schrott von der Erderwärmung durch CO² geglaubt.
    Und Ich hatte ehrlich gesagt richtig Angst vor dem Drohenden Klima Gau.

    2 Grad mehr 3 Grad mehr und dann beginnt auf dem Meeresboden das Methan auszugasen und dann schraubt sich das ganze ohne Ende hoch.

    Angst Angst und….

    Das hat der IPCC geschaft.

    Dann ist mitte 2008 die Ausgabe 5/2008 der Factum (www.factum-magazin.ch/wFactum_de/)
    bei mir eingetroffen.
    Der lange ausfürliche und Fundierte Bericht hies „Die überhitzte Prognose.“
    Darin wurden die Arbeiten von Ernst Georg Beck
    (180 Jahre atmosp. CO² Gasanalyse mittels Chemischer Methoden.)
    Die arbeit von Dr. Heinz Hug und anderer Wissenschaftler wurden ebenfalls erwähnt.
    Die Gasanalyse habe ich aufmerksam gelesen.

    Ab da war mir klar mit dem IPCC stimmt was nicht.
    Ich hatte vortan den Kopf wider frei zum klar dencken und eigene Beobachtungen zu machen.

    Und nach meinen Eigenen Beobachtungen wird es Seit 2003 in Karlsruhe Klimatisch kälter.
    Im folgenden viel mir dann auf das hinter der Klima sache eine riesige Lobby steckt.
    Das die Klimaangst vermarktet wird.

    Dann am 27.11.2009 hat die Faczum gemeldet.
    Hackerangriff IPCC: Kommen jetzt die guten Fragen?
    (27. November 2009/tl.) – Eine Studie belegt zweifelsfrei: Die Mehrheit der in Deutschland mit Klimatologie befassten Wissenschaftler bezweifelt die These einer von Menschen verursachten Klimaerwärmung. In der Öffentlichkeit finden sie aber wenig Gehör. Bahnt sich jetzt ein Umdenken an, nachdem es Hinweise auf kriminelle Manipulation von Daten durch Wissenschaftler des Weltklimarates gibt?
    mehr…

    12 Stunden internet Recherche brachten mich unter anderem auf die EIKE seite.
    Und nach dem Umrechnen des CO² ppm wertes in Prozent und einige weitere Spielcht dieser Art weis ich eigentlich nicht mehr wie ich überhaupt glauben konnte das CO² Klimarelevant sein soll.

    Heute bin ich ein manchmal unerträglicher „Klimaskeptiker.“

    mfG

    Persönlicher dank an Ernst Georg Beck und Dr.Heinz Hug

    Leider Regiert in Deutschland die Angst noch viel zu sehr.

  90. Zu Punkt 4

    „Die Abbildung zeigt die Änderung der globalen relativen Luftfeuchte für unterschiedliche Höhen anhand von NASA-Satellitendaten.“

    1948 gabe es keine Satelliten. Bitte geben Sie preis, woher die Daten sind (wahrscheinlich zeigen Sie die fehlerbehafteten sounding-Daten). Satellitenmessungen zeigen einen globalen Anstieg und keinen Abfall, den Ihre Messungen, oder was immer sie da abbilden, zeigen.

  91. Ich habe eine Frage an Herrn NB.

    Vorerst noch das. Ich bin kein Wissenschaftler, und habe auch keine Kenntnisse von Geologie, Metrologie oder tiefer gehenden physikalischen Zusammenhaengen.

    Jedoch glaube ich an Fakten und deren logischen Konsequenzen.

    Die Grafik bei 2. ‚Vostok Ice Cores 150000-100000 years ago‘ zeigt deutlich einen Anstieg der Temperatur vor dem Anstieg des CO2.

    Ich gehe davon aus, dass diese Daten glaubwuerdig sind. Dann kann ich folgende Aussage treffen:
    Vor 150000 Jahren ist in Ostsibirien fuer 50000 Jahre nicht die Temperatur dem CO2-Anstieg gefolgt, sondern genau anders herum. Das widerspricht der Theorie CO2 koennte de Temperatur beeinflussen diametral.

    Was ist Ihre Erklaerung dafuer?

    In diesem Zusammenhang eine zweite Frage:

    Niemand bezweifelt, dass der CO2 Gehalt der Atmosphaere seit mehreren Jahrzehnten steigt.
    Wenn ich das selbe Verhaeltnis wie oben erwaehnt zu Grund lege, wie kann es dann sein, dass die Temperatur seit 10 Jahren gleich bleibt bzw. faellt?

    Danke im Voraus.

    E.

  92. Lieber Herr Leistenschneider,

    wenn ich Sie richtig verstanden habe, so begründen Sie die „Gültigkeit als Gegenbeweis“ Ihres Vergleichs der Temperaturdaten aus einer lokalen Tropfsteinhöhle und auch die Steigungen bei DO- oder dem 8.3k event mit entsprechenden global-gemittelten Daten, auf sich die Klimaexperten im Focus beziehen, damit,
    dass der IPCC die lokalen CO2-Messungen in der Antarktis auch einfach als globales Mittel betrachtet und somit analog wie Sie vorginge?

    Also quasi nach dem Motto „Wie du mir, so ich dir“? Habe ich das richtig verstanden?

  93. Wissenschaft ist ja keine Disziplin der Mehrheit oder des Glaubens. Die seit zehn Jahren stagnierende bzw. rückläufige Erderwärmung trotz höherer CO2-Werte und die folgende Endwicklung liefert und wird noch Fakten liefern, die keine Statistik langfristig verschweigen kann. Wenn in den USA die seit mehr als 20 Jahren unter den Tisch gefallenen Roh-Daten der regierungsnahen Institute GISS und DCDC aufgearbeitet werden, wird man noch deutlicher erkennen, wessen traurig Kind die letzten IPCC-Berichte sind, denn diese Daten wurden dort eingearbeitet. Die Aufdeckung dieses Skandals besorgen in diesen Tagen die Amerikaner und sie werden mal wieder für uns die Schmutzarbeit machen, während wir weiter belanglos in Kommentaren so hervorragender Dokumentationen, wie hier von EIKE vorgelegt, herumstochern und Glaubenskriege führen. Das haben wir schon als Kinder gelernt, die Wahrheit wird ans Licht kommen. Verlierer wird leider der Wissenschaftsstandort Deutschland sein.

  94. Zitat von Herrn Rahmstorf im Anhang des Romanes von M. Crichton „Welt in Angst“(Umwelt-Krimi der ZEIT-WISSEN-Redaktion 2009):“In der Wissenschaft ist nichts endgültig, es gibt immer die Möglichkeit,dass scheinbar gesichertes sich als falsch erweist.“
    Sehr interessant ist dann seine Bemerkung s.o. bei These 7 : (eine Zustimmung aller Regierungen zur Zusammenfassung des IPCC-Berichtes war) „…deshalb mögich, weil an den enthaltenen wissenschaftlichen Ergebnissen beim besten Willen nicht zu rütteln ist.“ Das sagt doch alles !!!!!
    Übrigens – als Anahng zum gen. Roman findet sich ein recht interessanter Aufsatz des Autors zum Thema „Warum politisierte Wissenschaft gefährlich ist“. Er bezieht sich u.a. auf die Eugenik, auf deren theoretischer
    Grundlage Politiker (nicht nur Hitler) Rassenhetze mit all ihren Ausuferungen betrieben. Und da soll man heute nicht an eine gewollte Errichtung einer Welt-Ökodiktatur denken? Was ist denn die von Herrn Schellnhuber
    proklamierte „GROSSE TRANsFORMATION“?

  95. Jawoll, lieber Tobias.

    Und für alle, die es nicht glauben:
    Hitler war der erste ÖKODIKTATOR und außerdem Komplize von Michael Mann.

    Beweis auf Youtube gefunden:
    http://tinyurl.com/ykzx4jj

    (Unbestätigten Quellen zufolge war sogar ein Tempolimit auf der Autobahn geplant, die Alliierten konnten dies aber im letzten Moment abwenden.)

  96. @ Tobias
    so sehe ich das politisch auch!

    @ Marvin Müller, Wissender, NB
    danke für den Link, werde mir die Arbeit zu Gemüte führen. Was mich aber noch vor Studium derselben schon lange wundert, ist Folgendes: Wo immer man Naturwissenschaft betreibt, versucht man möglichst viele Experimente, Daten usw. zu sammeln. Warum aber wurde die Zahl der meteorologischen Mess-Stationen drastisch reduziert. Nun ich vermute, es waren qualitative Gründe. Ob berechtigt oder nicht, sei einmal dahingestellt.
    Warum aber hat man in den vergangenen 20 Jahren offensichtlich keinerlei Interesse, die Datenbasis drastisch zu vergrößern. Will man über den Klimawandel etwas wissen, müsste man dochj größtes Interesse haben, wesentlich mehr Temperatur- u.a. Daten zu bekommen. Bei dem vielen Geld, das ausgegeben wird, wären das doch Peanuts. Das, sorry, verstehe ich nicht, und das ist für mich auch völlig unseriös. Gerade so, als wöllte man es gar nicht so genau wissen. Haben Sie mir eine Antwort darauf?

  97. #38: Wissender sagt:
    am Mittwoch, 03.02.2010, 16:03

    Warum werden eigentlich die E-Mail adressen angezeigt. Finde ich nicht so prickelnd.

    Kommentar:
    Die Email wird abgefragt für die Bestätigungsmail. Diese dient nur zu Ihrer Information.
    Wie Sie ja sehen, werden die Adressen auf der Seite nicht angezeigt.
    Grüße
    Admin

    Es gab heute Nachmittag einen kurzen Zeitraum, in dem die Artikel in einem anderen Format angezeigt wurden – inklusive Email. War lustig zu sehen, welche einfache Erklärung es für einige Kürzel hier gibt…

  98. Hallo Herr Eber,

    „Wird reiner Kohlenstoff verbrannt (Stein- und Braunkohle), müsste die O2-Konzentration im selben Maß fallen, wie die CO2-Konzentration zunimmt.“

    Im Prinzip ja, aber dazu ein Zahlenbeispiel:
    20%-100ppm sind dann mit 19,9999% praktisch dasselbe. 100ppm Zunahme ist bei einem Spurengas wie CO2 eine deutliche relative Zunahme, bei O2 aber vernachlässigbar.

  99. Lieber Herr Boos,

    in #28

    danke für die Beantwortung meiner Frage.

    Zu:
    „“Das Kohlendioxid in der Atmosphäre stamme hauptsächlich aus den Ozeanen, argumentieren die Leugner.“
    Merkwürdig, unsere Verfechter der reinen und unbefleckten Wissenschaft stossen sich an dieser Unterstellung nicht (zumindest liest man hier nichts).“

    Gucken Sie mal unter http://tinyurl.com/ykpdwmw
    Ich hatte da schon erklärt, warum der CO2-Ansteig der letzten Jahrzehnte hauptsächlich durch industrielle Aktivitäten bedingt ist. Aber bei EIKE und seinen Lesern herrscht halt die Meinung vor, man wüßte es besser und beharrt standhaft auf der Vodoo-Physik der Ozeanausgasung, obwohl jeder in Chemie fitte Oberstufenschüler mit Hilfe des Henryschen Gesetzes beweisen kann, dass dies die Beobachtung nicht erkärt und außerdem schon in den 50iger Jahren mit Isotopenmessungen geklärt wurde, woher das CO2 stammt. Aber für EIKE wird halt die Wissenschaft gerne zugunsten der Stimmungsmache geopfert.

  100. Sehr geehrter Herr NF (#4),

    „Schneider, bleib bei deinen Leisten“

    Das Leben ist wie Fahrradfahren, man muss sich ständig vorwärts bewegen, wenn man das Gleichgewicht nicht verlieren will. Albert Einstein

    Viele Grüße RL

  101. Ich möchte mich noch Jürgen Wanninger anschließen. Bei mir war es noch das Wasser. Man haben die giftig reagiert, wenn man um die 90er einfach nach der Wirkung des H2Os nachgefragt hat. Wie kommt man als Erdbewohner eigentlich auch drauf, nach H2O und Sonne nachzufragen. Eine vernünftige Antwort war schwer zu finden bis sie es als Rückkopplungseffekt entdeckt haben.
    Was mir noch einfällt, dieses typische Verlautbarung der Interviewten, dass praktisch der natürliche Einfluss seit der Erfindung der Dampfmaschine auf null steht. Auch so ein Ding.

  102. Warum werden eigentlich die E-Mail adressen angezeigt. Finde ich nicht so prickelnd.

    Raimund Leistenschneider: Sehr dürftige Antwort.
    Punkt 1: Können Sie bitte ausführen, was „Climategate“ mit der Auswertung der Daten von GISS/HadCRU zu tun hat? Das schöne Bildchen in Ihrem Link zeigt eine Temperaturrekonstruktion anhand der Dichte von Baumringen. Und weshabl zeigen Satellitenmessungen und Messungen von Bodenstationen nahezu die gleiche Erwärmungsrate? Hat Christy die Daten auch gefälscht?

    Punkt 2: Kann man als Ingenieur nur eine Variable betrachten? Wie ich schrieb ist der Effekt durch den höheren Partialdruck vom CO2 in der Luft der wichtigere Faktor. Soll ich es Ihnen noch vorrechnen?

    Punkt 3: Ja und sie bestätigen, dass der Anstieg der CO2 Konzentration nicht durch Vulkane verursacht wurde.

    Punkt 4: Ja, Bilderchen das haben Sie gerne. Ob die Daten, auf Basis derer die Bildchen erstellt worden, korrekt sind oder nicht, dass interessiert Ingeniuere wohl nicht zu sehr.
    Und bitte was hat der Wasserdampfgehalt der Stratosphäre mit dem Wasserdampffeedback zu tun? Lesen Sie die Paper eigentlich auch, auf die Sie verweisen?

    Punkt 5: Genauso aussagelos wir Ihr „Argument“. Dass sie Sonne einen Effekt hat, ist klar. Die Frae ist nur wie gross und darauf liefern Sie keine Antwort.

    Punkt 6: Stimmt, ist alles gefälscht. Wie konnte ich das nur vergessen?? PS: Laut UAH ist der Januar der wärmste seit Messbeginn. Ist bestimmt auch gefälscht.

    Punkt 7 : Genau, so doofe Literatur mit peer review, das sieht man bei EIKE nicht gern; Lieber die tollen Berichte von Watts. Hier ein Link zur Studie: http://tinyurl.com/ydhrbhb

    Punkt 8: Oh Sieh haben etwas von der NASA oder Science, das diese Aussage stützt: W“ Der atmosphärische CO2-Gehalt war in der Atmosphäre nie konstant, sondern schwankte ganz erheblich. Anhand der gigantischen natürlichen CO2-Mengen, die allein durch eine Erwärmung freigesetzt werden und den CO2-Pegel triggern (Abbildung unter Punkt 2), gehen die wenigen GT anthropogenem CO2 im Hintergrundrauschen natürlicher Fluktuationen unter. „ Bitte her damit!!

    Punkt 9: Veizer ist auch noch Biologe geworden?

    Punkt 10: Eine Höhle, das ist Weltklima!

  103. #32: Jürgen Wanninger

    Ja, die Parallelen in jene Zeit sind leider unverkennbar. Im Nürnberger Prozess waren die Hauptanklagepunkte häfig mit Verschwörung(-zu …)usw. versehen. Es gibt den Vorwurf(juristisch) der Verschwörung tatsächlich und nicht nur den der Verbreitung irgendwelcher Verschwörungstheorien, heute meist im Umgang kritisch denkender Zeitgenossen gebräuchlich.
    Um jedoch nicht mit V-theoretikern in einen Topf geworfen zu werden, halte ich mich an eine real, stattgefundene Verschwörung, praktisch bewiesen. Dies ist der Reichstagsbrand, in dessen Anschluss das Hitlerregime die Persönlichkeitsrechte vollständig außer Kraft setzte, um mit jeglicher, aktiver Opposition aufzuräumen zu können und den noch zweifelnden Teil der Bevölkerung vom eigenen, „gewisse Opfer fordernden Tun“ zu überzeugen. Leider fügt sich in dieses Bild heute Einiges von damals ein. Heute wie damals scheint das erste Ziel die Errichtung einer Diktatur(heute Öko-)zu sein.

  104. #9, #24 Wissender,

    in #33 hatte ich bereits nach der genannten Studie gefragt. Leider hatte ich Ihren ‚Beitrag‘ #24 übersehen. Damit haben Sie sich für mich ausreichend disqualifiziert.

  105. Besten Dank den Herren vom EIKE für diese tolle Arbeit, in der so schön übersichtlich und gut verständlich das Lügengebäude des IPCC zerpflückt (besser: gesprengt) wird und ihm die Fakten gegenüber gestellt werden …

    Beim Lesen ist mir spontan ein Gedanke gekommen – die Experten mögen nachsichtig mit mir Laien sein:
    Laut Aussagen der Klima- und CO2-Hysteriker ist es allein der Mensch, der den Kohlenstoff-Haushalt der Atmosphäre durcheinanderbringt, weil er ständig große Mengen CO2 durch Verbrennen fossiler Stoffe in die Luft bläst. Der natürliche, weit überwiegende CO2-Anteil (97 %) würde in einem geschlossenen Kreislauf umgesetzt.

    Wenn dem so wäre, dann müsste durch die menschlichen Aktivitäten zwangsläufig die O2-Konzentration fallen – wenn auch nur geringfügig, aber eben doch messbar.

    Wird reiner Kohlenstoff verbrannt (Stein- und Braunkohle), müsste die O2-Konzentration im selben Maß fallen, wie die CO2-Konzentration zunimmt.
    Werden dagegen Kohlenwasserstoffe (Erdgas, Erdöl) verbrannt, müsste die O2-Konzentration überproportional sinken, insbesondere bei der Verbrennung längerkettiger CWe, weil ja nicht nur der Kohlenstoff, sondern auch der Wasserstoff verbrannt wird.

    Es wäre doch einmal interessant zu erfahren, ob ein solcher Zusammenhang schon einmal untersucht wurde – ich konnte leider nirgendwo Hinweise darauf finden.

  106. hallo NB,
    warum antwortest(reagierst,denn antworten tust du nie), du auf fragen die an NF gestellt werden.
    Also bist du auch NF.
    Was soll dein Unfug.
    Du hast verloren,such dir eine andere Sportart.
    Du bist ein wissentschaftlicher Amateur der vom PIK abschreibt.
    Ich habe in der Meteorolgie oder in der Physik noch nie was von einem NB/NF gehört.

    Also gehe davon aus dass dein Praktikum bei Schellnhuber/Rahmsdorf auslaüft.

    Kannst dich ja auf eine neue Eiszeit spezialisieren.

    Gruss
    U Meinen

    PS: zum wiederholten mal , warum bist du so ein
    anonymer Feigling;
    Kostet dich Offenheit deinen JOB?

  107. #31: windkanter dankt:

    „Danke an alle, die sich die Mühe machen, den Unfug, der hier veröffentlicht wird, ein wenig zu korrigieren“

    Dann korrigier‘ mal schön den Unfug, windkanter: :-)))

    „In the vast majority of stations we did not see indications for a global warming of the atmosphere.“
    H.J. Schellnhuber, Physical Review E68, 046133 (2003)

    Seit 2003 ist’s also zur globalen CRU-Hitzewelle gekommen, wa? :-)) Geh‘ weiter, Nebelkerzenwerfer. Dein CO2-Götze ist tot, und nur arktische Temperaturen bewahren uns vor’m Verwesungsgestank.

    „27,6 Grad: Gars war der rot-weiß-rote Kältepol!“
    NÖN Online – ?Vor 7 Stunden

    Kälterekorde in der Region – Weitere Schneefälle erwartet – Berliner Kurier

    Bislang kälteste Nacht dieses Winters in Berlin – Berliner Kurier

    Schnee im Anmarsch NRW mit Kälterekord
    – Kölnische Rundschau
    (… usw.)

  108. an #9, Wissender,

    können Sie auch einen Link zu der dort unter Punkt 7 genannten Studie geben?

    Ich kenne Sie nicht und so ist dieser Beitrag für mich und vermutlich andere Leser wertlos.

  109. #22 Tobias

    vollste Zustimung. Mir ging das ähnlich.

    Wurde erstmals skeptisch, als ich im Zuge von AR4 eine Stellungnahme gelesen habe, wonach der solare Einfluss aufs Klima belanglos sei, da die Sonne hinreichend gleichmäßig strahle.

    Da gingen bei mir alle Alarmlampen an. Ein hochkomplexes System mit chaotischer Zustandsfunktion wie unser Klima wird von außen periodisch angeregt und soll darauf nicht mit chaotischem und unvorhersagbarem Verhalten reagieren ??

    Das widerspricht allem, was ich während meines Physikstudiums gelernt habe.

    Was ich außerdem als höchst unenagenehm und Zeichen von geistiger Midnnrbemittelung empfinde, ist, dass im Rahmen der Klimadiskussion ein beißender Gestank von Intoleranz gegen Andersdenke – ja geradezu Denkverbote weht, wie er an die Zeit von 1933 bis 1945 in diesem Land erinnert.

    Denkverbote und Gegen-Demonstrationen z.B. von Greenpeace zur Berliner Konferenz erinnern erschreckend an die Anfänge vom Faschismus. Auch Adolf war von seiner Sache ganz überzeugt und glaubte für eine höhere Wahrheit zu handeln.

  110. Wenn Sie bei Ihren Texten den Maßstab ansetzen würden, den Sie beim IPCC anlegen, dann ist Ich kann mich nur der Meinung von KlausV #10 anschließen: „dieser Artikel mit Bomben und Granaten durchgefallen. Womöglich will uns EIKE genauso arglistig täuschen, so wie Sie das vom IPCC annehmen?“

    Danke an alle, die sich die Mühe machen, den Unfug, der hier veröffentlicht wird, ein wenig zu korrigieren

  111. @K.Boos

    „da ich kein Klimawissenschaftler bin …“

    Aber KLIMAEXPERTE dürfen wir uns zu Recht alle nennen. Dazu verfüge ich über ZWEI Thermometer, eines gar mit °C und °F-Skala, sowie ein Hygrometer. Fotodesigner sind wir schließlich auch alle, so wir über eine funktionsfähige Fotokamera verfügen.

    „Diese Annahme der „Leugnung“ kommt aus dem gedanklichen Arsenal der Inquisition, im Grunde ist es eine „religiöse Annahme“ – wie jede unwissenschaftliche Annahme kann diese Behauptung durch nichts falsifiziert werden, sie beruht auf einem Zirkelschluss primitivsten Zuschnittes.“

    Harte Worte. 😉
    Unvergeßlich: Rotary-Rahnstorf forderte eine „Qualitätskontrolle“ des berüchtigten „Qualitätsjournalismus“, eine Qualitäts-Qualitätskontrolle sozusagen. Rahnstorf-VerehrerIn hat die Zeichen der Zeit begriffen.
    taz-Blog-Beitrag:
    _________

    Zum Artikel

    “Klimahysterie” – das Versagen der Medien

    Nimm dies, SPIEGEL! Erfreulich klare Worte vom Potsdam-Institut für Klimafolgenforschung
    (gekürzt auch in der FAZ erschienen, nur Print, 31.8.2007)

    (taz.de/blogs/reptilienfonds/2007/09/02/
    klimahysterie-das-versagen-der-medien)

    » Wir sind sehr dankbar für die klaren Worte, und betrauern ebenfalls das bedrückende Versagen der Qualitätskontrolle unserer Medien. Machen wir uns nichts vor: Die Pressefreiheit hat versagt.

    Der Herr Bütikofer im SPIEGEL, ich zitiere ihn: ” Wir haben nur zehn bis 15 Jahre Zeit – und keine zweite Chance. Es ist zu verhindern, dass in 100 Jahren die Sahara bis Berlin reicht oder London, Hamburg und der Großteil von Holland vom Meer bedeckt werden. Wenn nicht ambitionierter Klimaschutz betrieben wird, dann kommt der Horror!!”

    Angesichts der Katastrophen, die uns lt. Herrn Bütikofer in spätestens 15 Jahren drohen, ist der Ausdruck “Klimaskeptiker” noch viel zu milde. Vielleicht wäre “Klimahetzer” angemessener für diese sinistren Figuren, die uns mit ihren Zweifeln unweigerlich in den Klimahorror stürzen. Nochmals Herr Bütikofer:

    “Die Deutschen sind die größten Klimaschweine!”

    Wobei ich nicht verstehe: “Wir Deutschen” oder “die Deutschen”. Ist Herr Bütikofer kein Deutscher mehr, oder ist er kein Klimaschwein? Sein 5er BMW hat Diesel, vielleicht liegt’s daran, und Herr Gabriel wird schon seine Gründe haben, wenn er wegen des Klimahorrors alleine im Flieger sitzen muss. Was macht Knut eigentlich?

    Nochmals zum Versagen der Presse-Qualitätskontrolle, und der garstigen Zweifel an der von uns Deutschen Klimaschweinen verursachten Weltkatastrophe.

    Man sollte die ZweiflerInen einfach “Klimanazis” nennen, das geschähe ihnen recht, und das Holocaustleugner-Gesetz ein wenig erweitern, so daß auch diese Klimanazis ihre gerechte Strafe bekommen.

    Wenn jeder Hobbyjournalist und Blogger schreiben kann, was er will, muß es ja soweit kommen.

    google: Klimalüge = 270.000 Hits, und es steigt täglich! So darf es nicht weitergehen!!!

    Hier ist der Staat gefragt, oder vielleicht besser die EU. Ein Europaweites Klimaleugnungsgesetz muß her, und dem Herrn Rahnstorf sollte man die Kontrolle über die Veröffentlichungen überlassen, oder noch besser, es dürfen nur noch seine Ergebnisse gedruckt und gezeigt werden.

    In befreundeten Ländern wie Iran oder Saudi-Arabien funktioniert die Sitten- bzw. Religionspolizei sehr gut, und die Menschen sind glücklich. Nehmen wir uns ein Vorbild, und schaffen wir eine Klimapolizei, selbstverständlich EU-Weit, vielleicht eine Behörde, die das Klimagesetz überwacht. Klimagesetzbehörde, KGB, das klingt vertraut und vertrauenerweckend.

    Die Frau Benito Ferrero-Waldner hat auf der Eurabia-Konferenz im letzten Jahr eine sehr schöne Anregung formuliert, wie man die Medien im zukünftigen Eurabia / Euromed-Land harmonisieren könnte, damit sowas nie wieder passiert:

    “Die Redefreiheit ist zentral für Europas Werte und Traditionen.
    Jedoch, ihre Aufrechterhaltung hängt von dem verantwortlichen Benehmen von Einzelpersonen ab. Weiterhin glauben wir nicht, dass die Medien von außen gelenkt werden sollen sondern vielmehr, dass Sie selbst Wege finden, um sich selbst zu zensieren. Was die Selbstzensur anbelangt, möchte ich Sie auch bitten das Bedürfnis der Überwachung innerhalb Ihrer eigenen professionellen Reihen zu bedenken.”
    (Benita Ferrero Waldner, EU-Kommissarin für Außenrelationen und die Europäische Nachbarschaftspolitik an ausgewählte Medienvertreter aus allen euromediterranischen Staaten am 22.05.2006)
    Benita Ferrero-Waldner – Intercultural dialogue: the media’s role Speech/06/321

    http://tinyurl.com/yel2nyj

  112. @#27 Günther Kosmann

    Korrekter Beitrag.

    “ … die Akzeptanz der globalen Erwärmung als Folge der CO2-Emissionen von vielen Medien, Teilen der Politik und dem Großteil Bevölkerung sogar in Deutschland gegenüber den PIKanten Hysterikern signifikant nachlässt, macht die Klimablockwarte aus Potsdam und deren klammheimliche Blockflöten(NB,NF und Kumpanen) so wütend.“

    Und Merkel verkündete unlängst, daß „Artenschutz“ vielleicht doch das bessere Vehikel sei, um von realen Problemen abzulenken. Man spürt förmlich, wie der „Elite“ das Gesäß aufs Gletschergrundeis geht. 🙂

    Aufschlußreich: Leserkommentare zu

    „Artikel aus Bergsteiger-Magazin dient Klima-Bericht
    1. Februar 2010
    Der Weltklimarat rutscht aufgrund falscher Daten zum Abschmelzen der Himalaya-Gletscher immer tiefer in eine Glaubwürdigkeitskrise. Der „Sunday Telegraph“ nennt nun weitere abstruse Quellen des UN-Gremiums: Offensichtlich wurde auch aus einer studentischen Arbeit sowie aus einem Bergsteiger-Magazin zitiert. … “

    http://tinyurl.com/y8awzqz
    (WELT.de)

    WELT-Leser informiert:

    „Sehr guter Kommentar – langsam oeffnet sich das Fass der wirklichen Information und wird die Politiker, die immer behaupten CO2 waere das Problem, hinwegschwemmen.
    Es ist wirklich unglaublich, wie oeffentlich rechtliche Medien plus das Zeitungswesen in den Klauen unserer Politiker/ Eliten haengen…uebrigens auch hier in England…
    Ich moechte an dieser Stelle der WELT ausdruecklich fuer diese freien Blogs danken!
    Ich bin mal gespannt wer das Fass zuerst sprengt – der Daily Telegraph mit Lord Lawsen ist definitiv ein heisser Anwaerter…auch das Wall Street Journal ist nicht mehr weit davon entfernt…“

    Und ich erlaube mir, an dieser Stelle dem E.I.K.E.-Team für Engagement und Aufklärung zu danken, im Geiste des berühmten Klimaleugners und Obrigkeits-Skeptikers.

    „Wenn kein Mensch mehr die Wahrheit suchen und verbreiten wird,
    dann verkommt alles Bestehende auf der Erde,
    denn nur in der Wahrheit sind Gerechtigkeit, Frieden und Leben!“

    Friedrich von Schiller (1759 -1805)

  113. noch ein Zusatz und von meiner Seite abschließend:
    Unter Punkt 2 des oben zitierten focus-online Artikel finden wir folgenden Satz:
    „Das Kohlendioxid in der Atmosphäre stamme hauptsächlich aus den Ozeanen, argumentieren die Leugner.“
    Merkwürdig, unsere Verfechter der reinen und unbefleckten Wissenschaft stossen sich an dieser Unterstellung nicht (zumindest liest man hier nichts).
    Wer in einem wissenschaftlichen Diskurs seinem Gegenüber unterstellt ein „Leugner“ zu sein macht folgendes:
    1. Er unterstellt seinem Gegenüber er wisse in Wirklichkeit dass er Unrecht habe und nur aus irgendeiner Bösartigkeit heraus die Unwahrheit gegen besseres Wissen behaupte.
    2. Das ist nun wirklich eine „Verschwörungstheorie“, sie setzt ja voraus dass derjenige der diese Behauptung aufstellt irgendwoher weiß, was im Gehirn der „Leugners“ vorgeht – eine Annahme derart totalitären Zuschnittes, dass es einem schwindlig werden könnte.
    3. Diese Annahme der „Leugnung“ kommt aus dem gedanklichen Arsenal der Inquisition, im Grunde ist es eine „religiöse Annahme“ – wie jede unwissenschaftliche Annahme kann diese Behauptung durch nichts falsifiziert werden, sie beruht auf einem Zirkelschluss primitivsten Zuschnittes. Was immer der als „Leugner“ bezeichnete auch vorbringen mag – da er ein „Leugner“ ist, also WILLENTLICH Falsches behauptet, kann er selber nichts zu seiner Entlastung beitragen.
    4. Diese Annahme hat einen „angenehmen“ Nebeneffekt – eine weitere Debatte erübrigt sich im Grunde, da es nicht mehr um einen Vernunftdiskurs geht, auf den sich beide Parteien beziehen, sondern darum dass die eine Seite die „Wahrheit hat“ – die andere sie willentlich „leugnet“.

    Unsere kritischen Beobachter hier stossen sich an dem vergleichsweise milden Spott und der Polemik der EIKE-Autoren, derartige aus dem Arsenal des Totalitarismus stammende Unterstellungen aus dem „Fakten, Fakten,Fakten-Magazin“ stossen ihnen scheinbar nicht auf.

    Ich für meinen Teil finde das merkwürdig.
    Zu ihrer Frage: Nein, da ich kein Klimawissenschaftler bin, war oder bin ich davon nicht betroffen.
    Aber andere Wissenschaftler sind das sicher, geht ja auch aus den Mails die im Rahmen von „Climategate“ bekannt geworden sind hervor.
    Zu der Frage wie ICH, als „Nichtfachmann“ da zu einer „Einschätzung“(und welcher genau) kommen kann, vielleicht später, an anderer Stelle – das ist eine schwierige Frage, die eine umfangreichere Antwort erfordert.
    Aber es ist klar – Anspruch auf „absolute“ Richtigkeit stelle ich nicht.
    Aber der Focus-Artikel hat sich in meinen Augen durch diesen „Leugnungsvorwurf“ als gänzlich unwissenschaftlich und als in einem Zirkelschluß befindlich erwiesen. Und die beteiligten Wissenschaftler äußern da auch keine Bedenken.
    Was mir wieder bedenklich erscheint.

  114. Sehr geehrter Herr Klaus V, (#10)

    Punkt 1
    die von Ihnen angeführte Grafik (Abbildung 1) stand nicht in Spektrum der Wissenschaft, sondern ist von Prof. Rahmstorf selbst. Unter (http://www.pik-potsdam.de/~boris/statcrew/suenje_12.01.06.pdf) sind vom PIK-Potsdam die DO-Ereignisse auf kleineren Zeitskalen aufgelistet (Folie 7). Dort erkennen Sie, dass der Anstieg, also der Temperatursprung von mehreren (!) °C zu höheren Temperaturen in Zeitspannen und wenigen 10 Jahren abläuft und damit die Aussage von Herrn Prof. Marotzke widerlegt ist. Auch Temperatursprünge zu tieferen Temperaturen um bis zu 4°C innerhalb von weniger als 50 Jahren sind dort auf der Folie abgebildet. Das Zitat aus S.d.W. zu den Temperaturschwankungen (Klimawandel) in der Zeit, in der die DO-Ereignisse abliefen, schilderte die Lebensbedingungen vor dem Verschwinden des Neandertalers, also vor ca. 30.000 Jahren. Es diente zur Unterstreichung, mit welcher Geschwindigkeit in der Vergangenheit Klimawandel abliefen. Zum Beleg dienten die Abbildung 1 und für die jüngere Zeit, die Abbildung 2. Anhand des obigen Links, können Sie sich von den schnellen, starken Temperaturanstiegen überzeugen, die Prof. Marotzkes Aussage eindrucksvoll widerlegen.

    „Punkt 10: Sie vergleichen ernsthaft den Temperaturanstieg in EINER Höhle in Tirol mit dem globalen Temperaturanstieg?“

    Dann müssten Sie auch das IPCC fragen, sie vergleichen wirklich den CO2-Gehalt aus Eisbohrkernen der Arktis/Antarktis mit dem Klima auf der Erde, bzw. ermitteln daraus eine vorindustrielle globale CO2-Konzentration. Eine rekonstruierte Temperaturreihe fußt auf geeigneten Proxys aus einer ganz bestimmten Gegend. Zum Abgleich dieses Ergebnisses werden dann weitere Proxys herangezogen, so wie dies im Text geschildert ist: „Mittels weiterer Proxys (13C, Strontium, Barium, Yttrium, Blei, Schwefel, sowie ins Meer gespülte Sedimente) hat das Forscherteam seine Temperaturreihen abgesichert, so dass aus Sicht von Mangini et al. die Temperaturwerte (blaue Kurve) genauer, als mit Vergleichsmethoden (rote Kurve) angegeben werden konnte“ Dort steht beispielsweise „ins Meer gespülter Sedimente“, so dass Sie selbst zu dem Schluss hätten kommen können, dass diese sicherlich nicht in eine Höhle gespült wurden.

    Viele Grüße RL

  115. Wir erleben zur Zeit den kältesten Winter weltweit mit sehr bemerkenswerten Temperaturen und das trotz steigendem CO2-Gehalt.
    Diese Tatsache und das die Super-Klima-Computermodelle, die Weltuntergangsprognosen und Scheckensszenarien,die ideologische Gleichschaltung unserer jungen Menschen,die sündhaft teuen und verantwortunglosen
    Klimagipfelzirkusveranstaltungen nur etwas für Profilneurotiker oder Profiteure wie Al Gore, R. Pachauri, den Hedgefonds, für die Boni-Banker und Vertreter von Homo sapiens politicus korruptus sind, dass der Heilige Klimatempel des IPCC vom Einsturz bedroht ist und die Akzeptanz der globalen Erwärmung als Folge der CO2-Emissionen von vielen Medien, Teilen der Politik und dem Großteil Bevölkerung sogar in Deutschland gegenüber den
    PIKanten Hysterikern signifikant nachlässt, macht die Klimablockwarte aus Potsdam
    und deren klammheimliche Blockflöten(NB,NF und Kumpanen) so wütend.
    Sie können einem ja auch fast Leid tun, war doch alles so schön geplant, mit der CO2-Abgabe zunächst die böse , umweltzerstörende ,kapitalistische Wirtschaft zu eliminieren, um dann einen neuen ökosozialistischen, supranationalen Menschen zu schaffen unter Indoktrination der GRÜNEN KHMER.
    Wissenschaft hat hierbei eine systemzerstörende Funktion einzunehmen, weil es um DAS mit aller -Macht zu erreichende Welt-Ziel geht:
    Frei nach Schellnhuber: „DIE GRÖSSTE TRANSFORMATION seit der Jungsteinzeit.“
    Schade ist nur, dass die Natur wieder einmal nicht mitspielt.
    Aber Jungs bleibt Euch treu: Eure Chance kommt spätestens im Jahre 2350 oder 5230 oder 3520 , wenn die Gletscher am K2 schmelzen und ihr die Hauptrolle in dem von Al Gore gesponsorten Film spielen dürft: „Mit dem Schneebesen zum Matterhorn“- wieder einmal eine unbequeme Wahrheit-

  116. Lieber Herr Boos,

    ich wiederhole meine Frage nochmal, ob Sie auch Opfer Ihrer Punkte „alternative Veröffentlichungen verhindert, sämtliche Forschungsmittel monopolisiert“ wurden?

    „Die „Schlammschlachten“ begründen sich allerdings aus den gesellschaftlichen Implikationen und nicht aus der „Wissenschaft“ selber.“

    Ich stimme Ihnen zu, dass sich die Schlanmmschlachten aus den gesellschaftlichen Implikationen begründen und nicht aus der Wissenschaft selber.

    Mein Kritikpunkt daran ist, dass bei diesen gesellschaftlich motivierten Schlammschlachten wissenschaftlich gewonnene Ergebnisse relativiert, infrage gestellt oder abgelehnt werden, ohne dass innerhalb der Wissenschaft dafür einen Grund zu finden ist! Wissenschaftliche Ergebnisse sind schließlich unabhängig von den gesellschaftlichen Implikationen gültig oder nicht gültig!

    „Aber wenn sie tatsächlich so ein Kämpfer für die reine und hehre Wissenschaft sind, dann hätte ich ihre mahnende Stimme ja schon früher hören sollen.“

    Die Dinge, die ich beschreibe, sind ja nicht neu, sondern seit Jahren in der wissenschaftlichen Literatur zu finden. Neu ist nur, dass wegen der gesellschaftlichen Implikationen immer mehr Menschen Interesse an diesen wissenschaftlichen Ergebnissen haben. Ohne Grundwissen ist es jedoch kaum möglich, diese Ergebnisse mit den gesellschaftlichen Implikationen zu verknüpfen. Stattdessen werden wissenschaftliche Erkenntnisse in die politische Diskussion einbezogen und aus der individuell-politischen Sichtweise statt der angemessenen wissenschaftlichen „neu bewertet“. Wenn es bei diesem Blödsinn bliebe, ginge es ja noch.

    Aber die Wissenschaft wird darüberhinaus missbraucht, denn statt den Mangel an Grundwissen zu erkennen, zu akzeptieren und zu beheben, entwickeln sich „außerwissenschaftliche Interpretationsschemen“ der wissenschaftlichen Ergebnisse, die sich dadurch auszeichnen, dass das „Interpretationsschema“ von der individuell präferierten Schlußfolgerung abhängt. Die Interpration wissenschaftlicher Literatur dient also nur als persönliche pseudoobjektive Rechtfertigung a posteriori.
    Nehmen Sie z.B. diesen EIKE-Artikel. Sind Sie nun in der Lage, zu entscheiden, wer, Fokus/PIK oder EIKE, nun die wissenschaftlichen Erkenntnisse korrekt wiedergibt?

  117. #9

    Punkt 1
    Antwort: Climategate und die nicht abgeschnittene Temperaturreihe (http://www.extremnews.com/nachrichten/natur-und-umwelt/ff4212c4e13174e/332412c4e22abe4). Weitere Belege, für die Auswahl „geeigneter“ Messstationen zur Klimaerwärmung und das Eliminieren ungeeigneter Messstationen in „SURFACE TEMPERATURE RECORDS: POLICY DRIVEN DECEPTION?“ by Joseph D’Aleo and Anthony Watts.

    Punkt 2
    Antwort: Ozeane sind Netto-Senken bei gleichbleibenden oder fallenden Temperaturen, nicht bei steigenden Temperaturen, Erläuterungen, siehe Text.

    Punkt 3
    Antwort: Wir verwechseln gar nichts, siehe der angegebene Link und die Daten der genannten Institute und Wissenschaftler.

    Punkt 4
    Antwort: Bilder sagen mehr als tausend Worte, daher die Abbildung anschauen, wird übrigens von der IPCC-Koryphäe Susan Salomon bestätigt, dass der Wasserdampf nicht zugenommen, wie nach den IPCC-Modellen hätte eintreffen müssen – Wasserdampf-Feedback – sondern abgenommen hat. Es gibt dieses postulierte Feedback nicht.

    Punkt 5
    Antwort: Wie die übrigen gegen gehaltenen Punkte, aussagelos.

    Punkt 6
    Antwort: 2009 konnte das GISS, entgegen aller Kälte- und Schnee Rekorde in weiten Teilen der Welt (Saudi-Arabien, China, Korea, Kanada, USA, Europa, etc.) die 2. höchsten Temperaturen seit 130 Jahren vermelden, weil Datenmanipulation vorgenommen wurde, indem hunderte Messstationen, die fallende Temperaturen auswiesen, einfach aus der Statistik rausfielen. Siehe unter: „SURFACE TEMPERATURE RECORDS: POLICY DRIVEN DECEPTION?“ by Joseph D’Aleo and Anthony Watts.

    Punkt 7
    Antwort: Von fragwürdigen, bzw. tendenziösen Studien halten wir nicht viel. Auseinandersetzen werden wir uns selbstverständlich damit. i.Ü. gibt es eine Vielzahl weiterer Beispiele und Untersuchungen, die zeigen, dass es sich nicht um Einzelfälle, sondern um „Normalfälle“ handelt. (http://www.surfacestations.org/)

    Punkt 8
    Antwort: Wer die NASA und Science einfach mit Mutmaßungen gleich setzt, hat sich selbst geoutet!

    Punkt 9
    Antwort: Ihre Aussage ist grundlegend falsch. Siehe das Zitat von Prof. Veizer im Text und die Erläuterungen zum Punkt.

    Punkt 10
    Wie die übrigen gegen gehaltenen Punkte, aussagelos

    Viele Grüße RL

  118. Koecher, du hast so wenig Ahnung, dass eine Diskussion mit dir keinen Sinn macht. „Mike’s Trick“ ist für eine Rekonstruktion von Temperaturen anahnd der Dichte von Baumringen verwendet worden. Was hat das mit der Anlayse der Temperaturentwicklung von GISS/HadCRu zu tun?

    michaelsclimate: Danke für den Verweis, sehr schöner Blog.

    Marvin Müller: Ich wette schon mal einen Kasten Bier, dass du keine vernünftige Antwort von den „Klimaexperten“ bekommen wirst.

  119. #10: KlausV hetzt:

    „Womöglich will uns EIKE genauso arglistig täuschen, so wie Sie das vom IPCC annehmen?“

    Vom IPCC ist es BEWIESEN. Pachauri ist als Lügner und gigantischer Abzocker entlarvt.
    EIKE wird von der occulten Klimarealistenlobby gesponsort, wa? :-)))

    Was meint V.Klaus denn zur Erdüberhitzungs-Expertise hier?:

    „In the vast majority of stations we did not see indications for a global warming of the atmosphere.“
    H.J. Schellnhuber, Physical Review E68, 046133 (2003)

    IPCC-Falschdatenlieferant klärt auf:

    “I’ve just completed Mike’s Nature trick of adding in the real temps to each series for the last 20 years (ie from 1981 onwards) amd from 1961 for Keith’s to hide the decline.“
    – Phil Jones,
    Ex-Boss der CRU-Fälscherwerkstatt, Rohdatenvernichter.

    “ (…) Und jetzt kritisiert die NOAA die IPCC:
    „Die vorhergesagte Temperaturerhöhung durch die IPCC ist unrealistisch, nachdem die globalen Durchschnittstemperaturen und die der USA nur um 0,5 Grad in 100 Jahren gestiegen sind. Sogar wenn dieser Anstieg richtig ist und man kann es dem Menschen als Ursache zuschreiben, ist es ein trivialer Wert in der natürlichen Variation der Erde. Und zu behaupten, die Beschleunigung würde sich verfünffachen (laut IPCC) in diesem Jahrhundert ist unglaubwürdig.“

    Also, in hundert Jahren ist die globale Durchschnittstemperatur nur um 0,5 Grad gestiegen. Dieser minimale Anstieg in einem Jahrhundert ist nichts ungewöhnliches und liegt in der normalen Bandbreite der natürlichen Fluktuation. Und seit 10 Jahren gibt es gar keine Erwärmung!

    Eigentlich könnte ich hier aufhören, denn es ist alles was die Klimahysteriker sagen damit widerlegt und als falsche Behauptungen und Lügen enttarnt! Aber machen wir weiter. … “

    http://tinyurl.com/yg7shq3

  120. Wenn man halbwegs serios für sich selbst eine Analyse der „Erwärmungstheorie“, ihrer Vorhersagen und ihrer (noch)durchzusetzenden(!!) Schlußfolgerungen für uns alle erstellt, dann bleibt kein einziger „Pluspunkt“ übrig, hinter dem sich ein Beweis verstecken könnte.
    Das gilt jedenfalls für mich, der ich noch vor etwa 2 Jahren ein totaler Anhänger dieser Theorie war. Ich sagte damals noch zu meinem Arbeitskollegen: „Gott gebs, dass sie sich irren. Andernfall kann uns nur der sorfortige Emmissionsstop retten“

    Mein Gott! War ich naiv!

    Vielen Dank an EIKE und die anderen „Realisten“ hier!

  121. Punkt 1 referenziert eine Graphik von Lord Monckton, die im Original wohl in folgendem Dokument zu finden ist: http://tinyurl.com/create.php. Dort ist es ohne den Stempel Fälschung abgebildet. Kann mir mal jemand erklären, wer da warum den Stempel drauf gedrückt hat? Ich referenzier mal prohpylaktisch die von Herrn Leistenschneider gelobten UAH Messungen, die auch eine Erhöhung dokumentieren. Oder ist das hier ein Aufguss der Leistenschneiderschen Behauptung, die UAH Daten würden die HadCrut Daten wiederlegen? Dann möchte ich gleich nochmal prophylaktisch auf eine direkte Gegenüberstellung von UAH und HadCrut im selben Zeitraum mit gleicher Skalierung verweisen, die sehr ähnliche Kurvenverläufe zeigen: http://preview.tinyurl.com/ylatstr. Vielleicht gibts ja irgendwann nochmal eine Erklärung, wo da Herr Leistenschneider einen konträren Verlauf sieht…

  122. S.g. Herr NF
    Wenn sie nur die „wissenschaftliche Frage“ interessiert und nicht die Frage wie bestimmte Thesen in gesellschaftlichen Zusammenhängen funktionalisiert werden, dann will ich sie da nicht bekehren. Die „Schlammschlachten“ begründen sich allerdings aus den gesellschaftlichen Implikationen und nicht aus der „Wissenschaft“ selber.
    Aber wenn sie tatsächlich so ein Kämpfer für die reine und hehre Wissenschaft sind, dann hätte ich ihre mahnende Stimme ja schon früher hören sollen.
    Was soll ich mir denken wenn sie ihre Bedenken erst jetzt, wo die Luft etwas dünner wird, zu hören bekomme? Gibt es einen „wissenschaftlichen Grund“ warum faire Diskussion erst jetzt so notwendig wäre?
    Sie gestatten wenn ich da Zweifel habe.
    In meinen Augen ist das ein politischer Kampf, der genau darum geführt wird, was die menschliche Geschichte seit je antreibt: Um Ressourcen und ihre Verfügbarkeit. Die AGW-ler sind da bisher nicht sonderlich zimperlich gewesen, der Widerstand dagegen braucht da keine Ratschläge von seiten jener, die bisher diese Zimperlichkeit nicht ins Spiel gebracht haben.
    Wer sich politisch benützen lässt, muss damit rechnen dass in der Politik die Bandagen etwas härter sind.

  123. Lieber Herr Boos,

    in welchem Zusammenhang und welche Leute (Klimatologen, Politiker, „Öko-Lobbyisisten“,??) haben gegen Sie diese Vorwürfe erhoben?

    Ich fragte eigentlich danach, ob Sie auch Opfer Ihrer Punkte „alternative Veröffentlichungen verhindert, sämtliche Forschungsmittel monopolisiert“ wurden?

    „Nein, es handelt sich nicht um Rache, es handelt sich darum dass das dies eine gesellschaftspolitische Frage ist“

    Mich interessiert eigentlich nur die wissenschaftliche, und meine Fragen bezogen sich auf die Schlammschlachten auf dem Gebiet.

  124. zu #9 „Wissender“

    Punkt 1:

    sh. „Mike’s Trick“

    Punkt 2:

    „Anteil des anthropogenen CO2 Ausstosses der in der Atmosphäre bleibt … ist nach Knorr etwa konstant, ca. 50%. Das bedeutet natürlich nicht, dass sich das anthropogene CO2 nicht in der Atmosphäre akkumuliert, ganz im Gegenteil.“

    Kommt Knorr zur selben Erkenntnis?

    Weiterhin bleibt offen, ob die nach Latif kühleren Ozeanoberflächen nun wieder CO2 aufnehmen oder aber nicht – ein Fazit für den geneigten Laien wäre schön.

    Punkt 3:

    Hier sind „Wissender“ und EIKE d’accord. Nichts anderes steht im Artikel. (Ein erster Verdacht der selektiven Wahrnehmung).

    Punkt 4:

    Dass es Kontroversen zum Thema Wasserdampf-Rückkopplung gibt, dürfte bei der Untersuchung eines komplexen Systems zwangsläufig sein.

    Schön ist, dass wenigstens die Abhängigkeit des Wasserdampfs von der Temperatur erkannt wurde.

    In diesem Zusammenhang wird zum Punkt 2 die These zum Nachlauf des CO2 hinter den Temperaturen nicht kritisiert. (Der Verdacht der selektiven Wahrnehmung erhärtet sich.)

    Punkt 5:

    „wertloses Palaver“ – von Rahmstorf? Von Svensmark und Veizer? Vom „Wissenden“?

    (Es kommt der Verdacht des Finger-In-Die-Ohren-Steckens, Augen-Zuhaltens und laut „Lala“-Singens als Ersatz für Sachargumente auf.)

    Punkt 6:

    War 2009 nicht das wärmste Jahr seit 200.000 Jahren? Erkannt aufgrund der Anzahl von Mückenfossilien an einem See in Kanada? Erkannt durch die Medien, denn die Studie kommt auf keinen direkten Zusammenhang mit den rekonstruierten Temperaturen?

    Oder haben wir, lt. Latif, eine Stagnation mit Abkühlung von Ozeanflächen und lediglich mit Erwärmung in Gebieten mit urbanen Ballungsräumen?

    Welche Schlussfolgerung aufgrund welcher Daten ist gemeint?

    Mein Wohnzimmerthermometer, weil ich mir in den 2000ern einen Holzkamin installiert und auch fleißig benutzt habe?

    Hier wegen der kontroversen Veröffentlichungen der letzten Monate doch bitte etwas genauer darstellen, falls möglich.

    Punkt 7:

    Warum wird die Studie von Menne nicht dargestellt?

    Nachdem aus Russland und Amerika deutliche Kritik wegen in Richtung Erwärmung selektierter Datensätze laut wurde, wäre dem geneigten Leser eine Darstellung wenigstens hinsichtlich des Tenors wünschenswert.

    Punkt 8:

    Wenn der heutige CO2-Gehalt der Atmosphäre der höchste seit 15 Mio. Jahren ist, wie hoch ist denn dann der anthropogene Anteil seit Beginn der Industrialisierung bzw. wie viele Millionen Jahre muss man zurück gehen, wenn man die CO2-Konzentration von 1850 in der Vergangenheit wiederfinden will?

    Punkt 9:

    Hier bitte ich um Erklärung, wie es zu einem gleichbleibenden CO2-Grundumsatz bei wachsender Biomasse kommen soll. Unterliegt die Agrarwissenschaft hier einem grundlegenden Irrtum oder werden Treibhäuser wegen des zusätzlichen Treibhauseffektes mit CO2 angereichert?

    Punkt 10:

    Und erst die Auswertung von Tiefseesedimenten – da kommen ja noch viel weniger Leute hin…

    „Ohman ohman.“ In diversen Foren löst dieser Ausruf bei mir einen Fluchtreflex aus. Oder soll dies der Hinweis auf Studien in einem Staat im Persischen Golf sein – etwa wie „Katarrh, Katarrh“?

    Wer so schlicht argumentiert und polemisiert, dem kann ich nur erstaunt zurufen „Mann, o(h) Mann“, ganz eindeutig sollte man sich ausschließlich an die Wissenschaft halten!

  125. S.g. Herr NF
    zur Frage was ich selber erlebt habe:
    „sondern sich auch dazu verstiegen (und das habe ich selber erlebt) „Klimawandelleugner“ mit „Holocaustleugnern“ zu vergleichen.“
    Steht eigentlich dort, diesen Vergleich habe ich mehrfach gehört.
    Zur Frage ob ich ein Recht auf „Rache“ einfordere.
    Nein, es handelt sich nicht um Rache, es handelt sich darum dass das dies eine gesellschaftspolitische Frage ist, in der diese „Päpste“ (eine Erscheinung die man gerade in Deutschland -aber nicht nur- antrifft) quasi aus „unangreifbarer Position“ ihre Erlässe zur Weltrettung verkünden. Kritik (auch spöttische) an der selbstgefälligen Aufgeblasenheit dieser von Bürokraten ermächtigten Herrschaften hat eine deutlich dekonstruierende Funktion. Nach wie vor bilden sich diese Leute ja ein, es gäbe im Grunde keine wissenschaftliche mögliche Position als die ihre – dabei gründet sich ihr „Alleinvertretungsanspruch“ lediglich auf die bornierten Interessen der politischen Kaste, die sich ihrer instrumentalisierend bedient.
    Spott ist da Herrschaftskritik, lesen sie mal Lichtenberg.
    Das ist nicht neu, die Geschichte ist voll damit – neu ist vielleicht nur, dass gerade unabhängig dünkende Naturwissenschaftler sich da dienstbar machen.

  126. Lieber Herr Boos,

    plädieren Sie hier für eine „Recht auf Rache“ auf Kosten der Wissenschaft?

    „verhindert, sämtliche Forschungsmittel monopolisiert – sondern sich auch dazu verstiegen (und das habe ich selber erlebt) “

    Erzählen Sie mal bitte, was haben Sie selber erlebt?

  127. Unter den Vorrednern sind einige, die den leicht polemischen Tonfall der EIKE-Kommentatoren bekritteln.
    Dazu sollte man doch folgendes festhalten:
    Die AGW-Befürworter haben über etliche Jahre jeden Andersdenkenden als Dilettanten bezeichnet, alternative Veröffentlichungen verhindert, sämtliche Forschungsmittel monopolisiert – sondern sich auch dazu verstiegen (und das habe ich selber erlebt) „Klimawandelleugner“ mit „Holocaustleugnern“ zu vergleichen. Da gab es nicht nur Vorschläge für „schwarze Listen“ (die es höchstwahrscheinlich gab und gibt), besonders fanatische selbsternannte „Weltretter“ forderten sogar strafrechtliche Verfolgung für Vertreter skeptischer Ansichten. Dieser Umschlag angeblich aufgeklärter Rationalisten in inquisitorische Methoden ist ja höchst aufschlussreich.
    Kurzum, wer sich so verhalten hat (und sich weiter so verhalten würde, wenn die Kritik nicht langsam unüberhörbar würde)sollte nicht ganz so empfindlich über ein wenig Spott sein.
    Wer sich selber (ein wenig wie ein Kleinkind) als „Wissender“ oder „real“-Wissenschaftler bezeichnet muss schon mit ein wenig Spott leben können.
    Und jene, die früher zu dieser inquisitorischen Stimmmung geschwiegen haben, haben nicht sonderlich viel Recht heute ihre „Grossfürsten“ des Klimawandels (die nahezu unbegrenzte Mittel zur Verfügung haben) als unter unsachlichen Angriffen leidend hinzustellen.

  128. Von diesem Text bin ich doch sehr enttäuscht. Abgesehen von den ständigen, untergriffigen Bemerkungen (ist das denn notwendig?), sind mir einige Ungereimtheiten aufgefallen. Exemplarisch möchte ich den ersten und letzten Punkt anmerken:

    zu Punkt 1: ich gratuliere Ihnen zu Ihrer Seeschärfe, wenn sie in der Grafik 1 von Rahmsdorf die Temperaturentwicklung im Zeitraum von 2-3 Jahrzehnten ablesen konnten. Auf meinem 13-Zoller entsprechen 5.000 Jahre auf der X-Achse einer Länge von 1 cm. 25 Jahre entsprechen bei mir also etwa 25 Mikrometern. War die Grafik im Spectrum der Wissenschaft größer abgebildet, etwa als herausnehmbares A0-Poster in der Heftmitte?

    Punkt 10: Sie vergleichen ernsthaft den Temperaturanstieg in EINER Höhle in Tirol mit dem globalen Temperaturanstieg?

    Wenn Sie bei Ihren Texten den Maßstab ansetzen würden, den Sie beim IPCC anlegen, dann ist dieser Artikel mit Bomben und Granaten durchgefallen. Womöglich will uns EIKE genauso arglistig täuschen, so wie Sie das vom IPCC annehmen?

  129. Wissenschaftlich ein wertloser Text; aus folgenden Gründen:

    Punkt 1: Wie kommen Sie darauf, der der Temperaturanstieg seit 1960 gefälscht ist? Welche Messungen widersprechen den Analysen von HadCRU/GISS? Mir sind keine bekannt. Ihnen scheinabr auch nicht, sonst hätten Sie wohl ausgeführt, was gefälscht wurde und wie Sie zu dieser Behauptung kommen.

    Punkt 2: Haben Sie Knorr gelesen? In der Arbeit geht es um die airborne fraction, also um den Anteil des anthropogenen CO2 Ausstosses der in der Atmosphäre bleibt. Dieser ist nach Knorr etwa konstant, ca. 50%. Das bedeutet natürlich nicht, dass sich das anthropogene CO2 nicht in der Atmosphäre akkumuliert, ganz im Gegenteil. Ansonsten vermisse ich eine Erklärung von Ihnen, warum die Ozeane nicht Netto-Senken sein sollten? Was die heutige Situation – um die es geht – mit der Lage vor einigen hunderttausend Jahren zu tun hat, bleibt ebenfalls schleierhaft. Auch vergessen Sie zu erwähnen, dass der Partialdruck von CO2 einen grossen EInfluss hat. Übrigens nimmt die Löslichkeit von CO2 in Wasser bei Temperaturanstieg schon ab, die Frage ist aber um wie viel: Nach einer neuen Studie – wurde hier x-Mal verlinkt, als Beweis, dass die Modelle das CO2 feedback überschätzen – sind es nur ca. 8 ppm/°C. Der Effekt durch den höheren Partialdruck ist derzeit noch viel höher.

    Punkt 3: Sie verwechseln Zeiträume. Der starke CO2 Anstieg in den letzten 200 Jahren ist nicht zu verwechseln mit Prozessen, die über Jahrmillionen ablaufen. Es gibt eine Reihe publizierter Schätzungen zu den CO2 Emissionen vo Vulkanen: Es sind demnach etwa 100 Mio Tonnen im Jahr. Viel weniger als die anthropogenen Co2 Emissionen. Zudem wäre zu erwarten, dass die CO2 Konzentration nach sehr grossen Vulkanausbrüchen einen Sprung nach oben macht, nichts deratiges wurde beobachtet.

    Punkt 4: Auch wieder ganz schwach von Ihnen. Die Studie von Paltridge beruht auf fragwüridgen Daten. Macht man das ganze mit anderen Daten – nach heutigem Stand vertrauenswürdigeren Daten – kommt man zum gegenteiligen Ergebnis. Siehe siehe z.B. Dessler.
    Zur Studie von lindzen gibt es eine umfrangreiche Widerlegung, ist bei Ihnen leider noch nicht angekommen.

    Punkt 5: Leider keine quantitative Abschätzung. Daher wertloses palaver.

    Punkt 6: Wo fallen die globalen Temepraturen? 2009 war das zweitwärmste Jahr. Die 2000er etwa 0,2°C wärmer als die 90er, genau wie vom IPCC projeziert.

    Punkt 7: Die Studie von Menne 2010 ist noch nicht bei euch angekommen? Naja, ist auch noch recht aktuell…So ein Foto ist natürlich viel schöner

    Punkt 8: Mutmassungen ohne Belege. Der heutige CO2 Stand ist der hächste seit 15 Mio jahren (siehe Science Studie vor ein paar Monaten). Aber scheinbar ist das gaaanz natürlich.

    Punkt 9: Leider kommt durch die Atmung Netto kein CO2 hinzu (bisschen schon durch die Transportwege der Lebensmittel etc.), aber den Beitrag hättet ihr euch auf Basis dieser wahrlich einfachen Erkenntnis sparen können.

    Punkt 10: Ohman ohman. Beck, haben Sie sich noch nicht genug ins Abseits geschossen?

    Ein klein wenig solltet Ihr euch schon an die Wissenschaft halten!

  130. @NF #4
    Ihr Zitat:
    —–
    „…dass es einen Konsens unter KLIMAFORSCHERN in den grundsätzlichen Fragen zur Klimatheorie gibt.“
    —–

    Konsens gibt es in der Politik, nicht in der Wissenschaft, und ein Wissenschaftler reicht, um den zu kippen.
    Ich gehe dicher nicht fehl in der Annahme, daß die Zahl der Klima-Forscher, die Marotzke nicht kennt, größer ist als der Konsens von dem er faselt.

  131. Auch wenn Teile dieser Zusammenstellung schon an anderer Stelle auch beschrieben wurden, so ist es doch sehr lehrreich, die wesentlichen Punkte gesammelt zu bekommen.

    Dazu passt auch die heutige Börsenmeldung, die Munich RV (ehem. Münchner Rück) hat 2009 2,5 Mrd. Euro Gewinn erzielt, weil Naturkatastrophen ausgeblieben sind (die zu Zahlungen geführt hätten)

    Deutlicher kann die reale Welt nicht zuschlagen!

  132. Diese Informationen, die im diesem Bericht hier inne sind, sollte eigentlich jedes Tageblatt veröffentlichen, um mal einen guten Lagebericht ihrer Lesern zu präsentieren als diese Erbschuldfanatiker zu befragen. Mir fehlt an dieser Argumentationsart von den Interviewten schon alleine die Motivation auf die Punkte sachlich einzugehen. Ja, was lese ich denn: einerseits, wenn es keine Menschen gäbe wäre alles geschlossen und für die Ewigkeit homogen, glücklich und zufrieden. Anderseits auch wieder nicht, aber dann halt nicht so rasant. Diesen Widerspruch entnehme ich ernsthaft! Liest die Interviews einfach noch mal durch. Ob hypothetisch stets konstant oder leicht variierend, egal, Hauptsache es passt zu einer Schlussfolgerung: Der Mensch ist schuld! Damit hat sich auch die Frage, ob ein politisches Motiv bei ihnen besteht oder ob es tatsächlich eine wissenschaftliche Feststellung ist, von selbst erledigt.

  133. Ich freue mich, dass Sie den Thesen von EIKE die Aussagen von Klimaforschern vorangestellt haben. Diese Fairness war bisher eher unüblich. Schön auch, dass Prof. Marotzke, der gelegentlilch als Beispiel für „andersdenkende, kritische Wissenschaftler“ genannt wurde, klarstellt, dass es einen Konsens unter KLIMAFORSCHERN in den grundsätzlichen Fragen zur Klimatheorie gibt.

    Jeder Leser mit Verstand sollte nun in der Lage sein zu entscheiden, wer seriöser informiert.
    Dass bei EIKE dazu teilweise kreative und kühne Klimmzüge notwendig sind, wird besonders bei Punkt 8 und 9 deutlich.

    Besonders amüsant ist die zweite Abbildung. Gehe ich recht in der Annahme, dass die „statistische Bearbeitung“ der Kurve durch Monckton darin bestand, drei violette Striche zu zeichnen und das Wort „Fälschung“ einzufügen? Wer solche „Fachleute“ braucht, dem ist nicht zu helfen.
    Und noch ein Tipp zu Punkt 1: Sind Temperaturanstiege begrenzt auf 10-20 Jahre schon ein „Klimawandel“? Die Zacken in Abb.1 sehen ja furchterregend steil aus, werden aber langweilig flach, wenn man statt Jahrtausenden einzelne Jahre auf der Zeitachse aufträgt. Netter Versuch!

    So, Schluss mit weiteren Tipps, die restlichen Punkte durchschauen Sie selbst, liebe Leser!

    PS: Schneider, bleib bei deinen Leisten 😉

  134. Gratulation zu Ihrem Artikel. Ich möchte noch einige Anmerkungen zu den Äußerungen von Herrn Marotzke zu den strengen Qualitätsprüfungen der wissenschaftlichen Grundlagen des IPCC Berichts 2007 in Punkt 7 machen. Ich frage mich seit wann Veröffentlichungen des WWFs (mehr als 10 in diesem Bericht zitiert) oder Greenpeace (8 Zitate) oder Berichte in einem Bergsteigerjournal oder Empfehlungen für Antarktistouristen zur Desinfektion ihrer Schuhe begutachtete wissenschaftliche Literatur sind. Auf eine Antwort unserer „Freunde“ NB, NF und Co bin ich gespannt. Ich möchte auch noch mal daran erinnern, daß Herr Rahmstorf vor einigen Jahren in einem FAZ-Interview sich gebrüstet hat, er für schwarze Listen von Wissenschaftlern, die sich skeptisch zur Klimaerwärmung äußern würden. Das traurige daran war bzw. ist, daß kein Politiker und auch keiner der Vorsitzenden der großen Wissenschaftsorganisationen in Deutschland wegen eines solchen Verhaltens kritisiert hat.

  135. Liebe EIKE-Autoren,

    wie wäre es, wenn Sie Ihre Überzeugungen, die ja nun immerhin konkrete Formen annehmen und damit diskutierbar sind, wissenschaftlich veröffentlichen?

  136. Hinweis an EIKE: Im Original fehlen die letzten Buchstabenspalten, was das Verständnis des Textes schwierig macht. Abhilfe: ich habe den Text markiert und als Kopie in ein Word-Dokument eingefügt. Der Text war dann komplett zu lesen!

    Ein herrlicher Text! Vielen Dank!! Und man braucht gar keine Fachkenntnisse, um ihn zu verstehen!

    Chris Frey

Antworten

Deine E-Mail-Adresse wird nicht veröffentlicht.


Wir freuen uns über Ihren Kommentar, bitten aber folgende Regeln zu beachten:

  1. Bitte geben Sie Ihren Namen an (Benutzerprofil) - Kommentare "von anonym" werden gelöscht.
  2. Vermeiden Sie Allgemeinplätze, Beleidigungen oder Fäkal- Sprache, es sei denn, dass sie in einem notwendigen Zitat enthalten oder für die Anmerkung wichtig sind. Vermeiden Sie Schmähreden, andauernde Wiederholungen und jede Form von Mißachtung von Gegnern. Auch lange Präsentationen von Amateur-Theorien bitten wir zu vermeiden.
  3. Bleiben Sie beim Thema des zu kommentierenden Beitrags. Gehen Sie in Diskussionen mit Bloggern anderer Meinung auf deren Argumente ein und weichen Sie nicht durch Eröffnen laufend neuer Themen aus. Beschränken Sie sich auf eine zumutbare Anzahl von Kommentaren pro Zeit. Versuchte Majorisierung unseres Kommentarblogs, wie z.B. durch extrem häufiges Posten, permanente Wiederholungen etc. (Forentrolle) wird von uns mit Sperren beantwortet.
  4. Sie können anderer Meinung sein, aber vermeiden Sie persönliche Angriffe.
  5. Drohungen werden ernst genommen und ggf. an die Strafverfolgungsbehörden weitergegeben.
  6. Spam und Werbung sind im Kommentarbereich nicht erlaubt.

Diese Richtlinien sind sehr allgemein und können nicht jede mögliche Situation abdecken. Nehmen Sie deshalb bitte nicht an, dass das EIKE Management mit Ihnen übereinstimmt oder sonst Ihre Anmerkungen gutheißt. Wir behalten uns jederzeit das Recht vor, Anmerkungen zu filtern oder zu löschen oder zu bestreiten und dies ganz allein nach unserem Gutdünken. Wenn Sie finden, dass Ihre Anmerkung unpassend gefiltert wurde, schicken Sie uns bitte eine Mail über "Kontakt"

*


Diese Website verwendet Akismet, um Spam zu reduzieren. Erfahre mehr darüber, wie deine Kommentardaten verarbeitet werden.